You are on page 1of 103

ĐÁP ÁN ĐỀ ÔN TẬP ĐẠI CƯƠNG VÀ VÔ CƠ (ĐỢT 1)

Thầy yêu cần các bạn nghiêm túc xem lại kết quả của từng bài và những bài không làm được thì tại
sao mình không làm được. TUYỆT ĐỐI KHÔNG làm để đối phó, không quan tâm đúng sai. Học như
vậy thì sẽ luôn thất bại trong mọi kì thi.

ĐỀ SỐ 1
Câu 1 (2,5 điểm) Cấu tạo nguyên tử. Phản ứng hạt nhân. Định luật tuần hoàn.
1.1. Nguyên tử hiđro (H) ở trạng thái cơ bản hấp thụ một photon có bước sóng 904 Å. Năng lượng này có đủ để
tách electron ra khỏi nguyên tử H không? Tính vận tốc electron bay ra (nếu có)?
1.2. Cho số hạt α thoát ra trong một mẫu chứa 1,00 mg một nguyên tố phóng xạ X (t1/2 = 138,4 ngày) bằng số
hạt α thoát ra của một mẫu 226Ra (t1/2 = 1601 năm) có khối lượng 4,55 gam.
a. Xác định khối lượng mol của X.
b. Biết rằng trong hạt nhân đồng vị X, số hạt notron gấp 1,5 lần số hạt proton. Tìm X.
c. Biết rằng một liều thuốc tối thiểu để giết chết một người bình thường là 1μg. Một cơ thể bình thường (70
kg) có hoạt độ phóng xạ tự nhiên (tạo hạt α) là 0,2Bq/kg, giá trị này không đổi trong nhiều năm. Sản phẩm
của phản ứng phân rã X là một đồng vị không có tính phóng xạ. Hãy cho sau bao nhiêu ngày thì việc khai quật
mộ một người bị đầu độc bởi nguyên tố X để xét nghiệm trở nên vô nghĩa (biết hoạt độ phóng xạ đo được nhỏ
hơn 0,3Bq/kg thì không chứng minh được.)
Câu 1 Lời giải Điểm
1 Năng lượng của electron trong nguyên tử H là: 0,25
13,6.Z2 13,6.12
E1   2
 2
 13,6(eV)  13,6.1,6.1019 (J)  2,17872.1018 (J)
n 1
 Năng lượng ion hoá của nguyên tử H là: I1 =  E1 = 2,17872.10-18 J
Năng lượng của photon do nguyên tử H hấp thụ là: 0,125
34 8
hc 6, 625.10 .3.10
E2 = = = 2,198.10-18 (J).
λ 9, 04.108
Vì E2 > I1 nên electron trong nguyên tử H bị tách ra.
Bảo toàn năng lượng: 0,125
2
1 mv
E 2  mv2  I1  = E2 – I1 = 1,928.10-20 J
2 2
2.1,928.1020 0,25
 v2 = 31
= 4,237.1010 (m/s)2  v = 205848 m/s.
9,1.10
2 a. Ta có phương trình phóng xạ của Ra: 0,25
88 Ra  86 Rn  2 He
226 222 4

Do số hạt α thoát ra trong hai trường hợp là như nhau nên số phân rã của X và Rađi 0,25
giống nhau
ln 2 4,55
AX  ARa   N    6, 022 1023  1, 665 1011 Bq
1601 365  24  3600 226
0,25
nX ln 2 nX
AX  X  N X  X   
6, 022 1023 138, 4  24  3600 6, 022  1023
 nX  4, 768  106 mol
mX 1103 0,125
Vậy M X    210
nX 4, 768 106
Z  N X  210 0,125
b. Theo đề bài ta có: X giải ra được :
N X  1,5Z X

210
ZX=84, NX=126. X là 84 Po
c. Hoạt độ phóng xạ của người là: 70  0, 2  14Bq 0,25
Sau thời gian t hoạt độ phóng xạ của người nhiễm thuốc để có thể đo được là
70  0,3  21Bq
Hoạt độ phóng xạ của thuốc là: 21-14=7 Bq.

Mặt khác hoạt độ phóng xạ ban đầu của thuốc được tính theo công thức: 0,25
ln 2 106
A0   N    6,022 1023  1,66 108 Bq
138, 4  24  3600 210
 t 1 A0 138, 4 1,66 108 0,25
A  A  e  t  ln
0
 ln  3391, 4
k A ln 2 7 ngày.
Vậy sau 3391,4 ngày thì việc khai quật mộ một người bị đầu độc bởi nguyên tố X để xét
nghiệm trở nên vô nghĩa.

Câu 2 (2,5 điểm) Cấu tạo phân tử. Tinh thể.


1. Biểu diễn cấu trúc hình học và cho biết trạng thái lai hóa của nguyên tử chlorine (Cl) trong các phân tử và
ion sau: [ClO4]-, F3ClO, [F2ClO2]+, [F4ClO]-
2. a. Sử dụng lí thuyết MO, xây dựng giản đồ năng lượng của ion NO-. Từ đó, viết cấu hình electron của
NO, NO+ và NO-. So sánh độ dài liên kết nitrogen-oxygen trong các tiểu phân trên.
b. Khi cho H+ tác dụng với NO- để tạo thành phân tử HNO. Hãy cho biết H+ sẽ liên kết với nguyên tử
nitrogen hay nguyên tử oxygen trong ion NO-. Giải thích?
3. Tinh thể BaTiO3 được tạo từ các ion Ba2+, Ti4+ và O2-. Các ion Ba2+ và Ti4+ tạo thành mạng lưới lập
phương tâm khối, trong đó ion Ba2+ chiếm vị trí các đỉnh và ion Ti4+ chiếm vị trí tâm hình lập phương. Các
ion O2- phân bố trên tất cả các mặt của hình lập phương.
a. Biểu diễn cấu trúc của một ô mạng cơ sở và cho biết phối trí của các ion trong mạng tinh thể.
b. Khối lượng riêng của BaTiO3 là 6,02 g/cm3 và bán kính của O2- là 1,26 Å. Xác định bán kính của các ion
còn lại trong mạng tinh thể.
Cho biết: M (BaTiO3) = 233 g/mol.
Câu 2 Nội dung Điểm
- 3
1 [ClO4] : Dạng tứ diện. Nguyên tử Cl lai hóa sp . 0,125

F3ClO: Dạng bập bênh. Nguyên tử Cl lai hóa sp3d


0,125

[F2ClO2]+: Tứ diện. Nguyên tử Cl lai hóa sp3


0,125

[F4ClO]-: Tháp đáy vuông. Nguyên tử Cl lai hóa sp3d2


0,125

2 a.
Giản đồ MO của NO-
AO(N) MO(NO) AO(O)

0,25
0,25

0,25

Cấu hình electron của:


NO- σ1s2 σ1s*2 σ2s2 σ2s*2 (πx = πy)4 σz2 (πx*1 = πy*1  Bậc liên kết = 2
NO σ1s2 σ1s*2 σ2s2 σ2s*2 (πx = πy)4 σz2 (πxy*)1  Bậc liên kết = 2,5
NO+ σ1s2 σ1s*2 σ2s2 σ2s*2 (πx = πy)4 σz2  Bậc liên kết = 3
Độ dài liên kết nitrogen-oxygen: NO < NO < NO do bậc liên kết giảm dần
+ -

b.
Khi NO- kết hợp với H+ để tạo thành phân tử HNO thì ion H+ sẽ tương tác với MO bị
chiếm chứa electron độc thân và có năng lượng cao nhất, chính là các MO πx* và πy* .
Sự đóng góp của nguyên tử N với các MO πx* và πy* lớn hơn so với nguyên tử O nên 0,25
H+ sẽ ưu tiên liên kết với nguyên tử N trong ion NO- và tạo thành phân tử HNO.
3 a.
Cấu trúc tinh thể BaTiO3

0,5
Số phối trí của Ti4+ = 6
Số phối trí của Ba2+ = 12
Số phối trí của O2- = 6.
b.
Khối lượng riêng của tinh thể là 6,02 g/cm3 và bán kính của O2- là 1,40Ao.
233,2 g
d  6,02 g.cm 3  a  4,01.108 cm  4,01 Ao
6,022.10 a 23 3

Ta có: 2r(Ti4+) + 2r(O2-) = a  r(Ti4+) = 0,745 Ao 0,5


Mặt khác ta có, 2r(Ba2+) + 2r(O2-) = a 2 = 5,67Ao  r(Ba2+) = 1,575 Ao

Câu 3 (2,5 điểm) Nhiệt hóa học. Cân bằng hóa học trong pha khí.
3.1. Pin nhiên liệu được nghiên cứu rộng rãi nhằm thay thế nguồn nhiên liệu hóa thạch ngày càng cạn kiệt.
Trong pin nhiên liệu, dòng điện được tạo ra do phản ứng oxi hóa nhiên liệu (hydrogen, carbon monoxide,
methanol, ethanol, propane, …) bằng oxygen không khí. Pin sử dụng nhiên liệu lỏng như methanol, ethanol
được đặc biệt quan tâm do có nguồn nhiên liệu sinh học dồi dào và pin hoạt động ở nhiệt độ thường. Trong
pin methanol – oxygen, phản ứng oxi hoá methanol khi pin hoạt động như sau:
2CH3OH(l) + 3O2(g)  2CO2(g) + 4H2O(g)
Cho biết:
(Cho rằng nhiệt dung không đổi trong khoảng nhiệt độ từ 298 K đến 400 K)
a) Tính giá trị năng lượng tự do Gibbs (rG 0 298 ) của phản ứng trên ở điều kiện chuẩn.
b) Tính rG 0400 của phản ứng.
c) Tính lượng nhiệt tỏa ra khi đốt cháy hoàn toàn 80 kg methanol ở 400 K.
d) Một bóng đèn compact công suất 5W được thắp sáng bằng pin nhiên liệu methanol – oxygen. Tính thời
gian (theo giờ) bóng đèn được thắp sáng liên tục khi sử dụng 50,0 gam methanol làm nhiên liệu ở điều kiện
chuẩn. Biết hiệu suất quá trình oxi hóa methanol là 60,0%.
3.2. Đun nóng hỗn hợp khí gồm SO2 và O2 có chất xúc tác, xảy ra phản ứng:
SO2 (g) + O2 (g)    SO3 (g) (*)

a) Tính hằng số cân bằng Kp của phản ứng ở 800C (chấp nhận biến thiên enthalpy, entropy của phản ứng
không phụ thuộc nhiệt độ). Nhiệt độ ảnh hưởng như thế nào tới cân bằng của phản ứng (*)?
b) Trong một thí nghiệm, người ta đưa từ từ oxygen vào một bình dung tích 2 lít chứa 0,05 mol SO2 có chất
xúc tác (thể tích của chất xúc tác không đáng kể) ở 1000C. Khi phản ứng đạt tới trạng thái cân bằng thì 0,03
mol SO3 được tạo thành, áp suất tổng của hệ là 1 atm. Tính Kp.
c) Cân bằng (*) sẽ chuyển dịch như thế nào trong trường hợp sau:
+ Cho một lượng helium (He) vào bình phản ứng để áp suất khí trong bình tăng gấp đôi?
+ Giả thiết thể tích khí trong bình tăng gấp đôi, lượng He cho vào bình phản ứng chỉ để giữ cho áp suất tổng
không đổi?
Cho các dữ liệu nhiệt động sau:

Chất H 298
0
(kJ.mol-1) S 298
0
(J.K-1.mol-1)
SO3 (g) -395,7 256,7
SO2 (g) -296,8 248,1
O2 (g) 0,0 205,0

Câu 3 Nội dung Điểm


1

0,25

Áp dụng công thức của định luật Kirchhoff:

0,25
c) Ở 400 K, cứ 64 gam CH3OH bị oxi hoá hoàn toàn thì toả ra một lượng nhiệt là
1274,59 kJ.
Vậy, nếu có 80.103 gam CH3OH bị oxi hoá hoàn toàn thì lượng nhiệt toả ra là 0,25

d) Số mol CH3OH bị oxi hóa là

 Năng lượng để thắp sáng (công có ích thực hiện trong khoảng thời gian t) là

 Thời gian để bóng đèn được thắp sáng liên tục là


0,25

2 

SO2 (g) + O2 (g) 
 SO3 (g) (*)
PSO3
Hằng số cân bằng : K p =
PSO2 .(PO2 )1/2
Ta có: Δ r G T0 =Δ r H T0 -T.Δ rST0 =-RTlnKp
Ở 250C: Δ r G 0298 =Δ r H 0298 -298.Δ rS0298
1
Δ r G 0298  (395, 7  296,8)  298.103 (256, 7  248,1  .205, 0) = -70,92 (kJ.mol-1). 0,25
2
-ΔG 0298 -
-70,92
-3
K p,298 =e RT
=e 8,314.10 .298
=2,7.1012
Khi Δ r H 0 = const, ta có:
K p,353 (98,9) 1 1
ln 12
 3
(  )  Kp,353 = 109,73 (atm-1/).
2,7.10 8,314.10 353 298
Khi tăng nhiệt độ tử 250C đến 800C, hằng số cân bằng Kp giảm, điều này hoàn toàn
phù hợp với nguyên lý Le Chatelier, do phản ứng thu nhiệt (chiều nghịch) làm tăng áp
suất của SO2 và O2, giảm áp suất riêng SO3 dẫn đến hằng số Kp giảm.
b. Tổng số mol của hệ:
PV 1.2
nhệ= = = 0,065 (mol). Tại thời điểm cân bằng:
RT 0,082.373
nSO3  0, 03(mol ); nSO2  0, 05  0, 03  0, 02( mol )
nO2  0, 065  0, 03  0, 02  0, 015(mol )
0,5
Vì áp suất tổng cùa hệ là 1 atm, do đó:
0, 03
pSO3 0, 065
Kp   3,12(atm1/2 )
pSO2 po2 0, 02 0, 015
0, 065 0, 065
c.
+ Nếu áp suất tăng gấp đôi do thêm He, nhưng thể tích không đổi, áp suất riêng phần
của các chất khí không đổi, do đó cân bằng không bị chuyển dịch.
+ Nếu áp suẩt tổng trong bình không đổi, nhưng giả thiết thể tích bình tăng gấp đôi, khi
đó sẽ làm giảm áp suất riêng phần của các chất (loãng khí), cân bằng (*) sẽ chuyển dịch 0,5
theo chiều nghịch.

Câu 4 (2,5 điểm) Động hóa học (không có cơ chế).


4.1. Quá trình phân huỷ hợp chất A2B theo phương trình sau: 2A 2 B 
k
 2A 2  B2 có phương trình tốc độ
phản ứng là v  k.C2A2 B . Dữ kiện động học của phản ứng được cho như bảng dưới đây:
Nhiệt độ (K) 967 967 1030 1030
Nồng độ đầu A2B (mmol.dm-3) 156 39 7 49
Thời gian nửa phản ứng của A2B (s) 380 1520 1442 206

(a). Tính năng lượng hoạt hoá Ea và hằng số tốc độ phản ứng k tại 967 K và 1030 K.
(b). Khi nồng độ đầu của A2B ở 1030 K là 54 mmol.dm-3, tính thời gian cần thiết để phản ứng chuyển hoá hết
37% A2B.
4.2. Cho phản ứng thuận nghịch bậc 1 – 1 giữa A và B như sau:

k
A   B có k = 300 s-1, k’ = 100 s-1.
k'
Ở thời điểm t = 0, chỉ có chất A mà không có chất B. Trong thời gian bao lâu thì một nửa lượng chất A chuyển
thành chất B?
Câu 4 Nội dung Điểm
4.1 a) Xét phản ứng bậc 2:

2A 2 B  k
 2A 2  B
t=0 C0 0 0
t C C0 – C (C0-C)/2
1 1 1 1 1
v  v A2B  kCA2 2B ;   2kt  k ' t  t 12  
2 C C0 2kC0 k 'C0
k' là tốc độ tiêu thụ của A2B.
1 1
k 967K    8,392  106 (dm3 .mmol1.s 1 )
2C0 .t 12 2  156,787  380
1 1
k1030K    4,914  105 (dm3 .mmol1.s 1 )
2C0 .t 12 2  7,066  1440 0,5

k2 E  1 1  k   1030  967  1 0,25


ln   a     E a  R  ln 2     232,34(kJ.mol )
k1 R  T2 T1   k1   1030  967 

b) Tại thời điểm t:


C = C0(1-y) = 54(1-0,37) = 34,02 (mmol.dm-3)
11 1  1 1 1  1  1 1 
t         110,7(s)
k '  C C0  2k  C C0  2  4,914 10  34,02 54,0 
5  0,5

4.2 k

A  B
k'
[B]eq
(k + k’)t  ln
[B]eq  [B]
[B]eq [B]eq [A]0 K
K   [B]eq  0,25
[A]eq [A]0  [B]eq 1+K
Tại thời điểm một nửa lượng chất A đã tham gia phản ứng:
[A]0 [A]0 K [A]0
[B]eq  [B] = [B]eq  = 
2 1+K 2
2[A]0 K  [A]0  [A]0 K [A]0 (K  1)
= =
2(1 + K) 2(1 + K)

 k + k’  2,303 lg 2K  t1/2 = 2,303 lg 2K


t1/2 K  1 k1 + k 2 K  1 0,5
k
Vì K  , nên:
k'
2,303 2k 2,303 2 . 300 = 2,75.10-3 (s) 0,5
t1/2 = lg = lg
k + k' k - k' 300 + 100 300 - 100
Câu 5 (2,5 điểm) Cân bằng acid – base và cân bằng ít tan.
Giả thiết dung dịch X gồm H3PO4 0,100M và CH3COOH 0,010M.
a) Tính pH của dung dịch X.
b) Tính độ điện li của CH3COOH trong dịch X.
c) Có kết tủa tách ra không khi trộn 1,00 mL dung dịch X với 1,00 mL dung dịch BaCl2 0,050M? Giải thích
bằng tính toán.
d) Trộn 3,00 mL dịch X với 2,00 mL dung dịch NaOH 0,465M, thu được dung dịch Y. Thêm rất từ từ từng
giọt BaCl2 0,050M vào 1,00 mL dung dịch Y cho tới dư. Bằng tính toán cho biết: Có kết tủa tách ra không?
Nếu có, cho biết kết tủa gồm những chất gì? Giả sử không có sự đồng kết tủa (cộng kết). Cho biết: pKai
(H3PO4) = 2,15; 7,21; 12,32; pKa (CH3COOH) = 4,76; pKw (H2O) = 14,00. pKs (Ba3(PO4)2) = 22,50; pKs
(BaHPO4) = 7,40.
Câu 5 Lời giải Điểm
a) Các cân bằng xảy ra trong dung dịch X:

0,5

0,25

0,25
0,5

0,5

Xét thứ tự kết tủa: 0,5

Vậy kết tủa Ba3(PO4)2 tách ra trước. Vì thêm từ từ từng giọt BaCl2 0,050M vào 1,00
mL dung dịch Y cho tới dư nên:

PO43- sẽ bị kết tủa hết, do đó chỉ có kết tủa Ba3(PO4)2 tách ra mà không có kết tủa
BaHPO4 tách ra.
Câu 6 (2,5 điểm) Phản ứng oxi hóa – khử. Pin điện (không liên quan phức chất).
Cho các nửa phản ứng của các ion nguyên tố Vanadi và thế khử chuẩn tại 298K:
VO2+(aq) + 2H+(aq) + e  VO2+(aq) + H2O E0 = + 1,00 V
VO2+(aq) + 2H+(aq) + e  V3+(aq) + H2O E0 = + 0,34 V
V (aq) + e  V (aq)
3+ 2+
E0 = - 0,26 V
1.Tính E0 cho nửa phản ứng sau ở 298 K:
VO2+(aq) + 4 H+(aq) + 3 e → V2+(aq) + 2 H2O(l)
2. Pin vanadi có thể được chế tạo bằng cách khử vanadi (V) bằng vanadi (II). Viết phương trình phản ứng khi
pin hoạt động. Tính ∆E0 cho pin ở 298 K?
3. Giá trị ∆E0 của pin vanadi tăng khi nhiệt độ tăng 1,76×10-4 V.K-1. Tính toán ∆H0 và ∆S0 đối với pin vanadi.
4. Pin vanadi được thiết lập như hình bên, sử dụng
các dung dịch được đệm ở pH = 1,00. Sau đó cho pin
phóng điện với dòng điện không đổi 10,0 A đến khi
điện thế của tế bào đạt 1,14V. Nhiệt độ được duy trì
ở 298 K và thể tích dung dịch trong mỗi cốc là 100,0
mL.
a. Nồng độ V3+ (aq) trong tế bào anot là bao nhiêu
khi điện áp của tế bào đạt 1,14V?
[VO2+] = 0,50M [V2+] = 0,50M
b. Cần bao nhiêu thời gian để đạt được hiệu điện thế [VO2+] = 0,10M [V3+] = 0,10M
này?

Câu 6 Nội dung Điểm


1 VO2+(aq) + 2H+(aq) + e  VO2+(aq) + H2O E10 = + 1,00 V
VO2+(aq) + 2H+(aq) + e  V3+(aq) + H2O E20 = + 0,34 V
V (aq) + e  V (aq)
3+ 2+
E30 = - 0,26 V
Từ phương trình G0 = -nFE0; G là hàm trạng thái  Cộng 3 phương trình trên 0, 5
được
VO2+(aq) + 4 H+(aq) + 3 e → V2+(aq) + 2 H2O(l)
1.E01 +1.E02 + 1.E03
E0 = = +0,36V
3
2 VO2+(aq) + 2H+(aq) + V2+(aq)  VO2+(aq) + V3+(aq) + H2O
0,5
E0 = 1,00 - (-0,26) = 1,26V
3 ∆H0 ∆S0
G0 = -nFE0 = H0 - T. S0 do vậy E0 = - + T.
nF nF
∆S0 𝜕(∆𝐸 0 ) ∆(∆𝐸 0 )
 nF = 𝑑𝑇 ≈ ∆𝑇 = 1,76.10  S = 17,0 J.mol-1.K-1.
-4 0
0,5
Tại 298K: G0 = -nFE0 = -121,6 kJ.mol-1 = H0 - T. S0
 H0 = -117 kJ.mol-1
4 0,0592 [VO2+ ].[V3+ ]
a) Theo phương trình Nernst: E = E0 - .log([VO+].[V2+].[H+]2 )
n 2
[VO2+ ].[V3+ ]
1,14V = 1,26 V - 2.(0,0592).pH - 0,0592. log( [VO+].[V2+] )
2
[VO2+ ].[V3+ ]
 log( [VO+].[V2+] ) = 0 0,5
2
Từ nồng độ đầu của các chất và thể tích ở hai cực  khi E = 1,14V thì nồng độ
các ion bằng 0,3M. Vậy [V 3+ ] = 0,3M.
b) Vì [V3+] tăng 0,20 M trong dung dịch 0,100 L, điều này tương ứng với sự đi qua
của
(1)×(96500 C.mol-1)×(0,20mol.L-1)×(0,100L) = 1930 C. 0,5
Ở cường độ dòng điện 10,0 A  t = 193 giây.

Câu 7 (2,5 điểm) Halogen. Oxygen – Sulfur.


1) Các hợp chất X, Y, Z đều cấu tạo gồm các nguyên tố Na, S, O trong đó MZ – MY = MY – MX = 16. Khử
Y bằng carbon (C) ở nhiệt độ cao rồi cho sản phẩm vào dung dịch HCl thu được một chất khí mùi trứng thối.
Khí này tác dụng với dung dịch HClO thu được sản phẩm chứa sulfur (S) có cùng số oxi hóa với sulfur (S)
trong Y.
Từ dung dịch X có thể trực tiếp điều chế Z bằng cách hòa tan vào Z một đơn chất, sau đó cô đặc dung dịch và
kết tinh để thu được một tinh thể ngậm 5 phân tử nước. Lọc vớt tinh thể rồi làm khô, đun nóng nhẹ được dung
dịch chứa Z với nồng độ C%.
a) Xác định X, Y, Z và viết các phương trình phản ứng minh họa.
b) Tính giá trị của C.
2) Acid florosulfuric có công thức là HSO3F là một axit mạnh. Ở trạng thái lỏng, HSO3F tự phân ly theo cân
bằng sau:
2HSO3F [HOSO2FH]+ + [SO3F]-
a) Xác định cấu trúc của HSO3F, [H2SO3F]+ và [SO3F]-
b) So sánh lực acid của HSO3F và H2SO4. Giải thích.
Hướng dẫn chấm:
Câu Hướng dẫn trả lời Điểm
1a Khí mùi trứng thối là H2S. Khi H2S tác dụng với HClO, sản phẩm sinh ra là H2SO4. Vậy
lưu huỳnh trong Y có số oxi hóa là +6.
Y là Na2SO4.
Na2SO4 + 4C → Na2S + 4CO 0,5
Na2S + 2HCl → 2NaCl + H2S
H2S + HClO → H2SO4 + HCl
MZ – MY = MY – MX = 16.
Vậy Z phải là Na2S2O3, X là Na2SO3
Na2SO3 + S → Na2S2O3 0,5
Na2S2O3 + 5H2O → Na2S2O3..5H2O
1b Khi đun nóng, Na2S2O3.5H2O nóng chảy trong nước kết tinh được dung dịch với 0,5
C% = (46+64+48)/(46+64+48+18.5) = 63,71%
2 Cấu trúc của HSO3F
O
0,25
S O H
O
F
Cấu trúc của [H2SO3F]+
O +
0,25
S O H
O
F H

Cấu trúc của [SO3F]-


-
O
02,5
S O
O
F
Lực axit HSO3F > H2SO4 vì nguyên tử F có độ âm điện lớn hơn nguyên tử O do đó liên
kết O-H trong HSO3F phân cực mạnh hơn liên kết O-H trong phân tử H2SO4. 0,25

Câu 8 (2,5 điểm) Đại cương hữu cơ (Quan hệ giữa cấu trúc và tính chất)
1.1. Sắp xếp các chất sau theo thứ tự giảm dần lực base và giải thích.

1.2. Cho các chất sau đây:

So sánh lực acid giữa (4) và (5). Giải thích.


1.3. So sánh và giải thích nhiệt độ sôi của các hợp chất sau:

1.4. So sánh và giải thích nhiệt độ nóng chảy của các hợp chất (9), (10), (11), (12).
Câu 8 Đáp án Điểm
1.1. - Thứ tự giảm dần lực bazơ: (2) > (1) > (3). 0,125
- Giải thích:
+ (2) có hiệu ứng +I gây ra bởi gốc hidrocacbon no (amin bậc III mạch
vòng) nên tính chất bazơ mạnh nhất. 0,125
+ Chất (1): Có hiệu ứng +I của gốc hidrocacbon no và hiệu ứng –I
của vòng benzen, nên lực bazơ của chất (1) yếu hơn lực bazơ của chất
(2). 0,125
+ Chất (3): Có hiệu ứng +I của gốc hidrocacbon no và hiệu ứng –C
giữa nguyên tử N với vòng benzen, tạo thành hệ liên hợp p-π, làm mật
độ electron trên N giảm mạnh →Lực bazơ giảm mạnh. Chất (1) không
có hiệu ứng – C do cấu trúc không đồng phẳng nên lực bazơ của chất
(1) mạnh hơn lực bazơ của chất (3). 0,125
1.2. - Lực axit của (5) mạnh hơn lực axit của (4). 0,125
- Giải thích: Vì (5) có khả năng tách proton H+ tạo hệ có tính thơm.
0,25

(5)
0,125
1.3. - Thứ tự nhiệt độ sôi của các chất: to sôi (7) > to sôi (8) > to sôi (6). 0,25
- (8) không có liên kết hiđro, (6) có liên kết hiđro nội phân tử, M(8) >
M(6) nên to sôi (8) > to sôi (6) 0,25
- (7) có liên kết hiđro liên phân tử nên có nhiệt độ sôi cao nhất.
0,25
1.4. - Thứ tự nhiệt độ nóng chảy của các chất: tonc
(12) > tonc
(11) > tonc
(10) 0,125
> tonc (9).
- (12) ở dạng ion lưỡng cực nên có nhiệt độ nóng chảy lớn nhất. 0,125

- (10) và (11) đều có khối lượng lớn hơn và nhiều liên kết hidro liên
phân tử hơn (9) nên nhiệt độ nóng chảy của (10), (11) cao hơn (9). 0,125
- Nhóm NH2 ở (11) vừa làm tăng momen lưỡng cực, vừa tạo nhiều liên
kết hiđro liên phân tử hơn so với nhóm NO2 ở (10) vì vậy nhiệt độ 0,125
nóng chảy của (11) cao hơn của (10).

ĐỀ SỐ 2
Câu 1. (2,5 điểm) Cấu tạo nguyên tử. Phản ứng hạt nhân. Định luật tuần hoàn.
1. a) Viết cấu hình electron của nguyên tử O (Z= 8), xác định bộ 4 số lượng tử cho electron cuối cùng của
nguyên tử oxi.
b) Áp dụng biểu thức gần đúng Slater, hãy tính năng lượng của các electron phân lớp, lớp và toàn nguyên
tử Oxi theo đơn vị eV.
c) Ở nhiệt độ rất cao, nguyên tử oxy có thể bị ion hóa và tồn tại dưới dạng ion O7+. Dựa vào công thức
tính năng lượng electron của Bohr.
Z2
En  13, 6  2 (eV )
n
Hãy tính bước sóng của bức xạ phát ra khi electron trong ion O 7+ dịch chuyển từ mức năng
lượng có n = 3 xuống mức có n=1.
Cho biết vận tốc ánh sáng C = 3,000.108 m.s-1; Hằng số Planck: h = 6.62×10–34 J.s
2. 210Po là một trong các đồng vị phóng xạ của nguyên tố polonium. 210Po phân rã , tạo thành đồng vị bền
206
Pb với chu kì bán rã 138,4 ngày.
a) Một thiết bị phát hiện được độ phóng xạ nhỏ nhất là 10-4 Ci. Tính lượng 210Po nhỏ nhất (theo gam)
mà thiết bị này có thể phát hiện được.
b) Chất thải phóng xạ chứa 210Po được coi là an toàn với môi trường khi độ phóng xạ không vượt quá
0,25 Ci. Một mẫu chất thải ban đầu chứa 0,10 mg 210PoCl2 được coi là an toàn phóng xạ sau thời gian ngắn
nhất bao nhiêu ngày?
Câu 1 Hướng dẫn chấm Điểm
1 1. a, Ở trạng thái cơ bản cấu hình electron của 8O là: 1s22s22p4
hay [2He] 2s22p4
Bộ 4 số lượng tử cho electron cuối cùng của 8O: 0,5
n = 2; l = 1; ml = -1; ms= -1/2.
( Z  b) 2
En  13, 6  (eV )
b. Theo slater (n* ) 2
Năng lượng của các electron trên phân lớp 1s:
(8  0,3) 2
E1s  13, 6   2  1612, 688(eV )
12 0,5
Năng lượng của các electron trên phân lớp 2s2p:
8  (5  0,35  2  0,85)
2

E2 s 2 p  13, 6   6  422,331(eV )
22
Năng lượng của toàn nguyên tử oxi:
EO  E1s  E2 s 2 p  1612, 688  422,331  2035, 019(eV )
c. Khi electron trong O7+ dịch chuyển từ mức năng lượng có n = 3 xuống mức có
n=1 phát ra một bức xạ có năng lượng: 0,5
2 2
C 8 8
h   E3  E1  13, 6  ( 2  2 ) 1, 602 1019 J  1, 24 1016 J
 3 1
Bước sóng của bức xạ là:
hC 6, 62 1034  3, 000.108
   1, 602 109 m  1, 602(nm)
1, 24 1016 1, 24 1016
Câu 1
2 a) Mối liên hệ giữa độ phóng xạ và khối lượng 210Po

Khối lượng 210Po nhỏ nhất thiết bị đó phát hiện được là


t .M . Amin (138, 4.24.3600).210.(104.3, 7.1010.10 6 ) 0.5
mmin  1/2   2, 23.1014 ( gam)
ln 2.N A ln 2.6, 022.1023
A t t A
b) Mối quan hệ giữa thời gian và độ phóng xạ: ln 0  ln 2.  t  1/2 .ln 0
A t1/2 ln 2 A
Thời gian ngắn nhất để mẫu thải được coi là an toàn phóng xạ là:
t A
tmin  1/2 .ln 0
ln 2 Amax
- Tính A0:
ln 2 m0 ( 210 PoCl2 )
A0  k .N 0  . .N A
t1/2 M ( 210 PoCl2 )
ln 2 0,1.103
= . .6, 022.1023  1, 242.1010 Bq  0,336Ci
138, 4.24.3600 210  71
138, 4 0,336
tmin  .ln  2817, 6 (ngày) 0.5
ln 2 0, 25.106

Câu 2. (2,5 điểm) Cấu tạo phân tử. Tinh thể


Trong mạng tinh thể của Beri borua, nguyên tử Bo kết tinh ở mạng lưới lập phương tâm mặt và trong đó tất
cả các hốc tứ diện đã bị chiếm bởi nguyên tử beri. Khoảng cách ngắn nhất giữa 2 nguyên tử Bo là 3,29A0.
1. Vẽ hình biểu diễn sự chiếm đóng của nguyên tử Bo trong một ô mạng cơ sở.
2. Có thể tồn tại bao nhiêu hốc tứ diện, hốc bát diện trong một ô mạng? Từ đó cho biết công thức thực
nghiệm của hợp chất này ( công thức cho biết tỉ lệ nguyên tử của các nguyên tố). Trong một ô mạng
cơ sở có bao nhiêu đơn vị công thức trên?
3. Cho biết số phối trí của Be và Bo trong tinh thể này là bao nhiêu?
4. Tính độ dài cạnh a0 của ô mạng cơ sở , độ dài liên kết Be-B và khối lượng riêng của beri borua theo
đơn vị g/cm3. Biết Be: 10,81 ; Bo 9,01
Câu Hướng dẫn chấm Điểm
2 1. 0,5
A

B
A
2. Có 8 hốc tứ diện, và 4 hốc bát diện. 0,75
Mỗi nguyên tử Be chiếm một hốc tứ diện nên trong một ô mạng có 8 nguyên tử Be.
NB= 8*1/8 + 6*1/2 = 4
NB : NBe = 1:2 nên công thức thực nghiệm của hợp chất này là Be2B.
Trong một ô mạng chứa 4 đơn vị công thức trên (Be8B)
3. Số phối trí của Be = 4; số phối trí của B = 8 0,5
0 0,75
4. a0 2 = 2*3,29 a0 = 4,65 A
1
Độ dài liên kết Be-B = a0 3 = 2,01A0
4
  m/V = 8 * 9,01  4 *10,81 *
8 3
1
23
= 1,90 gam/cm3
(4,65 *10 ) 6,022 *10

Câu 3 (2,5 điểm) Nhiệt hóa học. Cân bằng hóa học trong pha khí.
1. Hãy cho biết phản ứng 2Ni(l) + O2(k) → 2NiO(r) ở 1627 oC có thể tự diễn biến theo chiều thuận được không
nếu áp suất riêng phần của oxi nhỏ hơn 150 Pa?
Cho: G hình thành (NiO) ở 1627 oC là -72,1 kJ. mol–1; Áp suất chuẩn P0 = 1,000.105 Pa;
0

0oC trong thang Celsius là 273,15 K.


2. Người ta tiến hành tổng hợp NH3 với sự có mặt chất xúc tác Fe theo phản ứng sau:
1 3
N 2 + H 2 
 NH3
2 2
Khi tổng hợp tỉ lệ mol N2 và H2 là 1 : 3. Trong quá trình tổng hợp chúng ta thu được các số liệu thực
nghiệm sau:

Nhiệt độ Ở Ptổng = 10 atm Ở Ptổng = 50 atm


Lượng % NH3 chiếm giữ Lượng % NH3 chiếm giữ
350oC 7,35 25,11
450oC 2,04 9,17

a. Xác định Kp theo số liệu thực nghiệm của bảng trên.


b. Tính giá trị ΔH của phản ứng ở Ptổng đã cho.
Câu 3 Hướng dẫn chấm Điểm
1. Từ phản ứng: 2Ni (l) + O2 (k) 2NiO (r) (1) 0,5
ta có: G phản ứng = -72,1.2 = -144,2 kJ/mol = -144200 J/mol
0

ΔG 0 -144200
 lnK = - =- = 9,127
RT 8,3145.1900,15
 K = 9200,38.
K K 0,5
Đối với phản ứng (1): Δn (k) = -1  K = Δnp(k)  -1p
P0 P0
→ Kp = K.P0 = 9200,38.(1,000.10 )
-1 5 -1

1 1,000.105
Mặt khác: K p = với p O2 là áp suất cân bằng của oxi  pO2 = =
p O2 9200,38
10,87 (Pa)
Vậy phản ứng có xảy ra nếu 10,87Pa < p O2 < 150Pa.

2.a. 1 3 0,25
N 2 + H 2 
 NH3
2 2
PNH3
Hằng số cân bằng Kp được xác đinh theo biểu thức: K p = 1/2
P
N2 . PH3/22
* Tại 350oC: Ptổng = 10 atm
Theo đề, tại cân bằng lượng NH3 chiếm 7,35% nên PNH3  0,735 atm
→ PN2  PH2  9, 265 atm
Mặt khác lượng N2 và H2 ban đầu lấy theo tỉ lệ 1: 3 nên PN2 = 2,316 atm và 0,25
PH2  6,949 atm
0,735
Do đó: K p1-1 = 1/2 3/2
 2,64.102
(2,316) . (6,949)
0,25
* Tại 350oC: Ptổng = 50 atm
Tại cân bằng lượng NH3 chiếm 25,11% nên PNH3  12,555 atm 0,25
→ PN2  PH2  37, 445 atm
PN2 = 9,361 atm và PH2  28,084 atm
12,555
Do đó: K p1-2 = 1/2 3/2
 2,76.102
(9,361) . (28,084)

* Tại 450oC: Ptổng = 10 atm


PNH3  0, 204 atm ; PN2 = 2,449 atm và PH2  7,347 atm
0,204
Do đó: K p2-1 = 1/2 3/2
 6,55.103
(2,449) . (7,347)

* Tại 450oC: Ptổng = 50 atm


PNH3  4,585 atm ; PN2 = 11,354 atm và PH2  34,061 atm
4,585
Do đó: K p2-2 = 1/2 3/2
 6,84.103
(11,354) . (34,061)

2.b. Tại áp suất tổng Ptổng = 10 atm: 0,5


K p2-1 ΔHo 1 1
ln  (  )
K p1-1 R T1 T2
6,55.103 ΔHo 1 1
Thay số: ln 2
 (  )
2, 64.10 8,314 623 723
 ΔHo = ‒52,199 J.mol‒1
Tại áp suất tổng Ptổng = 50 atm:

K p2-2 ΔHo 1 1
ln  (  )
K p1-2 R T1 T2
6,84.103 ΔHo 1 1
ln 2
 (  )
2, 76.10 8,314 623 723
 ΔHo = ‒51,613 J.mol‒1

Câu 4 (2,5 điểm) Động hóa học (không có cơ chế).


1. Ở 3100C sự phân hủy XY3 (khí) xảy ra theo phản ứng :
2XY3 (khí)   2X (rắn) + 3Y2 (khí) (1)
Theo thời gian phản ứng, áp suất chung của hệ đo được là:
t (giờ) 0 5,5 6,5 8
P (mmHg) 733,32 805,78 818,11 835,34
Hãy chứng minh phản ứng trên là bậc 1. Tính hằng số tốc độ của phản ứng và thời gian nửa phản ứng.
2. Khi tiến hành phản ứng trên ở nhiệt độ T, sau 1 giờ, áp suất tổng cộng trong bình tăng 10% so với lúc bắt
đầu (giả sử bậc phản ứng không đổi)
a. Tính hằng số tốc độ của phản ứng ở nhiệt độ T.
b. Tính hệ số nhiệt độ của phản ứng, biết năng lượng hoạt hóa của phản ứng là 249,72 kJ/mol.
Hướng dẫn chấm ĐIỂM
1. Giả sử phản ứng bậc 1. Vì áp suất tỉ lệ với nồng độ chất nên phương trình động học có thể biểu
1 P
diễn theo áp suất riêng phần: k = ln 0
t Pt
Với P0 là áp suất đầu của XY3, Pt là áp suất của XY3 ở thời điểm t.
Gọi 3x là áp suất riêng phần của Y2 ở thời điểm t, ta có:
2XY3 (khí)   2X (rắn) + 3Y2 (khí) (1)
Ban đầu P0 0 0
t P0 - 2x 2x 3x
p Y2 = 3x và Pt = P0 – 2x.
Pchung = P0 + x  x = Pchung - P0
Pt = P0 – 2x = P0 – 2.(Pchung - P0) 0,75
  Pt = 3P0 – 2Pchung
1 P
Từ k = ln 0 ta có:
t Pt
1 P0
k = ln
t 3P0 - 2Pchung
Thay số: k1 = 0,04 giờ-1 ; k2 = 0,04045 giờ-1; k3 = 0,04076 giờ-1;
k1  k2  k3. Vậy phản ứng (1) là phản ứng bậc nhất.
Hằng số tốc độ trung bình của phản ứng là:
1 0,5
k  (0,04 + 0,04045 + 0,04076) = 0,0404 giờ-1 .
3
Thời gian nửa phản ứng của phản ứng (1) là:
0,693 0,693
t1/2 = = = 17,153 (giờ).
k 0,0404
2. a. Ở nhiệt độ T: hằng số tốc độ phản ứng là k’
Sau 1 giờ : Pchung = 1,1Po 0,75
Ta có:
1 P0
k ' = ln
t 3P0 - 2Pchung
0,25
1 P0
k’ = ln = 0,2231 giờ-1
1 3P0 -2.1,1P0 0,25
k' E  1 1
b. ln  a   
k R  583 T 

 ln 0, 2231  249720  1  1 
0, 0404 8,314  583 T 
 T = 603K
603583
k'
Có:   10
k
603583
 0, 2231   10  γ = 2,35
0, 0404

Câu 5 (2,5 điểm) Cân bằng acid – base và cân bằng ít tan.
1. Tính thể tích dung dịch H2S 0,1M cần thêm vào 100 ml dung dịch chứa đồng thời CdCl2 0,01M và HCl
0,01M để nồng độ Cd2+ giảm còn 10-6M. (Khi tính bỏ qua sự tạo phức cloro của Cd2+)
Cho: Cho pKa1,2 (H2S) = 10-7,02 ; 10-12,9 ; log*βCdOH+ = -10,2; pKs (CdS) = 24
2. Dung dịch A gồm axit H2C2O4 0,1M và axit HA. Biết độ điện li của HA trong dung dịch A là 3,34.10-2 %.
a. Tính pKa của HA. Biết để trung hòa hoàn toàn 10 ml dung dịch A cần 25 ml dung dịch NaOH 0,12M.
b. Thêm 90 ml dung dịch NH3 0,04M vào 10 ml dung dịch A. Tính pH của dung dịch thu được.
Cho H2C2O4 có: pKa1 = 1,25 và pKa2 = 4,27; NH4+ có pKa = 9,24
HƯỚNG DẪN CHẤM ĐIỂM
1
Gọi thể tích dung dịch H2S là V ml.
Phản ứng tạo kết tủa:
Cd2+ + H2S    CdS↓ + 2H+
 K = Ka1.Ka2.Ks-1 = 106,08
1 0,1V 1
C
100 + V 100 + V 100 + V
0,1V - 1 3 0,5
C’ -
100 + V 100 + V
0,1V - 1 3
TPGH: H2S ( M); H+ ( M); CdS↓
100 + V 100 + V
Vì môi trường axit nên quá trình tạo phức hiđroxo của Cd2+ và sự phân li của H2S là không đáng
kể.
Xét cân bằng:
CdS↓ + 2H+ 
 Cd2+ + H2S K = 10-6,08
3 0,1V - 1
C
100 + V 100 + V
3 0,1V - 1
[] - 2.10-6 10-6 + 10-6
100 + V 100 + V 0,5

 0,1V - 1 
  106  .106
 100 + V  = 10-6,08  V = 10,68 ml
2
 3 
  2.106 
 100 + V 
2
a. H2C2O4 + 2NaOH → Na2C2O4 + 2H2O
HA + NaOH → NaA + H2O
Ta có: 10.0,1.2 + 10.CHA = 25.0,12
CHA = 0,1M.
[A-] =  CHA.α HA = 0,1.3,34.10-4 = 3,34.10-5M
[HA] = CHA - [A-] ≈ 0,1M nên HA phân li không đáng kể
Trong dung dịch có các cân bằng:
H2C2O4    H+ + HC2O4-
 Ka1 = 10-1,25 (1)
HC2O4-    H+ + C2O42-
 Ka2 = 10-4,27 (2)
HA 
  H+ + A-
 KHA (3)
H2O 
  H+ + OH-
 KW
Giả sử KHA << Ka1
Ka1 >> Ka2 >> KW nên tính theo cân bằng (1), ta có:
H2C2O4   H+ + HC2O4- Ka1 = 10-1,25 (1)
Ban đầu 0,1M
[] 0,1 – x x x 0,5
2
 x = 10-1,25
0,1- x
0,25
 x = 0,052M = [H+]
[H + ].[A - ] 0, 052.3,34.105
KHA = = = 10-4,76
[HA] 0,1
b. Thêm NH3 dung dịch A:
CNH3 = 0,04.90/100 = 0,036M; CHA = CH2C2O4 = 0,1.10/100 = 0,01M.
Các phản ứng xảy ra:
NH3 + H2C2O4 → NH4+ + HC2O4- K1 = 107,99 >> 102 : phản ứng hoàn toàn.
0,5
NH3 + HC2O4- → NH4+ + C2O42- K2 = 104,97 >> 102 : phản ứng hoàn toàn.
NH3 + → NH4 + A + -
K3 = 104,48
>> 102 : phản ứng hoàn toàn.
Dung dịch sau phản ứng:
NH4+ 0,03M; NH3 0,006M; A- 0,01M; C2O42- 0,01M
Có các cân bằng sau: HA
NH4+ + H2O    NH3 + H3O+
 Ka (NH4+) = 10-9,24 (1)
NH3 + + H2O    NH4+ + OH-
 Kb (NH3) = 10-4,76 (2)
A- + H2O 
  HA + OH-
 Kb (A-) = 10-9,24 (3) 0,25
2-
C2O4 + H2O   
 HC2O4 + OH
- -
Kb1 = 10 -9,73
(4)
So sánh các hằng số K, ta thấy cân bằng chiếm ưu thế trong dung dịch là cân bằng (2). Do đó có
thể xem dung dịch thu được như một dung dịch đệm gồm NH3 0,006M và NH4+ 0,03M.
Gần đúng: pH = 9,24 + lg(0,006/0,03) = 8,54.
Câu 6 (2,5 điểm) Phản ứng oxi hóa – khử. Pin điện (không liên quan phức chất).
Cho giản đồ quá trình khử - thế khử: quá trình khử diễn ra theo chiều mũi tên, thế khử chuẩn được ghi trên
các mũi tên và đo ở pH = 0.

+0,293

0 0
+0,55 +1,34 Ex -0,408 Ey
Cr(VI) (Cr2O2-
7) Cr(V) Cr(IV) Cr3+ Cr2+ Cr
-0,744

1. Tính E 0x và E 0y .
2. Dựa vào tính toán, cho biết Cr(IV) có thể dị phân thành Cr3+ và Cr(VI) được không?
3. Viết quá trình xảy ra với hệ oxi hóa – khử Cr2 O 72- /Cr3+ và tính độ biến thiên thế của hệ ở nhiệt độ 298
K, khi pH tăng 1 đơn vị pH.
4. Phản ứng giữa K2Cr2O7 với H2O2 trong môi trường axit (loãng) được dùng để nhận biết crom vì sản
phẩm tạo thành có màu xanh. Viết phương trình ion của phản ứng xảy ra và cho biết phản ứng này có thuộc
loại phản ứng oxi hóa – khử hay không? Vì sao? Ghi số oxi hóa tương ứng trên mỗi nguyên tố.
Cho: E Cr O2-/Cr3+ = 1,33 V; Hằng số khí R = 8,3145 J.K–1.mol–1
0
2 7

Hằng số Farađay F = 96485 C.mol–1.


Câu 6 Hướng dẫn chấm Điểm
1 Từ giản đồ ta có: 3.(-0,744) = -0,408 + 2 E 0y  E 0y = -0,912 (V) 0.5
0,55 + 1,34 + E 0x – 3.0,744 = 6.0,293  E 0x = +2,1 (V)
Cr(IV) có thể dị phân thành Cr3+ và Cr(VI) khi ΔG0 của quá trình < 0.
2 2Cr(IV) + 2 e  2Cr3+ (1) E10 = E 0x = 2,1 V
 G10 = -n E10 F = - 2.2,1.F
0,5
0,55  1,34
Cr(VI) + 2 e  Cr(IV) (2) E = 0
2 = 0,945 (V)
2
 G 02 = -n E 02 F = - 2.0,945.F
Từ (1) và (2) ta có: 3Cr(IV)  2Cr3+ + Cr(VI) G 30 0,5
G = G - G = - 2.(2,1 - 0,945).F < 0  Vậy Cr(IV) có dị phân.
0
3
0
1
0
2

3 Cr2 O 72- + 14H+ + 6e 2Cr3+ + 7H2O 0,5


RT [Cr2 O 72- ].(10-pH )14
E1 = 1,33 + ln
6.F [Cr 3+ ]2
RT [Cr2 O 72- ].(10-(pH + 1) )14
E 2 = 1,33 + ln
6.F [Cr 3+ ]2
8,3145 . 298
Độ biến thiên của thế: E 2 - E1 = .14ln10-1 = -0,138 (V).
6 . 96485
4 +6 -2 +1 -1 +1 +6,-2/-1 +1 -2 0,5
Cr2 O 7 + 4H2O2 + 2H  2CrO5 + 5H2O
2- +

Phản ứng trên không phải là phản ứng oxi hóa-khử vì


số oxi hóa của các nguyên tố không thay đổi trong quá trình
.
phản ứng. Trong CrO5, số oxi hóa của crom là +6 và của oxi
là -2,
-2 -1 do peoxit CrO5 có cấu trúc:
O
-1
O O
Cr
+6
O O
Câu 7 (2,5 điểm) Halogen. Oxygen – Sulfur.
1. Chất lỏng A trong suốt, không màu; về thành phần khối lượng, A có chứa 8,3% hiđro; 59,0% oxi;
còn lại là clo. Khi đun nóng A đến 1100C, thấy tách ra khí X, đồng thời khối lượng giảm đi 16,8%,
khi đó chất lỏng A trở thành chất lỏng B. Khi làm lạnh A ở dưới 00C, thoạt đầu tách ra tinh thể Y
không chứa clo; còn khi làm lạnh chậm ở nhiệt độ thấp hơn nữa sẽ tách ra tinh thể Z chứa 65% clo
về khối lượng. Khi làm nóng chảy tinh thể Z có thoát ra khí X. Cho biết công thức và thành phần
khối lượng của A, B, X, Y, Z?
2. Cho 6,00 gam mẫu chất chứa Fe3O4, Fe2O3 và các tạp chất trơ. Hòa tan mẫu vào lượng dư dung
dịch KI trong môi trường axit (khử tất cả sắt thành Fe2+) tạo ra dung dịch A. Pha loãng dung dịch A
đến thể tích 50ml. Lượng I2 có trong 10ml dung dịch A phản ứng vừa đủ với 5,50 ml dung dịch
Na2S2O3 1,00M (sinh ra S4 O 62 ). Lấy 25 ml mẫu dung dịch A khác, chiết tách I2, lượng Fe2+ trong
dung dịch còn lại phản ứng vừa đủ với 3,20 ml dung dịch KMnO4 1,00M trong dung dịch H2SO4.
a. Viết phương trình hóa học của các phản ứng xảy ra (dạng phương trình ion thu gọn).
b. Tính phần trăm khối lượng Fe3O4 và Fe2O3 trong mẫu ban đầu.

Hướng dẫn chấm Điểm


1. Đặt tỉ lệ số nguyên tử H:O:Cl trong A là a:b:c.
8,3 59 32, 7
Ta có: : : = 8,3 : 3,69 : 0,92 = 9 : 4 : 1
1 16 35,5
Ta thấy, không tồn tại chất ứng với công thức H9O4Cl. Tuy nhiên, tỉ lệ H:O là 9:4 gần với tỉ lệ
của các nguyên tố trong phân tử H2O.
- Có thể suy ra chất lỏng A là dung dịch của HCl trong H2O với tỉ lệ mol là 1 : 4 với C% HCl =
36,5
.100% = 33,6%
36,5  18.4
- Khi tăng nhiệt độ sẽ làm giảm độ tan của khí, hợp chất X thoát ra từ A là khí hiđro clorua 0,5
(HCl).
33, 6  16,8
- Do giảm nồng độ HCl  C% HCl còn lại = .100% = 20,2%
100  16,8
 Chất lỏng B là dung dịch HCl nồng độ 20,2%.
(Dung dịch HCl ở nồng độ 20,2% là hỗn hợp đồng sôi, tức là hỗn hợp có thành phần và nhiệt độ 0,5
sôi xác định).
- Khi làm lạnh dung dịch HCl ở dưới 0oC, có thể tách ra tinh thể nước đá Y.
- Khi làm lạnh ở nhiệt độ thấp hơn, thì tách ra tinh thể Z là HCl.nH2O.
35,5
- Tinh thể Z có khối lượng mol phân tử là = 54,5 g/mol
0, 65
 thành phần tinh thể Z là: HCl.H2O.
2. Fe O  8H  2Fe3  Fe2  4H O (1)
3 4 2
Fe O  6H  2Fe3  3H O (2)
2 3 2
2Fe3  3I  2Fe2  I (3) 0,5
3
2S O 2 
I S O 2   3I (4)
2 3 3 4 6
5Fe2  MnO   8H  5Fe3  Mn 2  4H O (5)
4 2
Trong 25 ml: n Fe2  5n MnO  5x3, 2x1x103 =0,016 (mol)
4

→ trong 10ml n Fe2 = 6,4x10-3(mol)


Từ (3) và (4): n Fe2 = n S O 2 = 5,5x1x10-3 = 5,5x10-3(mol)
2 3

Từ (3): n Fe3 = n Fe2 =5,5x10-3(mol) =2( n Fe3O4 + n Fe2O3 )


Có thể xem Fe3O4 như hỗn hợp Fe2O3.FeO
n FeO = n Fe3O4 = 6,4x10-3 – 5,5x10-3 = 9x10-4(mol) 0,5
1
n Fe2O3 = n 3  n Fe3O4 =1,85x10-3(mol).
2 Fe
Trong 50 ml : n Fe3O4 =4,5x10-3(mol) → m Fe3O4 =1,044 gam
→ % khối lượng Fe3O4 = 1,044/6 x 100% = 17,4%
n Fe2O3 = 9,25x10-3(mol) → m Fe2O3 =1,48 gam
→ % khối lượng Fe2O3 = 1,48/6 x 100% = 24,67%

Câu 8. (2,0 điểm) Đại cương hữu cơ (quan hệ giữa cấu trúc và tính chất)
1. Gọi tên theo danh pháp IUPAC các chất hữu cơ sau:

2.
a) Quy kết các giá trị nhiệt độ sôi (oC) sau: 116, 194, 201 phù hợp cho 3 đồng phân vị trí của nitrophenol.
Giải thích ngắn gọn.
b) So sánh và giải thích ngắn gọn tính axit của H trong các phân tử sau:
H

H
A B C
3. Xác định cấu dạng bền của các hợp chất A, B trong các môi trường: a) metanol; b) octan.
Cl

Me HO OH
O
A B
Câu Đáp án Điểm
10 0,5

0,5
a) + Quy kết nhiệt độ sôi 0,25
OH OH OH

NO2

NO2
NO2
o
t C: 201 194 116
+ Giải thích:
- o-nitrophenol có liên kết H nội phân tử=> nhiệt độ sôi thấp nhất
- p-nitrophenol có liên kết H liên phân tử bền nhất và H/OH linh động nhất vì nhóm 0,25
NO2 gây hiệu ứng –I, -C mạnh.
- m-nitrophenol có liên kết H liên phân tử , H/OH không linh động bằng H/OH của
p-nitrophenol vì nhóm NO2 gây hiệu ứng –I.
b)
- So sánh: Tính axit của: A > C > B. 0,25
- Giải thích: A tạo ra anion là hệ thơm nên bền hơn cả, B tạo ra anion là hệ phản thơm
nên anion kém bền nhất, C tạo ra anion hệ không thơm. 0,25
Các cấu dạng có thể có của A, B:
Cl
Cl
Cl 0,25
Me
Me
O O
A O
A1 Me
A2

OH
HO OH
HO
O . .. O
H
B H
B1 B2
Giải thích:
+ Metanol là dung môi phân cực, các chất tồn tại ở các dạng momen lưỡng cực lớn nhất 0,25
hay liên kết hidro liên phân tử với dung môi: nên chất A tồn tại ở dạng A1; chất B tồn tại
ở dạng B1.
+ Octan là dung môi không phân cực, các chất tồn tại ở các cấu dạng sao cho momen
lưỡng cực nhỏ, phân tử ít phân cưc nhất: nên chất A tồn tại ở dạng A2; chất B ở dạng B2
ĐỀ SỐ 3
Bài 1 (2,5 điểm): Cấu tạo nguyên tử. Phản ứng hạt nhân. Định luật tuần hoàn
1.1. Một nguyên tử hiđro khi chuyển từ trạng thái kích thích ứng với n =5 về trạng thái ứng với n=2 phát ra
ánh sáng màu xanh. Một ion He+ (ZHe=2) khi chuyển từ trạng thái kích thích ứng với n=c về trạng thái ứng
với n=t sẽ phát ra ánh sáng màu xanh giống như vậy. Tìm giá trị của c, t.

HƯỚNG DẪN CHẤM Điểm


Nguyên tử hiđro khi chuyển từ trạng thái kích thích ứng với n =5 về trạng thái ứng với n=2 0,25
thì phát ra bức xạ có năng lượng :
1 1 
n = 5 → n = 2: E5 – E2  2,18 1018   2  2 
5 2 
Một ion He (ZHe=2) có cấu tạo 1 hạt nhân, 1 electron giống nguyên tử Hidro  Năng 0,5
+

lượng của electron có dạng :


Z2 Z 2 1, 602 1019
E  13, 6  2 (eV)  13, 6  (J)
n n2
Khi chuyển từ trạng thái kích thích ứng với nc về trạng thái ứng với nt sẽ phát ra ánh sáng
có năng lượng giống như vậy tức là :
Z2 Z 2 1, 602 1019
E  13, 6  2 (eV)  13, 6  (J)
n n2
1 1 1 1
E  13, 6  Z 2 1, 602 1019  ( 2  2 )  2,18  2 2 10 18  ( 2  2 )
nc nt nc nt
1 1 1 1 1 1 1 1 1
 2,18 1018  ( 2
 2 )  ( 2  2 )  2 ( 2  2 )  2  2
5 2 nc nt 2 5 2 10 4
+
Vậy He chuyển từ n = 10 về n = 4. 0,25
1.2. Hãy sắp xếp (có giải thích) các hạt vi mô cho dưới đây theo chiều giảm dần bán kính hạt:
Rb+ (Z = 37); Y3+ (Z = 39); Kr (Z = 36), Br- (Z = 35), Se2- (Z = 34), Sr2+ (Z = 38)
HƯỚNG DẪN CHẤM Điểm
Chiều giảm dần bán kính hạt: 0,25
2- - + 2+ 3+
Se > Br > Kr > Rb > Sr > Y
Giải thích : các hạt có cùng cấu hình e ; hạt nào có Z càng lớn  lực hút giữa hạt nhân và 0,25
e càng lớn làm bán kính giảm.
1.3. Urani tự nhiên chứa khoảng 99,3% 238 235
92U; 0,7% 92U (về khối lượng) cùng với lượng nhỏ các đồng vị
phóng xạ là sản phẩm phân rã của các đồng vị trên, như 226 210
88Ra, 84Po … Một mẫu quạng urani có khối
lượng 10 kg lấy từ mỏ Nông Sơn (Quảng Nam) có hoạt độ phóng xạ của 226 4
88Ra bằng 7,51.10 Bq.
238 210 235
a) Hoạt độ phóng xạ của 92U, 84Po và 92U trong mẫu quạng nói trên bằng bao nhiêu? Cho rằng có cân
bằng thế kỉ giữa các đồng vị phóng xạ khởi đầu các họ phóng xạ tự nhiên và các con cháu của chúng. Cho
chu kì bán rã của 238 226 235
92U bằng 4,47.10 năm, của 88Ra bằng 1620 năm, của 92U bằng 7,038.10 năm (1 năm
9 8

có 365 ngày).
b) Những ước tính trung bình cho rằng sự phân hạch 1 kg 235U sinh ra 6,55.1010 kJ. Tính xem trong bao
nhiêu kg quạng urani nói trên có chứa một lượng 235U mà sự phân hạch tỏa ra năng lượng bằng 1,82.108
kWh.
HƯỚNG DẪN CHẤM Điểm
238 226 210
a) 92U, 88Ra và 84Po có số khối khác nhau một số nguyên lần của 4, cúng cùng họ 0,25
phóng xạ (họ Urani-radi) và nằm trong trạng thái cân bằng thế kỉ. Hoạt độ phóng xạ (A)
của 3 đồng vị nằng bằng nhau:
A( 238 226 210
92U) = A( 88Ra) = A( 84Po) = 7,51.10 Bq
4

Khối lượng 23892U trong mẫu là: 0,25


7,51.104 .4,47.109 .238.365.24.60.60
m( 238 238 238 238 23
92U) = [A( 92U)/k( 92U)][M( 92U)/6,022.10 ] = = 6,04 g
0,693.6,022.1023
235
Khối lượng 92Utrong mẫu là: 6,04.(0,7/99,3) = 4,26.10 g -2

Hoạt độ phóng xạ của 235


92U là: 0,25
0,693.4,26.10−2 .6,022.1023
A( 235 235 235 3
92U) = k(( 92U). N( 92U) = 7,038.108 .365.24.60.60.235 = 3,4.10 Bq

b) Khối lượng 235U cần dùng là: (1,82.108.3600)/6,55.1010 = 10 kg 0,25


Khối lượng quặng tương ứng là: 10.10/4,26.10-5 = 2,35.106 kg
Bài 2 (2,5 điểm): Cấu tạo phân tử. Tinh thể
2.1.
a) Viết công thức Lewis cho PF5. Sử dụng mô hình VSEPR để biểu diễn cấu trúc cho biết dạng hình học của phân tử
này.
b) Ở trạng thái rắn Photphopentaclorua có cấu trúc ion với sự có mặt hai ion PCl 4 và PCl 6 . Sử dụng mô hình
VSEPR hãy biểu diễn và cho biết dạng hình học của các ion này.
HƯỚNG DẪN CHẤM Điểm
2 2 5 2 2 5
a) 9F: 1s 2s 2p ; 15P: 1s 2s 2p 0,25
Công thức Lewis: PF5 có dạng AX5E0. Theo VSEPR  có cấu trúc
lưỡng tháp tam giác

b) PCl 4 có dạng AX4E0  cấu trúc tứ diện: 0,25

PCl 6 có dạng AX6E0  cấu trúc bát diện:

2.2.
a) Vẽ giản đồ năng lượng obitan phân tử (MO) cho phân tử O2. Từ đó cho biết cấu hình electron của các tiểu
phân: O2; O 2 ; O2 và O 22
b) Sắp xếp năng lượng ion hóa của các tiểu phân sau đây theo thứ tự từ thấp đến cao: O, O2, O 2 ; O2 . Giải
thích.
HƯỚNG DẪN CHẤM Điểm
a) Giản đồ MO của O2: 0,5
Cấu hình electron của các phân tử và ion: 0,5
( ) 2 (*2s ) 2 (2p ) 2 (2p ) 4 (*2p ) 2
O2: 2s
O 2 (2s ) (2s ) (2p ) ( 2p ) ( 2p )
2 * 2 2 4 * 3
:
O 2 (2s ) (2s ) (2p ) (2p ) (2p )
2 * 2 2 4 * 1
:
O 22 (2s ) (2s ) (2p ) (2p )
2 * 2 2 4
:
b) Thứ tự năng lượng ion hóa: O2-< O2 < O2+ < O 0,5
Giải thích: Electron có năng lượng cao nhất của O2 , O2, O2 đều nằm ở trên mức năng
- +

lượng cao nhất 2p*. Nhưng đối với O2- electron tách ra từ cặp e ghép đôi và từ ion âm 
O2- có năng lượng ion hóa thấp nhất. Ion O2+ có điện tích dương, làm cho việc tách e khó
hơn so với phân tử O2 (cả hai cùng tách e độc thân từ 2p*). Electron có năng lượng cao nhất
của O nằm ở AO 2p, thấp hơn so với năng lượng của electron trên MO 2p* của những tiểu
phân còn lại  O có năng lượng ion hóa cao nhất.
2.3. Thực nghiệm cho biết ở pha rắn, vàng (Au) có khối lượng riêng là 19,4 g/cm3 và có mạng lập phương
tâm diện. Độ dài cạnh của ô mạng đơn vị là 4,070.10-10 m. Khối lượng mol nguyên tử của Au là 196,97
g/mol.
a) Tính phần trăm thể tích không gian trống trong mạng lưới tinh thể của Au.
b) Xác định trị số của số Avogadro.
HƯỚNG DẪN CHẤM Điểm
a) Cạnh hình lập phương = a, khoảng cách 2 đỉnh kề nhau: a = 4,070.10-10 m 0,25
Khoảng cách từ đỉnh đến tâm mặt lập phương là nửa đường chéo của mỗi mặt vuông:
½(a√2) = a/√2 < a
Đó là khoảng cách gần nhất giữa 2 nguyên tử bằng 2 lần bán kính nguyên tử Au.
4,070.10-10m : √2 = 2,878.10-10m = 2r
R: bán kính nguyên tử Au = 1,439.10-10 m
Mỗi ô mạng đơn vị có thể tích = a3 = (4,070.10-10m)3 = 67,419143.10-30 m3 và có chứa 4
nguyên tử Au
4
Thể tích 4 nguyên tử Au là 4 nguyên tử x (4/3)ᴨr3 = 4.3(3,1416).( 1,439.10-10)3 = 49,972.10-
30
m3
Độ đặc khít = 49,972.10-30/67,419143.10-30 = 0,74054 = 74,054%
Độ rỗng = 100%-74,054% = 25,946%

b) 1 mol Au = NA nguyên tử Au có khối lượng 196,97 gam 0,25


196,97 𝑔
1 Nguyên tử Au có khối lượng = 𝑁 𝑛𝑔.𝑡𝑢
𝐴
3 𝐾𝑙 4𝑛𝑔𝑡𝑢 𝐴𝑢 4.196,97
Tỉ khối của Au rắn: d = 19,4 g/cm = =
𝑉 𝑜 𝑚𝑎𝑛𝑔 𝑁𝐴 .𝑎3
4.196,97
19,4 = 𝑁 −30 .106
→ NA = 6,02386.1023
𝐴 .67,419143.10

Bài 3 (2,5 điểm) Nhiệt hóa học. Cân bằng hóa học trong pha khí.
3.1. Khi đốt cháy 3,9 g hơi benzen ở 250C, 1atm với một lượng oxi dư toả ra 163400 J sản phẩm là CO2(k)
và H2O(l).
a. Hãy tính nhiệt toả ra khi đốt cháy 7,8g hơi benzen và oxi dư trong bom nhiệt lượng kế ở 250C sản phẩm là
CO2(k) và H2O(l).
b. Tính nhiệt độ của ngọn lửa benzen cháy ở áp suất 1,0 atm, 250C trong 2 trường hợp sau đây:
i. Trong không khí (20% oxi và 80% nito về thể tích).
ii. Trong oxi nguyên chất .
C0p (CO2 khí) = 26,80 +42,3.10-3T (J/mol.K) C0p (N2 khí) = 27,10 +6,00.10-3T (J/mol.K)
∆H bay hơi của nước lỏng ở 373K là 40,66kJ/mol;
0

C p của nước lỏng là 75,3 J/mol.K; C0p của nước khí là 30,2+1,00.10-2T (J/mol.K);
0

HƯỚNG DẪN CHẤM Điểm


a. Tính nhiệt lượng toả ra khi đốt cháy Benzen trong bom nhiệt lượng kế 0,25
Ở 298K: C6H6 (h) + 15/2 O2 6CO2 (k) + 3H2O(l)
- Nếu phản ứng này xảy ra ngoài không khí thì nhiệt toả ra là QP = H = 163400 J.
78/3,9= 3268000J /mol = 3268 kJ/mol.
- Nếu phản ứng xảy ra trong bom nhiệt lượng kế:
Nhiệt toả ra khi đốt cháy 1 mol hơi benzen trong bom nhiệt lượng kế là nhiệt cháy đẳng tích:
U = H -nRT = -3268.103 – (-2,5).8,314.298
= -3261,806.103 J/mol = -3261,806 kJ/mol.
Vậy khi đốt cháy hơi benzen trong bom nhiệt lượng kế ở 250C bởi Oxi dư sẽ toả ra một
lượng nhiệt là 3261,806kJ/mol.
b. Tính nhiệt độ của ngọn lửa đốt cháy Benzen 0,25
Giả sử nhiệt độ ngọn lửa < 373K => Nước tạo ra ở thể lỏng, nên khí trong ngọn lửa chỉ có
CO2 (và có N2 nếu đốt cháy trong không khí)
Xét đốt cháy Benzen trong không khí:
CP của các khí trong ngọn lửa = 6.CP (CO2) + 4.(15/2)CP (N2)=973,8 + 0,4338.T
Nhiệt toả ra khi đốt cháy 1 mol benzen hơi dùng để tăng nhiệt độ của hỗn hợp khí sản phẩm
và làm tăng nhiệt độ của 3 mol H2O lỏng từ nhiệt độ đầu là 298K lên nhiệt độ của ngọn lửa
T T
 -H = ∫298(973,8 + 0,4338. T)dT + ∫298 Cp nước lỏng dT
T T
= ∫298(973,8 + 0,4338. T)dT + ∫298 3.75,3. dT
0,4338
 3268.1000 = 973,8(T- 298) + 2 ( T2 – 2982) +225,9. (T-298)
 0,2169.T2 + 1199,7.T – 3644772,188= 0
T = 2179,361 => nước tạo ra ở thể khí.
Khi nước tạo ra ở thể khí thì, nhiệt toả ra do phản ứng đốt cháy 1 mol hơi benzen dùng để: 0,25
1. tăng nhiệt độ của 7,5 mol N2 (nếu có) và 6 mol CO2 từ nhiệt độ đầu đến nhiệt độ ngọn lửa
2. tăng nhiệt độ của 3 mol nước lỏng từ 298K đến nhiệt độ sôi.
3. dùng để chuyển pha 3 mol nước lỏng tại 373K
4. dùng để tăng nhiệt độ của 3 mol nước hơi từ 373K lên nhiệt độ ngọn lửa
a. Nếu đốt cháy benzen ngoài không khí thì: 0,25
CP của N2 và CO2 = 6.CP (CO2) + 4.(15/2)CP (N2) = 973,8 + 0,4338.T
T 373
 -H = ∫298(973,8 + 0,4338. T)dT + ∫298 3. Cp nước lỏng dT + 3. Hhoá hơi nước +
T
∫373 3. Cp nước hơi dT
T 373 T
= ∫298(973,8 + 0,4338. T)dT + ∫298 3.75,3. dT + 3. 40,66.1000 + ∫373 3. (30,2 +
10−2 T) dT
 3268.1000= 1199,7 (T-298) +0,2169.(T2- 2982) + 121980 + 90,6.(T-373)+ 0,015.(
T2- 3732)
 0,2319T2 + 1290,3.T- 3558672,923 = 0
T = 2204,96K
b. Nếu đốt trong oxi tinh khiết 0,25
CP của khí CO2= 6.CP (CO2) = 160,8 +0,2538.T
T 373
-H = ∫298(160,8 + 0,2538. T)dT + ∫298 3. Cp nước lỏng dT + 3. Hhoá hơi nước +
T
∫373 3. Cp nước hơi dT
T 373 T
= ∫298(160,8 + 0,2538. T)dT + ∫298 3.75,3. dT + 3. 40,66.1000 + ∫373 3. (30,2 +
10−2 T) dT
 3268.1000= 386,7 (T-298) +0,2169.(T2- 2982) + 121980 + 90,6.(T-373)+ 0,015.( T2-
3732)
 0,2319T2 + 477,3.T- 3316398,923 = 0T = 2890,082K
Kết luận: Phản ứng đốt cháy benzen trong oxi tinh khiết thì nhiệt độ ngọn lửa cao hơn đốt
cháy trong không khí.
𝟏
3.2. Cho bảng dữ kiện nhiệt động của phản ứng: SO2 (k) + 𝟐O2 (k)  SO3 (k) (1)
Nhiệt độ ( C)
o
530 550
ΔrG (kJ/mol)
o
–16,03 –15,31
a. Ước lượng hằng số cân bằng Kp1 của phản ứng (1) ở 650oC.
Cho 15,19 g sắt (II) sunfat được đun nóng trong bình chân không 1,00 L tới 650oC thì xảy ra các phản ứng
sau:
2FeSO4 (r)  Fe2O3 (r) + SO3 (k) + SO2 (k) (2)
2SO3(k)  2SO2(k) + O2(k) (3)
Khi hệ đạt đến trạng thái cân bằng, áp suất riêng phần của oxy là 21,28 mmHg.
b. Tính áp suất tổng ở trạng thái cân bằng và giá trị Kp2 của phản ứng (2).
c. Tính phần trăm FeSO4 bị phân hủy?
HƯỚNG DẪN CHẤM Điểm
1
a. SO2 (k) + 2O2 (k)  SO3 (k) (1) 0,25
ΔH = -44938 J/mol; ΔS = -36 J/molK
ΔG ở 650oC = -11710 J/mol → lnKp1 = 1,526 → Kp1 = 4,6
b. 2SO3(k)  2SO2 (k) + O2 (k) (3) có Kp3 = (Kp1)-2 = 0,047 0,25
Phản ứng (2): 2 FeSO4 (s)  Fe2O3 (s) + SO3 (g) + SO2 (g)
Phân hủy: - - -
Cân bằng: P-a P+a => Ptổng = 2P
Phản ứng (3): 2 SO3(g)  2SO2 (g) + O2 (g)
Áp suất ban đầu: P P 0
Phản ứng: -a +a +a/2
Cân bằng: P-a P+a a/2
Ở cân bằng thì áp suất của O2 là 21,28/760 = 0,028 atm 0,25
a/2 = 0,028 atm → a = 0,056 atm
( P  0, 056) 2
Kp3 = ( P  0,056) 0,2028  0,047 →  1, 6786 ( P  0, 056)  1, 296
2

( P  0,056) ( P  0, 056) 2
( P  0, 056)
→ P + 0,056 = 1,296P – 0,073
→ 0,296P = 0,129 → P = 0,436 atm → P tổng = 2P = 0,872 atm
2 FeSO4 (s)  Fe2O3 (s) + SO3 (g) + SO2 (g) 0,25
Kp2 = PSO3PSO2 = (P-a)(P+a) = (0,436 – 0,056)(0,436 + 0,056) = 0,187
c. Số mol SO3 = SO2 do phân hủy FeSO4: 0,5
PV = nRT → n = PV/PT = (0,436)1 /(0,082×923) = 5,76×10-3 mol
Số mol FeSO4 = 2nSO3 = 0,01152 mol
Khối lượng FeSO4 phân hủy = 0,01152 ×152 = 1,751 g
Phần trăm khối lượng FeSO4 = 1,751/15,19 = 11,53 %.

Bài 4 (2,5 điểm) Động hóa học (không có cơ chế).


4.1. Sunfuryl clorua (SO2Cl2) được sử dụng rộng rãi trong công nghiệp. Sunfuryl clorua là một chất lỏng không
màu, có mùi cay, sôi ở 70oC. Khi nhiệt độ trên 70oC nó sẽ phân hủy tạo thành SO2 và Cl2 theo phản ứng:
SO2Cl2(k)  SO2(k) + Cl2(k)
Một bình kín thể tích không đổi chứa SO2Cl2(k) được giữ ở nhiệt độ 375K. Quá trình phân hủy
SO2Cl2(k) được theo dõi bằng sự thay đổi áp suất trong bình. Kết quả thu được như sau:
Thời gian, t(s) 0 2500 5000 7500 10000
Áp suất, P(atm) 1,000 1,053 1,105 1,152 1,197
a) Chứng tỏ rằng phản ứng phân hủy SO2Cl2 là phản ứng bậc 1. Tính hằng số tốc độ của phản ứng ở 375K.
b) Nếu phản ứng trên được tiến hành ở 385K, áp suất của bình sau 1 giờ là 1,55 atm. Tính năng lượng hoạt
hóa của phản ứng phân hủy trên.
HƯỚNG DẪN CHẤM Điểm
a) Nếu phản ứng SO2Cl2(k)  SO2(k) + Cl2(k) là phản ứng bậc 1 thì ta có: 0,25
1 C 1 P
k  ln o  ln o
t Ci t Pi
Ở đây, Po là áp suất đầu (Po = 1 atm), Pt là áp suất SO2Cl2 sau thời gian t. Gọi x (atm) là áp 0,5
suất SO2Cl2 đã phản ứng: x = Ptổng – 1 và Pt = Po – x = 1 – x.
Ta có:
Thời gian, t(s) 2500 5000 7500 10000
Áp suất tổng P(atm) 1,053 1,105 1,152 1,197
x = (Ptổng – 1) 0,053 0,105 0,152 0,197
PSO Cl = 1 – x
2 2
0,947 0,895 0,848 0,803
Thay vào biểu thức tính k ta được:
1 1 1 1
k1  ln  ln  2,178.10 5 ( s 1 )
t 1  x 2500 0,947
1 1
k2  ln  2, 219.105 ( s 1 )
5000 0,895
1 1
k3  ln  2,198.105 ( s 1 )
7500 0,848
1 1
k4  ln  2,194.10 5 ( s 1 )
10000 0,803
Nhận xét: Các giá trị hằng số tốc độ k1, k2, k3 và k4 khác nhau không đáng kể do sai số thực 0,25
nghiệm. Vậy giả thiết phản ứng bậc 1 là đúng:
k1  k2  k3  k4 (2,178  2, 219  2,198  2,194).105
k   2,197.105 ( s 1 )
4 4
b) Tính năng lượng hoạt hóa của phản ứng: 0,25
Ở 385K, Ptổng = 1,55 atm  1,55 = 1 + x và x = 0,55 atm.
Ta có: PSO2Cl2 = 1 – 0,55 = 0,45 atm.
k2 E 1 1
Sử dụng phương trình Arrhenius: ln   a    ta có:
k1 R  T2 T1 
2, 218.104 E  1 1 
ln 5
 a     Ea  277,529(kJ / mol )
2,197.10 8,314  385 375 

4.2. Tốc độ của phản ứng khử HCrO4 – bằng HSO3 – được biểu diễn bằng phương trình tốc độ:
V = k.[HCrO4 – ][HSO3 – ]2[H+]
Trong một thí nghiệm với các nồng độ ban đầu: HCrO4– = 10-4 mol/l; HSO3 – = 0,1 mol/l; H+ cố định bằng
10-5 mol/l thì sau 15 giây thấy nồng độ HCrO4- giảm xuống còn 5.10-5 mol/l.
a) Sau bao lâu nồng độ HCrO4– sẽ bằng 1,25.10–5 M? Tính hằng số tốc độ của phản ứng.
b) Nếu nồng độ đầu của HSO3– là 0,01M thì sau bao lâu nồng độ của HCrO4– sẽ bằng 5.10–5 M.
c) Nếu nồng độ ban đầu của HSO3– và H+ đều bằng 10–3 M và được giữ cố định thì cần thời gian bao lâu để
một nửa lượng HCrO4 – bị khử?
HƯỚNG DẪN CHẤM Điểm
– – –4
a. Vì nồng độ HSO3 = 0,1M >> HCrO4 = 10 M và H = const +
0,25
=> phản ứng là giả bậc nhất với HCrO4 –
=> thời gian bán phản ứng là 15 giây
=> để HCrO4 – còn 1,25.10 – 5 M thì cần thời gian là :
1 10−4
t = 𝑘′ ln1,25.10−5= 45 (giây)
Khi phản ứng là giả bậc 1 với HCrO4 – ta có 0,25
2 ln 2 0, 693
k '  k .  HSO3-  .  H      0, 0462
t1 15
2

0, 0462 0,25
=> k  2 5
 4, 62.105 ( M 3 .s 1 )
0,1 .10
b. Phản ứng là bậc 2 với HSO3 – nên khi giảm nồng độ HSO3 – từ 0,1M xuống 0,01M thì tốc độ 0,25
phản ứng giảm 100 lần
=> để HCrO4 – giảm còn 5.10 – 5 M thì thời gian cần là: 15. 100 = 1500 giây.
c. Nếu nồng độ đầu của HSO3 – và H+ đều bằng 10 – 3 M và không đổi khi đó k” giảm 100 lần so 0,25
với k’ => tốc độ phản ứng giảm 100 lần
Hay thời gian để ½ lượng HCrO4 – bị khử là 1500 giây
Bài 5 (2,5 điểm) Cân bằng acid – base và cân bằng ít tan.
5.1. Trộn 10,00 mL dung dịch CH3COOH 0,20 M với 10,00 mL dung dịch H3PO4, thu được dung dịch A có
pH = 1,50.
a) Tính C H3PO4 trong dung dịch H3PO4 trước khi trộn.
b) Tính độ điện li của CH3COOH trong dung dịch A.
c) Thêm từ từ Na2CO3 rắn vào dung dịch A cho đến pH = 4.0, thu được dung dịch B. Tính số gam Na2CO3 đã
dùng.
Cho biết: H3PO4: pKa1 = 2.15; pKa2 = 7.21; pKa3 = 12.32;
CH3COOH: pKa = 4.76; CO2 + H2O có pKa1 = 6.35; pKa2 = 10.33;
HƯỚNG DẪN CHẤM Điểm
a) pHA = 1,50  Không cần tính đến sự phân li của nước. 0,5
Các quá trình xảy ra trong dung dịch A:

H3PO4   H+ + H2PO4-
 Ka1 = 10-2,15 (1)

 H+ + CH3COO-
CH3COOH 
 Ka = 10-4,76 (2)

 H+ + HPO42-
H2PO4- 
 Ka2 = 10-7,21 (3)

 H+ + PO43-
HPO42- 
 Ka3 = 10-12,32 (4)
Vì Ka1 >> Ka >> Ka2 >> Ka3 nên pHA được tính theo (1):
H3PO4    H+
 + H2PO4- Ka1 = 10-2,15 (1)
[] (0,5C – 10-1,5) 10-1,5 10-1,5
1,5 1,5
10 .10
 1,5
 102,15
(0,5C  10 )
 C H3PO4 = C = 0,346M
b) CH3COOH    H+ + CH3COO- Ka = 10-4,76 (2) 0,25

[] (0,1 – x) 10-1,5 x
1,5
10 .x
  104,76
(0,1  x)
5, 49.105
 x = 5,49.10-5 M   CH3COOH  .100  0,055%
0,1
c) Tại pH = 4,00 ta có: 0,25
[H 2 PO 4 ] K a1 102,15 [H 2 PO 4 ]
 +    0,986
[H 3 PO 4 ] [H ] 104 [H 2 PO 4 ]  [H 3PO 4 ]
[HPO24 ] K a2 107,21

 +  4
 103,21  [HPO24 ]  [H 2 PO 4 ]
[H 2 PO 4 ] [H ] 10
[CH 3COO  ] K 104,76 [CH 3COO  ]
 +a    0,148
[CH 3COOH] [H ] 10 4 [CH 3COO  ]  [CH 3COOH]
Tương tự:
[CO32  ] 1010,33

 4
 1  [CO32  ]  [HCO3 ]
[HCO3 ] 10
[HCO3 ] 106,35
 4
 1  [HCO3 ]  [CO 2 ]
[CO 2 ] 10
Như vậy. khi trung hòa đến pH = 4,00 thì chỉ có 14,8% CH3COOH và 98,6% nấc 1 của H3PO4 0,5
bị trung hòa, còn CO32- phản ứng với H+ của 2 axit tạo thành CO2.
2H3PO4 + CO32-   2H2PO4- + CO2 + H2O
CH3COOH + CO32-   2 CH3COO- + CO2 + H2O
Vậy:
nCO2-  0,5.(14,8%.nCH3COOH  98,6%.n H3PO4 )
3

 0,5.20.103 (14,8%.0,1  98,6%.0,173)


 n CO2-  1,85.103 (mol)  m Na2CO3  0,1961(gam)
3
5.2. Lấy 3 giọt dung dịch gồm Ba2+ 0,03 M và Sr2+ 0,03 M vào ống nghiệm rồi thêm tiếp 6 giọt dung dịch đệm
axetat gồm CH3COOH 0,1 M và CH3COONa 0,05 M. Sau đó thêm 1 giọt dung dịch K2Cr2O7 1M vào ống
nghiệm này. Hỏi có kết tủa xuất hiện hay không? Nếu có hãy cho biết màu sắc và thành phần của kết tủa.
(Giải thích bằng những tính toán cụ thể). Các giọt dung dịch đều có thể tích như nhau.
Cho NH4+ có pKa = 9,24 H2CO3 có pKa= 6,35; 10,33 CH3COOH có pKa= 4,76
Cr2O72– + H2O ⇌ 2CrO42– + 2H+ K a = 2,3.10-15
pKs(BaCrO4)= 9,93 pKs(SrCrO4)= 4,65 Độ tan của CO2: LCO2=0,03 M

HƯỚNG DẪN CHẤM Điểm


Nồng độ các chất trong ống nghiệm sau khi thêm hóa chất: 0,25
Ba2+ 0,009 M Sr2+ 0,009 M CH3COOH 0,06 M CH3COONa 0,03 M K2Cr2O7 0,1M
pH của hệ do dung dịch đệm CH3COOH 0,06 M /CH3COONa 0,03 M quyết định
pH= pKa + lg Cb/Ca = 4,76 +lg 0,03/0,06 = 4,46
kiểm tra điều kiện: pH= 4,76 nên [H+] và [OH-] khác xa 10-7
Ca, Cb >> [H+] và [OH-]  tính pH như trên là đúng.
Cr2O72- + H2O ⇌ 2CrO42- + 2H+ K a = 2,3.10-15 0,25
[] 0,1-x 2x 10-4,46
2 −4,46
(2𝑥) . (10 )2
= 2,3. 10−15
(0,1 − 𝑥)
x= 2,187.10-4  [CrO42-] = 4,374.10-4 M
Xét [Ba2+].[ CrO42-] = 3,94.10-6 > Ks(BaCrO4)= 10-9,93 nên có kết tủa BaCrO4 0,5
[Sr2+].[ CrO42-] = 3,94.10-6 <Ks(SrCrO4)= 10-4,65 nên không có SrCrO4 kết tủa.
Vậy thu được kết tủa màu vàng chỉ có BaCrO4.

Bài 6 (2,5 điểm) Phản ứng oxi hóa – khử. Pin điện (không liên quan phức chất).
6.1. Cho biết giản đồ Latimer của iot và mangan trong môi trường axit như sau:
+1,20V

1,70 V 1,14 V 1,45 V 0,54 V


H 4 IO6  IO3   HIO  I3  I

+1,51V
0,56 V 2,26 V 0,95 V 1,51 V 1,18 V
MnO 4  MnO 42  MnO 2  Mn 3   Mn 2   Mn
+1,70 V +1,23 V

Lập luận để viết phương trình hóa học (dạng ion thu gọn) của phản ứng xảy ra khi cho dung dịch KI tác dụng
với dung dịch KMnO4 (trong môi trường axit) trong trường hợp sau phản ứng còn dư ion I.
HƯỚNG DẪN CHẤM Điểm
Từ giản đồ Latimer của Iot  HIO không bền vì 0,25

E HIO/I >E IO /HIO  HIO tự oxi hóa khử thành IO3 và I 3
o o 
3 3

 Giản đồ Latimer của I được viết gọn lại:


1,70V 1,20V
H 4IO6   IO3   I3 
0,54V
I
Từ giản đồ của Mn  MnO 24 và Mn3+ không bền vì chúng có thể khử bên phải lớn hơn thế khử 0,25
bên trái  chúng sẽ tự chuyển thành hai tiểu phân ở ngay bên cạnh giống như ở HIO.
Do E oMn 2 /Mn  1,18 V  Mn không thể tồn tại trong dung dịch khi có mặt H+. Vì vậy không 0,25
cần xét quá trình Mn2+  Mn.
 Giản đồ Latimer của Mn được viết gọn lại:
1,70V 1,23V
MnO4   MnO2   Mn 2
Vì E oH IO /IO  1, 70 V  E oI /I  0,54 V và E oIO  1, 20 V  E oI /I  0,54 V 0,25
4 6 3 3 3 3

Nên H 4 IO 6 hoặc IO3 đều có thể oxi hóa I  thành I 3 .


Như vậy I  chỉ bị oxi hóa thành I 3 .
Vì E oMnO /MnO và E oMnO 2 đều lớn hơn E oI /I nên MnO 4 và MnO2đều có thể oxi hóa I  thành 0,25
4 2 2 /Mn 3

  
I nên khi I dư thì MnO và MnO2 không thể tồn tại.
3 4

Như vậy MnO 4 bị khử hoàn toàn thành Mn2+. 0,25


 Phương trình phản ứng xảy ra: 2MnO  15I  16H  5I  2Mn  8H 2O

4
  
3
2

6.2. Ở các cây cầu bê tông, cốt thép (thành phần chính là Fe) có thể bị ăn mòn. Quá trình ăn mòn này bắt đầu
như sau:
(1) Fe(r) → Fe2+(aq) + 2e
(2) O2(k) + 2 H2O(l) + 4e → 4 OH-(aq)
Một pin đã được thiết lập để xảy ra các phản ứng như quá trình ăn mòn ở trên (ở 25oC). Sơ đồ pin được
cho như sau:
Fe(r) │ Fe2+(aq) ║OH-(aq), O2(k)│Pt(r)
Cho thế tiêu chuẩn ở 25oC: Fe2+(aq) + 2e → Fe(r) ; Eo = - 0,44V
O2(k) + 2H2O(l) + 4e → 4 OH-(aq) ; Eo = 0,40V
a) Hãy tính suất điện động của pin (∆Eopin) tại 25oC.
b) Viết phương trình phản ứng xảy ra khi pin hoạt động.
c) Tính hằng số cân bằng của phản ứng trong pin tại 25oC.
d) Cho pin hoạt động trong 24h tại điều kiện chuẩn, tạo ra dòng điện không đổi 0,12A. Hãy tính khối lượng
Fe bị oxi hóa thành Fe2+ sau 24h pin hoạt động. Giả sử H2O và O2 dư trong suốt quá trình.
HƯỚNG DẪN CHẤM Điểm
a) E  E (ph¶i)-E (tr¸i)  0, 40 V  (0, 44 V)  0,84 V
0 0
0,25
b) 2Fe + O2 + 2H2O   2Fe2+ + 4OH- 0,25
G 0  nFE  4.96485.0,84 J.mol 1  3,24.105 J .mol 1 0,5
c)
G 0   RT ln K  ln K  130,7  K  5,78.1056
d) Q=I.t=0,12A.2460,60s=10368 C 0,25
n(e-)= Q/F = 10368C/(96485C.mol-1) = 0,1075 mol
m(Fe) = n(Fe)/M(Fe) = 1/2.0,1075 mol.55,85 g.mol-1 = 3,00 gam.

Bài 7 (2,5 điểm) Halogen. Oxygen – Sulfur.


7.1. Xác định các chất từ A – G và viết các phương trình phản ứng hóa học đã xảy ra.
A    I2 HNO
KNO3 , H 2 SO4
  B KOH

 D

KOH CO 2000C
N2H4

D E C

Ag

G
HƯỚNG DẪN CHẤM Điểm
A – KI; B – HIO3;C – I2O5;D – KIO3; E – HI; G – H [AgI2] 1,0
Phản ứng:
2KI + KNO3 + H2SO4 I2 + KNO2 + K2SO4 + H2O
3I2 + 10HNO3 6HIO3 + 10NO + 2H2O
3I2 + 6KOH t
0
C
5 KI + KIO3 + 3H2O
HIO3 + KOH  KIO3 + H2O
2HIO3 t
0
C
I2O5 + H2O
5CO + I2O5 5CO2 + I2
2I2 + N2H4 4HI + N2
4HI + 2Ag 2H[AgI2] + H2
Mỗi phương trình đúng được 0,125 điểm

7.2. Đơn chất A cháy trong oxy tạo ra khí B. Khí B tiếp tục bị oxi hóa tạo ra chất C khi có mặt xúc tác. B phản
ứng với H2O tạo ra axit yếu D trong khi C pư với H2O tạo ra axit mạnh E. Mặt khác, A phản ứng với khí F có
màu vàng lục tạo thành chất lỏng G màu vàng tươi, rất độc. G có hai đồng phân cấu trúc. G có thể tiếp tục bị
clo hóa tạo thành chất lỏng H màu đỏ anh đào, sôi ở 590C. Phần trăm khối lượng của A trong G và H lần lượt
là 47,41% và 31,07%. Cả G và H phản ứng với nước đều tạo ra hỗn hợp sản phẩm trong đó có A, B. Xác định
các chất từ A đến H và viết các pt phản ứng xảy ra.
HƯỚNG DẪN CHẤM Điểm
A: S; B: SO2; C: SO3; D: H2SO3; E: H2SO4; F: Cl2; 0,5
G: S2Cl2 (Gọi a là số nguyên tử S trong G, dựa vào % khối lượng của A trong G: %A = 47,41
32.a 47, 41
   M =67,5a.
MG 100

Vì G có hai đồng phân nên chọn a = 2  G: S2Cl2. Hai đồng phân của G: 0,25
Cl S Cl

S Cl S S
Cl
H: tương tự dựa vào % của S và tính chất vật lý của H nên chọn H: SCl2
Các phương trình phản ứng: 0,75
S + O2  SO2
2SO2 + O2  V2O5 ,t 0
 2SO3
SO2 + H2O H2SO3
SO3 + H2O  H2SO4
2S + Cl2  S2Cl2
S2Cl2 + Cl2  2SCl2
2S2Cl2 + 2H2O  3S + 4HCl + SO2
2SCl2 + 2H2O  S + SO2 + 4HCl

0,5
0,5

Bài 8 (2,5 điểm) Đại cương hữu cơ (Quan hệ giữa cấu trúc và tính chất)
8. 1. Sắp xếp (có giải thích) theo trình tự tăng dần tính axit của các chất trong từng dãy sau:
COOH COOH CH2COOH

a) ; ; ;
N COOH N
(A) (B) (C) (D)
b) Sắp xếp (có giải thích) theo trình tự tăng dần nhiệt độ nóng chảy của các chất sau:
COOH COOH COOH

; ;
S
N
(A) (B) (C)
HƯỚNG DẪN CHẤM Điểm
CH2COOH COOH 0,75
-I1 COOH
-I2 -I4
< C O <
< -C3 N -I3
H O N -C4
(D) (C) (A) (B)
a)
Vì: - I1 < - I2 nên (C) có tính axit lớn hơn (D).
(A) và (B) có N nên tính axit lớn hơn (D) và (C)
(A) có liên kết hiđro nội phân tử làm giảm tính axit so với (B).
b) Tăng dần nhiệt độ nóng chảy của các chất: 0,75
COOH COOH COOH

< <
S N
(C) (A) (B)
Vì:M C < MA.
(B) có thêm liên kết hiđro liên phân tử với N của phân tử khác.

8.2. Giải thích kết quả sau đây về moment lưỡng cực (đơn vị Debye, D) và cho biết chiều của momen lưỡng
cực trong mỗi trường hợp:

HƯỚNG DẪN CHẤM Điểm


- Chiều của moment lưỡng cực trong mỗi trường hợp: 1,0

- Với hợp chất (I): dễ dàng tạo vòng thơm khi chuyển dịch cặp electron từ nguyên tử cacbon của
vòng 7 về phía nguyên tử oxi, đồng thời trong hợp chất có hệ cộng hưởng và điện tích âm được
phân bố đều trên 2 nguyên tử oxi, do đó momen lưỡng cực hướng từ vòng 7 về phía giữa hai
nguyên tử oxi.
- Với hợp chất (II) tương tự như hợp chất (I) về cấu trúc của vòng, tuy nhiên do nguyên tử Br
có độ âm điện lớn hơn C nên nguyên tử Br mang 1 phần điện tích âm, dẫn tới có thêm 1
momen lưỡng cực ngược hướng với momen lưỡng cực hướng về phía oxi, dẫn tới giá trị
momen lưỡng cực của toàn phân tử hợp chất (II) nhỏ hơn so với hợp chất (I).
ĐỀ SỐ 4
Câu 1. (2,5 điểm) Cấu tạo nguyên tử, phân tử, định luật tuần hoàn
1.1. Bước sóng của quang phổ phát xạ (hoặc hấp thụ) của nguyên tử hydrogen được tính theo công thức:
1  1 1 
 R  2  2  , trong đó R = 1,0974.107 m-1 (hằng số Rydberg)
  nt nc 
a. Một vạch sóng trong dải Balmer có bước sóng  = 433,9 nm, hãy xác định nc của bước nhảy.
b. Hãy tính giới hạn trên và giới hạn dưới của dải Balmer.
1.2. Dựa vào mô hình giếng thế một chiều, hãy xác định năng lượng (kJ/mol) của 10 electron π được giải
toả đều trên toàn khung phân tử decapentaen (C10H12), biết rằng khoảng cách trung bình giữa 2 nguyên tử
C trong mạch là lC–C = 1,4 Å và 10 electron π chiếm 5 mức năng lượng ở trạng thái cơ bản. Độ dài giếng thế
được tính theo công thức gần đúng L = (N + 1) lC–C, ở đây N là số nguyên tử C trong mạch.
1.3. Đồng vị 131
53 I dùng trong y học thường được điều chế bằng cách bắn phá bia chứa 52 Te bằng neutron trong
130

lò phản ứng hạt nhân. Trong phương pháp này, trước tiên 130
52 Te nhận 1 neutron chuyển hóa thành 131
52 Te , rồi
đồng vị này phân rã   tạo thành 131
53 I . Biết chu kì bán hủy của 131
53 I là 8,02 ngày.
a. Viết phương trình các phản ứng hạt nhân xảy ra khi điều chế 131
53 I.
b. Trong thời gian 3 giờ, 1ml dung dịch 131
53 I ban đầu phát ra 1,08.1014 hạt   .
- Tính nồng độ ban đầu của 131
53 I trong dung dịch theo đơn vị mol / l .
- Sau bao nhiêu ngày, hoạt độ phóng xạ riêng của dung dịch 131 3
53 I chỉ còn 10 Bq/ml?

Câu 1 Hướng dẫn Điểm


1.1 a. Dải Balmer bao gồm các vạch ứng với sự chuyển electron từ các mức năng
(1,0đ) lượng cao n≥3 về mức năng lượng thấp nt =2. 0,25
- Vạch sóng có bước sóng λ = 433,9 nm ứng với :
1 7 2 1 1
= 1,0974. 10 = 𝑐 . ( − )
433,9.10−9 2 2 𝑛2 𝑐
1
 2  0, 04  nc  5
nc 0,25
Vậy vạch sóng có bước sóng λ = 433,9 nm ứng với sự chuyển electron từ mức
năng lượng nc = 5 về mức nt = 2.
b. Dải Balmer :
- Vạch đầu : nt = 2 ; nc =3
1 1 1
= 1,0974. 107 . (22 − 9)
𝜆max 0,25
 max  656(nm)
- Vạch cuối : : nt = 2 ; nc = 
1 1
= 1,0974. 107 . 22
𝜆 min 0,25
⇒ 𝜆min = 364(𝑛𝑚)
1.2 Trong phân tử decapentaen có 10 nguyên tử C nên độ dài giếng thế là:
(0,5đ) L = (10 + 1). l = 11x lC-C 0,25
10 electron π sẽ chiếm 5 mức năng lượng và
được biểu diễn trên giản đồ năng lượng:
Áp dụng công thức tính năng lượng:
E = n2h2/8mL2
Năng lượng của 10 electron π được giải toả đều
trên toàn khung phân tử decapentaen là:
E = 2(12 + 22 + 32 + 42 + 52) E1
= 16813258 J/mol 0,25
= 1681,3258 kJ/mol

1.3 a.
(1,0đ) 130
52 Te  0 n 52 Te
1 131
0,25

131
52 Te 131
53 I  
b.
53 I có trong 1 ml dung dịch ban đầu . Số nguyên tử
Gọi N0 là số nguyên tử 131 131
53 I
có trong 1 ml dung dịch sau thời gian t là:
ln 2
 .t
N  N 0 .e t  N 0 .e t1/2

Số hạt   phát ra trong thời gian t = 3 giờ


1, 08.1014 0,25
N 0  N  N 0 (1  e t )  1, 08.1014  N 0  ln 2
 1016 nguyên tử
 .3
1 e 8,02.24

1016 0,25
 Nồng độ ban đầu của 131
53  16, 6 mol / l
I trong dung dịch là
6, 022.1023.0, 001
Hoạt độ phóng xạ riêng (tính cho 1 ml dung dịch) ban đầu là
ln 2 ln 2
A 0  N 0  .N 0  .1016  1010 Bq / ml
t1/2 8, 02.24.60.60
t 0,25
A 1 t1/2 10 3
    10  t  186, 49 ngày
A0  2  10
Câu 2. (2,5 điểm) Cấu tạo phân tử -Tinh thể
2.1. Một kim loại M có khối lượng riêng là 5,96 g/cm3, kết tinh theo cấu trúc mạng lập phương với cạnh của
ô mạng cơ sở là 307 pm. Biết khối lượng mol nguyên tử của M là 50,94.
a. M kết tinh theo kiểu mạng lập phương nào ?
b. Số phối trí của M trong cấu trúc này là bao nhiêu? Giải thích.
c. Tính phần trăm thể tích không gian trống trong ô mạng cơ sở của M?
2.2. Từ giản đồ MO chung cho phân tử kiểu A2 và AB, hãy :
a.Viết cấu hình e của N2, O2, CO, NO và cho biết từ tính của các phân tử này
b. So sánh độ bền của các phân tử trên. Biết:
Phân tử N2 O2 CO NO
0
d(A ) 1,097 1,207 1,128 1,150

Câu 2 Hướng dẫn Điểm

a. Thể tích của ô cơ sở của M là :


24
v = (307 pm)3 = (3,07108 cm) 3 = 28,9344 10 cm3
Khối lượng của ô cơ sở là :
24 24
m = 28,9344 10 cm3 5,96 g/cm3 = 172,4493  10 gam.
Vì chưa biết M kết tinh theo kiểu mạng lập phương nào nên gọi n là số nguyên tử
M trong một ô cơ sở thì khối lượng một nguyên tử M là :
172, 4493  1024
mV  (gam)
n
Biết rằng khối lượng mol nguyên tử của M (AV) là 50,94 gam
o o 0,25
A m
Theo: V = V  N ( N là số Avogađro)
172, 4493
24
50,94 = n  10  6,022  1023
2.1 103,8489
50,94 = n n = 2,0386  n  2.
Trong mỗi ô cơ sở của mạng tinh thể M chứa 2 nguyên tử.

0,25
Mỗi ô cơ sở của mạng lập phương tâm khối có : 8 đỉnh, mỗi đỉnh chứa 1/8 nguyên
tử, 1 tâm của khối lập phương chứa 1 nguyên tử.
Tổng số : 8 đỉnh 1/8 nguyên tử mỗi đỉnh + 1 nguyên tử ở tâm = 2 nguyên tử
Vậy, M kết tinh theo kiểu mạng tinh thể lập phương tâm khối.

0,5
b. Số phối trí của 1 nguyên tử trong cấu trúc tinh thể đơn chất là số nguyên tử gần
nhất vây quanh nó. Trong cấu trúc này, M kết tinh theo kiểu mạng lập phương tâm
khối, số phối trí của
B
nguyên tử V là 8.
c. Mặt chéo mạng tế bào cơ A B A
sở của V (hình vẽ)
Độ đặc khít = E E
V2 nguyªn tö a

V« m¹ng c¬ së C

Từ hình vẽ, ta có: AD2 = D C


a D
a2 + a2= 2a2 0,5
xét mặt ABCD: AC2 = a2 + AD2 = 3a2
mặt khác, ta thấy AC = 4r = a√3
3,07  108  1,732
𝑎
 r = √3 = 4 = 1,329310 8 cm
4
4 4
V của 2 nguyên tử = 2 3 r3 = 2 3 (3,1416)( 1,3293108)3 =
19,67831024 cm3.
19,6783  10 24 cm3
24
 Độ đặc khít = 28,9344  10 cm = 0,68 hay 68%
3

 Phần trăm thể tích không gian trống trong ô mạng cơ sở của M là 32%.
* Yêu cầu học sinh vẽ giản đồ MO cho từng phân tử, sau đó: 0,25
+ Viết cấu hình e của N2, O2, CO, NO và cho biết từ tính của các phân tử này
Phân tử N2 O2 CO NO
Cấu  s2 s*2 x2 y2 z2   z2 x2 y2 x*1 *y1   x2 y2 z2   x2 y2 z2 x*1
hình
0,5
2.2 N 3 2 3 2,5
Từ tính Nghịch từ Thuận từ Nghịch từ Thuận từ
+ So sánh độ bền của các phân tử trên:
d N2 < d CO < d NO < d O 2  Độ bền N2 > CO > NO > O2

0,25
Câu 3. (2,5 điểm) Nhiệt hóa học - Cân bằng hóa học trong pha khí
3.1. Nhiệt hóa học
Xét quá trình hoá hơi 1 mol nước lỏng ở 25oC và 1at. Cho biết nhiệt dung đẳng áp của hơi nước, của nước
lỏng và nhiệt hoá hơi của nước tương ứng là: CP(H2Okhí) = 33,47 J/K.mol; CP(H2Olỏng) = 75,31 J/K.mol; H
o
hh (100 C, 1at) = 40,668 KJ/mol.
Các dữ kiện trên được chấp nhận giá trị coi như không đổi trong khoảng nhiệt độ khảo sát.
a. Tính H, S và G của hệ trong quá trình hoá hơi trên.
b. Từ kết quả thu được hãy kết luận quá trình hoá hơi của nước trong điều kiện trên có thể diễn ra hay
không? Vì sao?
3.2. Cân bằng hóa học trong pha khí
Ở 820oC hằng số cân bằng của các phản ứng:
CaCO3 CaO (r) + CO2(k) (1) K1= 0,2
C(r) + CO2(k)  2CO(k) (2) K2= 2
Trong một bình chân không dung tích 22,4 lít ở 820oC, người ta cho 1 mol CaCO3 và 1 mol C. Xác định số
mol của CO và CO2 khi hệ ở trạng thái cân bằng.
Câu 3 Hướng dẫn Điểm
Biểu diễn quá trình qua sơ đồ:

3.1
H, S, G = ?
H2O (l, 1at, 298K) H2O (k, 1at, 298K)
0.25
(I) (III)
(II)
H2O (l,1at, 373K) H2O (k, 1at, 373K)
a) Với quá trình (I): 0.25
H1 = CP(H2O, l) .(373 - 298) = 75,31 .75 = 5648,25 J/mol
T2 373
S1 = CP .ln = 75,31 .2,303lg = 16,91 J/K.mol
(H2O l) T1 298
0.25
- Với quá trình (II):
H2 = 40,668 KJ/mol
Hhh 40668
S2 = = = 109,03 J/K.mol
T 373 0.25
- Với quá trình (III): H3 = CP(H2O, k) .(298- 373) = 33,47.(-75) = -2510,25 J/mol
T2 298
S3 = CP .ln = 33,47 .2,303lg = -7,52 J/K.mol
(H2O k) T1 373
- Đối với cả quá trình: H = H1 + H2 + H3 = 43,806 KJ/mol
S = S1 + S2 + S3 = 118,42 J/K.mol;
G = 43806 – 298.118,42 = 8516,84 (J/mol) > 0
b) Quá trình trên là một quá trình đẳng nhiệt, đẳng áp nên GT,P được sử dụng để 0,25
đánh giá chiều hướng của quá trình và cân bằng của hệ.
GT,P = 8,57 KJ/mol > 0  Vậy quá trình hoá hơi này là một quá trình không thuận
nghịch nhưng không thể tự diễn ra. 0,25
Gọi x là số mol CaCO3 bị phân huỷ ; y là số mol C tham gia phản ứng
Ta có:
CaCO3 CaO (r) + CO2(k) (1) K1= 0,2
X x x
C(r) + CO2(k)  2CO(k) (2) K2= 2
y y 2y
Số mol của hỗn hợp khí: (x -y+ 2y) mol = x + y 0,25
RT 0,082.(273  820)
Từ (1) ⟹ K1 = P CO2 = 0,2 (atm) = (x – y). = (x – y).
3.2 V 22,4
2 0,25
0,2 P
x–y= = 0,05 (3) ; Từ (2) K2 = CO = 2
4 PCO2
PCO = 2PCO2  0,4 = 0,632 (atm)
0,25
RT 0,4
2y = 0,4 y= = 0,079 (mol)
V 2.4 0,25
Vậy: n CO2 = (x – y) = 0,05 (mol) ; nCO = 2y = 0,158 (mol)
Câu 4. (2,5 điểm) Động hóa học
4.1. Nitrogen oxide phân hủy thành nitrogen và oxygen ở nhiệt độ 5650C, phản ứng tỏa nhiệt.
2N2O(k)  2N2(k) + O2(k)
Phản ứng này tuân theo quy luật động học bậc 2 khi thực hiện hoàn toàn trong pha khí.
a. Nồng độ đầu của N2O là 0,108 mol.L-1, hằng số tốc độ phân hủy bậc hai của N2O là
1.10-3 L.mol-1.s-1.
Tính nồng độ N2O sau 1250 giây ở 5650C.
b. Năng lượng hoạt hóa của phản ứng bậc hai ở 5650C là 234 kJ.mol-1.
Tính hằng số tốc độ phản ứng ở 6000C?
4.2. Xét các phản ứng song song

k1
Các năng lượng hoạt hóa ứng với k1 và k2 lần lượt là 45,3 và 69,8 kJ.mol-1. Biết ở 320K thì  1, 00 .
k2
k1
Xác định nhiệt độ mà tại đó  2, 00 .
k2
Câu 4 Hướng dẫn Điểm
4.1 a) Phản ứng có bậc bằng 2 nên:
1 1 0,25
  k .t
[ N 2O]t [ N 2O]0
1 1
   1.103.l.mol 1.s 1.1250.s
[ N 2O]t 0,108.mol.l 1 0,5
1
 [ N 2O]t  0, 0951mol.l
b) Áp dụng phương trình Arrhenius:
k2 Ea 1 1
ln  (  ) 0,25
k1 R T1 T2
k2 234.103.J .mol 1 1 1
ln  (  )
1.103 l.mol 1.s 1 8,314.J .mol 1 K 1 838K 873K 0.5

 k2  3,844.103 l.mol 1.s 1


4.2 k1
*Với T = 320; 1
k2

0.25

0.25

k1
*Với  2, 00 có:
k2
0.25

0.25
Câu 5. (2,5 điểm) Cân bằng acid – base và cân bằng ít tan
Hấp thụ hoàn toàn 0,010 mol khí H2S vào nước cất, thu được 100,0 mL dung dịch A.
a. Tính nồng độ cân bằng của các ion trong dung dịch A.
b. Trộn 10,0 mL dung dịch A với 10,0 mL dung dịch FeCl2 0,02 M, thu được 20,0 mL dung dịch B. Có
kết tủa xuất hiện từ dung dịch B hay không?
c. Tính giá trị pH của dung dịch B để có thể tách được ion Fe2+ hoàn toàn ra khỏi dung dịch dưới dạng kết
tủa, biết rằng ion Fe2+ được coi là tách hoàn toàn ra khỏi dung dịch khi nồng độ còn lại của sắt (II) trong dung
dịch là 10–6 M.
d. Để điều chỉnh pH của dung dịch B đến khi kết tủa hoàn toàn ion Fe2+ (nồng độ còn lại của sắt (II) trong
dung dịch là 10–6 M) ta có thể dùng dung dịch đệm axetat. Tiến hành như sau, đầu tiên cho CH3COOH đặc
vào 20,0 mL dung dịch B đến nồng độ 0,10 M; sau đó cho từ từ CH3COONa vào dung dịch thu được đến khi
hết tủa hoàn toàn Fe2+ thì hết m (gam). Tính giá trị của m. Coi thể tích dung dịch không đổi sau khi cho thêm
đệm axetat.
Cho biết: pKS(FeS) = 17,2; pKa1(H2S) = 7,02; pKa2(H2S) = 12,90;
pKa(CH3COOH) = 4,75; *(FeOH+) = 10-5,92; M(CH3COONa) = 82.
Câu 5 Hướng dẫn Điểm
Nồng độ của H2S trong dung dịch A: C = 0,10 M.
Do Ka1(H2S) >> Ka2(H2S) và C.Ka1(H2S) >> KW, do vậy có thể bỏ qua cân bằng 0.25
phân li nấc 2 của H2S và cân bằng phân li của H2O. Cân bằng chính quyết định
a. pH trong dung dịch là:
H2S ⇌ H+ + HS- Ka1 = 10-7,02
[] (0,1 –x) x x
 [H+] = [HS ] = x = 10 ; [S ] = 10-12,9 M.
- -4,01 2- 0.25
Sau khi trộn:
C(Fe2+) = 0,01 M; C(H2S) = 0,05 M.
Nhận xét: Do Ka1(H2S) >> Ka2(H2S); C.Ka1(H2S)  C. *(FeOH+) >> KW, do
vậy có thể bỏ qua cân bằng phân li nấc 2 của H2S và cân bằng phân li của H2O.
Các cân bằng chính quyết định pH trong dung dịch là: 0.25
+ - -7,02
H2S ⇌ H + HS Ka1 = 10
2+
Fe + H2O ⇌ H +
+ FeOH * = 10-5,92
+
+ - +
b. [H ] = [HS ] + [FeOH ]
 [H+] = C(H2S). Ka1/([H+] + Ka1) + C([Fe2+]). */([H+] + *)
Thay Ka1 = 10-7,02, * = 10-5,92 vào phương trình trên 
[H+] = 1,26.10-4 M
 [Fe2+] = C([Fe2+]).[H+]/([H+] + *) = 9,905.10-3 M.
 [S2-] = C(H2S). Ka1 Ka2/([H+]2 + [H+]Ka1 + Ka1 Ka2) = 3,78.10-14 M;
 [Fe2+].[S2-] = 9,905.10-3. 3,78.10-14 = 10-16,426 > KS(FeS)
 Có kết tủa FeS xuất hiện.
0.5
Tại thời điểm kết tủa hoàn toàn Fe thì [Fe ] + [FeOH ] = 10 M
2+ 2+ + -6

*b h h 0.25
[Fe2+ ](1+ ) = 10-6 M ® [Fe2+ ] = 10-6. = 10-6.
h h +*b h +10-5,92
Mặt khác:
c. K a1.K a2 10-19,92
[S2- ] = (0,05- 0,01). 2 = 0,04. 2
h +K a1.h + K a1.K a2 h +10-7,02.h +10-19,92
[Fe2+].[S2-] = Ks = 10-17,2.

0.5
10-19,92 h
 0,04. 2  7 , 02
106. 5, 92
 1017, 2
h 10 .h  10 -19,92
h 10
 h = 10-5,09  pH = 5,09.
Phản ứng tạo kết tủa:
Fe2+ + H2S ⇌ 2H+ + FeS
Trước pư 0,01 0,05
Sau pư - 0,04 0,02
Gọi C là nồng độ CH3COONa thêm vào dung dịch để đến pH = 5,09.
d. [H  ]  0, 02  [HS  ]  ([CH 3COOH]  0,10)
107,02  105,09 
105,09  0, 02  0, 04 5,09 7,02
  (C  0,1) 5,09 4,76
 0,10  0.25
10  10  10  10 
 C = 0,278 M. 0.25
 m(CH3COONa) = 0,456 gam.
Câu 6. (2,5 điểm) Phản ứng oxi hóa khử. Pin điện (không liên quan phức chất).
Dung dịch X thu được sau khi trộn 100 ml dung dịch KMnO4 0,04M, 50 ml H2SO4 2M, 50 ml dung
dịch FeBr2 0,2M
6.1. Tính thành phần cân bằng của hệ.
6.2. Tính thế của điện cực Pt nhúng vào dung dịch X.
6.3. Thiết lập sơ đồ pin, tính sức điện động của pin được ghép bởi điện cực Pt nhúng vào dung dịch X và
điện cực calomen bão hoà. Viết phản ứng xảy ra khi pin hoạt động.

Cho biết: E 0 Fe 3 / Fe 2  0,771V ; E 0 MnO4 / Mn 2  1,51V ; Ecal  0,244V ; E Br2 / 2 Br   1,085V ;
Ka(HSO4-) = 10-2
Câu 6 Hướng dẫn Điểm
6.1 Nồng độ ban đầu các chất sau khi trộn :
(1,0đ) CKMnO4=0,02M
CFeBr2=0,05M 0,25
CH2SO4=0,5M
H2SO4 → H+ + HSO4-
0,5 0,5 0,5
KMnO4 → K+ + MnO4-
0,02 0,02 0,02
FeBr2 → Fe2+ + Br-
0,05 0,05 0,05
Do E Fe3+/Fe2+ < E Br2/ 2Br- < E MnO4-/ Mn2+ nên các phản ứng xảy ra theo thứ tự
0 0 0

5Fe2+ + MnO4- + 8H+  5Fe3+ + Mn2+ + 4H2O K = 1062,5


Bđ 0,05 0,02 0,5
Sau - 0,01 0,42 0,05 0,01 0,25
2MnO4 + 10Br + 16H  2Mn + 5Br2 + 8H2O K = 10
- - + 2+ / 71,8

Bđ 0,01 0,1 0,42 0,01


Sau - 0,05 0,34 0,02 0,025 0,25
TPGH của hệ: Fe3+: 0,05M Mn2+: 0,01M K+ : 0,02M
H+: 0,34M Br2: 0,025M
HSO4-: 0,5M Br- : 0,05M
Xét cân bằng:
HSO4- H+ + SO42- Ka = 10-2
C 0,5 0,34
 0,5 –x 0,34+ x x
x(0,34  x)
Ka =  10-2  x = 0,0137
0,5  x
Vậy TPCB của hệ: Fe3+¨: 0,05M Mn2+: 0,01M K+ : 0,02M
H+ : 0,3537M Br2 : 0,025M SO42- : 0,0137 M 0,25
- -
HSO4 : 0,4863M Br : 0,05M

6.2 Thế của điện cực Pt nhúng vào dd X được tính theo cặp Br2/Br- 0,25
(0,5đ) Br2 + 2e 2Br-
0,0592 Br2 
EBr2/ Br - = E0 Br2/ Br- + lg
Br  
2
2
0,25
0,0592 0,025
= 1,085 + lg = 1,115 V
2 (0,05) 2
6.3
(1,0đ) Vì thế của điện cực Pt nhúng vào dung dịch X = 1,115V > Ecal = 0,244V nên:
+ điện cực Pt là điện cực dương 0,25
+ điện cực Calomen là cực âm
Sơ đồ pin:
(-) Hg Hg2Cl2dd KCl dd X  Pt ( +) 0,25
Phản ứng xảy ra khi pin hoạt động:
Tại cực (-) : 2Hg + 2Cl-  Hg2Cl2 + 2e
Tại cực (+) Br2 + 2e  2Br- 0,25
2Hg + 2Cl + Br2  Hg2Cl2 + 2Br
- -

Sức điện động của pin:


Epin = EPt – Ecal = 1,115– 0,244 = 0,871 V 0,25
Câu 7. (2,5 điểm) Halogen. Oxygen-Sulfur.
7.1. Cho dòng khí chlorine đi chậm qua thủy ngân(II) oxit thu được khí màu vàng nâu A1. Hấp thụ A1 vào
dung dịch KOH ở nhiệt độ thấp được dung dịch chứa muối A2. Đun nóng dung dịch này được dung dịch chứa
muối A3. Khi nhỏ giọt H2SO4 đặc vào A3 rắn được khí màu vàng A4. Chiếu sáng A4 bằng tia tử ngoại tạo ra
chất lỏng A5 màu lục. A5 tác dụng với KCl cho muối A6 và khí màu vàng lục. Phản ứng của A4 với lượng
dư ozon tạo ra chất lỏng A7 màu đỏ thẫm, dẫn diện. A7 tác dụng với dung dịch KOH tạo ra hỗn hợp muối A3
và A6, trong khi phản ứng tương tự của A4 tạo ra hỗn hợp muối A3 và A8.
Xác định các hợp chất A1 – A8 và viết phương trình hóa học cho các phản ứng.
7.2. Để xác định hàm lượng khí độc H2S trong không khí người ta làm thí nghiệm như sau : Lấy 50 lít không
khí(D = 1,29 g/lít) nhiễm khí H2S cho đi qua thiết bị phân tích có chứa dung dịch CdSO4 dư. Sau đó axit hóa
toàn bộ hỗn hợp thu được và cho tất cả lượng H2S sinh ra hấp thụ hết vào ống đựng 10 ml dung dịch I2 0,015
M. Lượng I2 dư tác dụng vừa đủ với 12,5 ml dung dịch Na2S2O3 0,008 M. Viết các phương trình hóa học xảy
ra trong quá trình thí nghiệm và tính hàm lượng H2S trong không khí theo ppm (số microgam chất trong 1 gam
mẫu).
Câu 7 Hướng dẫn Điểm
7.1 A1: Cl2O; A2:KClO; A3: KClO3; A4: ClO2; A5: Cl2O4; 0,25
(1,5đ) A6: KClO4 ; A7: Cl2O6, A8: KClO2.
2Cl2 + HgO  HgCl2 + Cl2O
Cl2O + 2KOH  2KClO + H2O (Cl2O tương ứng với HClO)
3KClO  2KCl + KClO3
KClO3 + H2SO4(đ) K2SO4 + ClO2 + H2O + HClO4
2ClO2  as
 Cl2O4 0,125
ClO2 + O3  chất lỏng màu đỏ, dẫn điện ×10
2ClO2 + 3/2O3  Cl2O6
Cl2O6 + 2KOH  KClO3 + KClO4 (A6)
Cl2O4 + KCl  KClO4 + Cl2
2ClO2 + 2KOH  KClO3 + KClO2 + H2O
7.2 H2S + Cd2+  CdS + 2H+ (1) 0,25
(1,0đ) CdS + 2H+  Cd2+ + H2S (2)
H2S + I2  S + 2I- + 2 H+ (3)
I2 + 2S2O3  2I + 2S4O6
2- - 2-
(4) 0,25
-4
nI2 = 0,01 x 0,015 = 1,5x 10 mol
nS2O32- = 0,0125 x 0,008 = 10-4 mol
Từ (1)  (4) ⇒nH2S = nI2(3) = 1,5x10-4 - 10-4/2 = 10-4 mol 0,25
Khối lượng không khí : m = 1,29 x 50 = 64,5 gam.
Hàm lượng H2S trong không khí là:
(10-4 x 34x 106): 64,5 = 52,7 ppm 0,25
Câu 8. (2,5 điểm) Đại cương hữu cơ( Quan hệ giữa cấu trúc và tính chất)
8.1. Cho cấu tạo của hợp chất hữu cơ E

a. Hãy chỉ rõ trạng thái lai hóa của từng nguyên tử N ở cấu tạo của E
b. Proton hóa tối đa E thu được F. Hãy gán các giá trị Pka (ở 25 oC): 1,8; 6,0; 9,2 vào từng trung tâm axit
trong công thức của F. Giải thích.
8.2. Cho dãy hợp chất sau:

a. So sánh khả năng phản ứng thế electrophin của A với benzen và cho biết vị trí phản ứng ưu tiên ở A. Giải
thích.
b. So sánh nhiệt độ nóng chảy, nhiệt độ sôi của dãy hợp chất trên. Giải thích.
Câu 8 Hướng dẫn Điểm
8.1 sp 2 COOH 0,25
N
sp 3
NH 2
N sp 2
a. H (E) 0.25
- Nguyên tử N nhóm NH ở trạng thái lai hóa sp2, cặp e chưa chia ở obitan p xen
phủ với 5 obitan p khác tạo thành hệ thơm được lợi về mặt năng lượng nhưng
“mất” tính bazơ.
- Nguyên tử N thứ hai ở trạng thái lai hóa sp2, cặp e chưa chia ở obitan sp2 không
tham gia vào hệ thơm nên còn tính bazơ.
- Nguyên tử N nhóm NH2 ở trạng thái lai hóa sp3.
6,0 COOH 1,8 0,25
H N
NH 3 9,2
N
b. H 0.25
+ 3 +
- Nhóm NH3 là axit liên hợp của nhóm H2Nsp , nhóm NH là axit liên hợp của
nhóm Nsp2.
- Bazơ càng mạnh thì axit liên hợp càng yếu, vì thế giá trị 9,2 là thuộc nhóm NH3+
còn giá trị 6,0 thì thuộc nhóm NH+.
8.2 a. 0,25
- Mật độ e π ở mỗi vị trí của A (6e/5 vị trí) lớn hơn ở mỗi vị trí trong vòng benzen
(6e/6 vị trí) nên A dễ tham gia phản ứng thế electrophin hơn benzen.
- Sự tạo thành phức σ ở vị trí 2 (ở giai đoạn quyết định tốc độ phản ứng) bền hơn
ở vị trí 3 do điện tích dương

b. * Nhiệt độ nóng chảy: G > E > D > A > B > C 0,25


* Nhiệt độ sôi: G > E > D > B > C > A 0.25
* Giải thích:
- G, E và D có phân tử khối lớn hơn và có nhiều nhóm phân cực hơn so với A, B, 0,25
C;
- G tạo liên kết hiđro liên phân tử mạnh hơn E nên tonc và tos đều biến đổi
theo thứ tự: G > E > D > A, B, C. 0.25

- Ở trạng thái rắn, lực Van deVan (Fv~ p.p’/rn với n ≥ 4) phụ thuộc chủ yếu vào
khoảng cách giữa các phân tử (r). Vì r < r < r nên tonc theo giảm theo thứ tự
A B C
0.25
A > B > C.
- Ở trạng thái sôi, lực Van deVan phụ thuộc chủ yếu vào điện tích p và p’ của
lưỡng cực (vì khi đó khoảng cách giữa các phân tử quá lớn).
0.25
Vì µB> µC> µA nên tos giảm theo thứ tự B > C > A.

ĐỀ SỐ 5
Câu 1: (2,5 điểm) Cấu tạo nguyên tử. Phản ứng hạt nhân. Định luật tuần hoàn.
1.1. X là nguyên tố thuộc nhóm A, hợp chất với hydrogen có dạng XH3. Electron cuối cùng trên nguyên tử X
có tổng 4 số lượng tử bằng 4,5.
a) Xác định nguyên tố X và viết cấu hình electron nguyên tử của X.
b) Ở điều kiện thường XH3 là một chất khí. Viết công thức cấu tạo của XH3, oxide bậc cao nhất và hydroxide
bậc cao nhất của X.
1.2. Khi phóng tia lửa điện qua các nguyên tử hydrogen ở áp suất thấp, các electron bị kích thích lên trạng thái
năng lượng cao hơn. Sau đó, electron nhanh chóng chuyển về mức năng lượng cơ bản (n = 1) và bức xạ ra
photon với các bước sóng khác nhau tạo thành dãy phổ. Tính bước sóng (λ) nhỏ nhất và bước sóng lớn nhất
theo nm của dãy phổ nếu electron chuyển từ n > 1 về n = 1.
Biết trong hệ một electron, một hạt nhân, năng lượng của electron được tính theo công thức:
Z2
En = - 13,6. 2 (eV). Cho: h = 6,626.10-34 J.s; c = 3.108 m/s; 1eV = 1,6.10-19J.
n
1.3. Trong mặt trời, có xảy ra một chuỗi các phản ứng hạt nhân nằm trong chu trình cacbon-nitơ như sau:
1 H  6 C  A   (1) ; A  B  01 e (2); 1 H  B  C   (3);
1 12 1

1
1H  C  D   (4); D  E  01 e (5); 1 H  E  6 C  F   (6).
1 12

a) Hoàn thành các phản ứng hạt nhân trên, viết phương trình tổng quát cho chu trình carbon - nitrogen.
b) Hạt nhân nào được coi là xúc tác của quá trình? Hạt nhân nào được coi là hạt nhân trung gian?
c) Tính năng lượng giải phóng ra nếu có 1 gam 1H tham gia vào chu trình này.
Cho: Khối lượng mol nguyên tử của 1H và F lần lượt là 1,00782 g/mol và 4,00260 g/mol. Khổi lượng của
positron 01 e là 9,10939.10-28 g. Hằng số Avogadro N = 6,022136.1023. Tốc độ ánh sáng trong chân không c =
2,998.108 m/s.

Câu 1 Nội dung Điểm


1.1. a) Với hợp chất với hydrogen có dạng XH3 nên X thuộc nhóm IIIA hoặc nhóm VA
* TH1: X thuộc nhóm IIIA, ta có sự phân bố electron theo orbital:

0,5
Vậy electron cuối cùng có: l = 1, m = -1, mS = +1/2
Mà n + l + m + mS = 4,5  n = 4
Cấu hình electron nguyên tử X: 1s22s22p63s23p63d104s24p1 (Ga)
* TH2: X thuộc nhóm VA, ta có sự phân bố electron theo orbital:

Vậy electron cuối cùng có: l = 1, m = +1, mS = +1/2


Mà n + l + m + mS = 4,5  n = 2
Cấu hình electron nguyên tử X: 1s22s22p3 (N).
b) Ở điều kiện thường XH3 là một chất khí nên nguyên tố X phù hợp là Nitrogen
Hợp chất XH3 Oxide bậc cao nhất Hydroxide bậc cao nhất
của X của X
CTPT NH3 N2O5 HNO3 0,5
CTCT O O O
N
H N O N H O N
H
H
O O O

1.2. - Bước sóng dài nhất ứng với sự chuyển từ mức n = 2 về mức n = 1:
Emin  E2  E1  1, 63.1218 ( J )
hc 6, 626.1034.3.108
 max   18
 1, 22.107 (m)=122 (nm)
Emin 1, 63.10 0,5
- Bước sóng ngắn nhất ứng với sự chuyển từ mức n =  về mức n = 1:
Em ax  E  E1  2,18.1218 ( J )
hc 6, 626.1034.3.108
 min   18
 91, 2.109 (m)=91,2 (nm)
Em ax 2,18.10
1.3. a) 11 H  126 C  137 N  00  (1) ; 13
7 N  136 C  01 e (2)
1
1 H  136 C  147 N  00  (3) ; 1
1 H  147 N  158 O  00  (4)
0,5
15
8 O  157 N  01 e (5) ; 1
1 H  157 N  126 C  24 He  00  (6)
Phương trình tổng: 4 11 H  42 He  2 01 e
b) Trung gian: 13N, 13C, 14N, 15O, 15N.
Xúc tác: 12C 0,25

c) Độ hụt khối tính cho 1 mol phản ứng chung (4 mol H) là:
∆m = 4.(MH – me) – (MHe – 2me) – 2me = 4MH – MHe – 4me
 ∆m = 4. 1,00782 – 4,00260 – 4. 9,10939.10-28. 6,022136.1023 = 0,02649 g
 ∆E = ∆m.c2 = 0,02649.10-3. (2,998.108)2 = 2,3809.1012 J 0,25
Tính cho 1g 1H thì năng lượng giải phóng:
2,3809.1012 1
. = 5,9052.1011 J
4 1, 00782

Câu 2: (2,5 điểm) Cấu tạo phân tử. Tinh thể.


2.1. Xác định cấu trúc phân tử của các phân tử và ion sau đồng thời cho biết kiểu lai hóa các AO hóa trị của
nguyên tử trung tâm: SOF4, TeCl4, BrF3, I3-, ICl4-?
2.2. Sử dụng phương pháp cặp electron hay phương pháp liên kết hoá trị (viết tắt là VB –
Valence Bond) và mô hình VSERP hãy cho biết sự tạo thành liên kết, trạng thái lai hóa của
nguyên tử trung tâm, dạng hình học của phân tử SO 2 Cl2 .
2.3. Chromium (Cr) có cấu tạo mạng tinh thể lập phương tâm khối, bán kính nguyên tử của Cr là 1,26 Å. Khối
lượng mol nguyên tử của Cr là 52 g/mol. Xác định khối lượng riêng của Cr và độ đặc khít của mạng tinh thể
trên.

Câu Nội dung Điểm


2
2.1.
Chất Trạng thái lai hóa Dạng hình học của phân tử
SOF4 sp3d lưỡng tháp tam giác
TeCl4 sp3d bập bênh
BrF3 sp3d hình chữ T cụp 0,5
I-3 sp3d thẳng
ICl-4 sp3d2 vuông phẳng

2.2. Xét phân tử SO 2 Cl2

0,5

Ở trạng thái kích thích S lai hoá sp3 với cấu trúc tứ diện, góc lai hoá 1090 28 . Khi hình
thành phân tử, 4 AOsp3 chứa electron độc thân sẽ xen phủ với 2 AO 2p chứa electron độc
thân của hai nguyên tử O và 2 AO3p chứa electron độc thân của hai nguyên tử Cl tạo 4
liên kết  . Ngoài ra, còn có sự xen phủ bên của 2 AO3d2 chứa electron độc thân của S với
2 AO 2p chứa electron độc thân của 2 nguyên tử O tạo hai liên kết  .
Do các electron trên liên kết S = O đẩy mạnh hơn trên liên kết S - C1 nên góc liên kết
khác góc lại hoá ban đầu và phân tử có cấu trúc tứ diện lệch.
Sơ đồ cấu tạo phân tử:

0,5

Cấu trúc phân tử:

2.3. Hình vẽ:

0,5

Trong một ô mạng cơ sở có chứa:


1
8.  1 = 2 nguyên tử Cr
8
Khối lượng Cr trong một ô cơ sở:
2. 52
m= = 1,727. 1022 (g)
6, 022.1023
Gọi a là độ dài của cạnh hình lập phương, r là bán kính nguyên tử Cr
Vì Cr có cấu tạo mạng tinh thể lập phương tâm khối nên: 4r  a 3
4r 4. 1, 26
 a= = = 2,91 (Å) = 2,91. 108 (cm)
3 3
Khối lượng riêng của Cr: 0,5
m 1, 727.1022
D= = 8 3
= 7,01 (g/cm3)
V (2,91.10 )
4 4
2. r 3 2. .(1, 26)3
Vkim loai
Độ đặc khít = .100% = 3 3 .100% = 3 .100% = 68%.
Vhinh lap phuong a (2,91)3

Câu 3 (2,5 điểm) Nhiệt hóa học. Cân bằng hóa học trong pha khí.
3.1. Đưa 81 gam hơi nước, được duy trì ở 373 K, vào một hỗn hợp nước (1350 gam) và nước đá (135 gam)
đang đạt cân bằng ở 0oC. Tính nhiệt độ cuối của nước.
Cho biết:
- Enthalpy hóa hơi chuẩn của nước (lỏng): 44,1 kJ.mol-1.
- Enthalpy nóng chảy chuẩn của nước đá: 5,98 kJ.mol-1.
- Nhiệt dung riêng đẳng áp của nước (lỏng): 75,3 J.mol-1.K-1.
3.2. Kristian Birkeland, là người đầu tiên mô tả cách thức các hạt mang điện có nguồn gốc từ Mặt trời tương
tác với từ trường của Trái đất để tạo ra hiện tượng cực quang. Ông có thể cải tiến thiết kế súng điện từ để sản
xuất nitric acid làm phân bón nhân tạo. Phương pháp này còn được gọi là phương pháp Birkeland. Phương
pháp này tạo ra một dòng quang điện làm tăng nhiệt độ khí lên 3000oC, sử dụng để điều chế nitric acid bằng
cách biến đổi N2 khí quyển qua các công đoạn sau:

 2NO(g) (1)
N2(g) + O2(g) 


 2NO2(g) (2)
2NO(g) + O2(g) 


3NO2(g) + H2O(l)   2HNO3(g) + NO(g) (3)
a) Tại sao ngày nay phương pháp Birkeland không còn được dùng trong sản xuất công nghiệp nữa ?
b) Quá trình sản xuất công nghiệp nitric acid hiện nay khác gì với phương pháp Birkeland?
c) Hằng số cân bằng Kc của phản ứng (2) là 1,65.105. Thí nghiệm sau được tiến hành để nghiên cứu các
đặc trưng của phản ứng này. Nạp khí NO2 và NO vào bình phản ứng với dung tích 0,100 m3 chứa một lượng
chưa xác định oxygen. Nồng độ khí NO2 ban đầu là x mol/L và nồng độ khí NO ban đầu là (x + 0,0024)
mol/L. Ở cân bằng, nồng độ tổng của tất cả các khí trong bình phản ứng là 0,2246 mol/L và có chứa chính
xác 2 mol O2. Tính số phân tử
oxygen trong bình phản ứng trước khi bắt đầu thí nghiệm.
d) Ở nhiệt độ nào, hiệu suất tạo ra NO cao hơn: 25oC hay 3000oC ? Giải thích.
Dữ kiện nhiệt động học (25oC, 1 bar):
Chất  H o (kJ/mol)
f 298 So (J/mol.K)
298
O2 (g) 0 205,03
NO (g) 90,25 210,65
N2 (g) 0 191,61

Câu 3 Nội dung chính cần đạt Điểm


- Để làm tan hoàn toàn nước đá (7,5 mol) cần: Q = 7,5.5,98 = 44,85 kJ
0,25
- Quá trình ngưng tụ hơi nước (4,5 mol) giải phóng lượng nhiệt:
0,25
Q’ = 4,5.44,1=198,45 kJ
- Lượng nhiệt cần để làm nóng hỗn hợp nước (75 + 7,5 = 82,5 mol) từ 0oC lên
0,25
ToC là: 82,5.75,3.(T – 0) (J)
3.1 - Lượng nhiệt giải phóng ra khi ngưng tụ 4,5 mol hơi nước và tạo thành 4,5
0,25
mol nước ở 100oC, rồi làm nguội về ToC là:
Q’’ = 198,45.103 + 4,5.75,3.(100 – T)
Từ đó ta có:
198,45.103 + 4,5.75,3.(100 – T) = 44,85.103 + 82,5.75,3.(T – 0)
→ T = 28,7oC
a) Sản xuất nitric acid bằng phương pháp Birkeland tiêu tốn nhiều năng lượng 0,25
nên ngày nay không còn được sử dụng.
b) Trong phương pháp Birkeland, phương pháp điều chế NO2 là tạo ra hồ 0,25
quang điện, còn trong công nghiệp hiện nay là oxi hóa ammonia.
c) CO2 = 0,02 M

2NO(g) + O2(g)    2NO2(g) (2)



Ban đầu: x + 0,0024 0,02+ y x
3.2
Phản ứng: 2y y 2y
Cân bằng: x + 0,0024-2y 0,02 x + 2y 0,5
(x  2y)2
Kc   1, 65.105 (*)
(x  0, 0024  2y) .0, 02
2

Mặt khác: x + 0,0024 – 2y + 0,02 + x + 2y = 0,2246 (**)


Giải (*) và (**) ta được: x = 0,1011 và y = 0,05
→ CO2 (ban đầu) = 0,02 + 0,05 = 0,07 M
n O2 (ban đầu) = 0,07.100 = 7 mol.
Số phân tử O2 = 7.6,022.1023 = 42,154.1023 (phân tử)
d) Tại 25oC:
 r H o298 = 2.90,25 = 180,5 kJ.
0,25
 rSo298 = 2.210,65 - 205,03 – 191,61 = 24,66 J/K
 r G o298   r H o298  T. rSo298 = 180,5.103 – 298.24,66 = 173151,32 J
K 298 ≈ 4,449.10-31
Tại 3000oC:
K  Ho  1 1  0,25
ln 3273  r 298    → K 3273 ≈ 0,0256
K 298 R  298 3273 
Vì K 3273 > K 298 nên ở 3000oC cho hiệu suất cao hơn.

Câu 4 (2,5 điểm) Động hóa học (không có cơ chế).


4.1. Hydrocarbon 4 vòng (A) có nhiều tiềm năng trong công nghệ năng
lượng Mặt trời. Dưới tác động của bức xạ tử ngoại, chất B bị đồng phân
hóa thành A. Phản ứng chuyển hóa ngược lại A → B là quá trình tỏa
nhiệt  r H o298 = -92,5 kJ). Đây là một tính chất đầy hứa hẹn để chế tạo
các thiết bị lưu trữ năng lượng Mặt trời. Xét một pin năng lượng Mặt
trời với 10 mol B. Dưới tác động của ánh sáng, B chuyển thành A với
độ chuyển hóa 85 %, sau đó pin không
bị chiếu xạ nữa.
a) Tính năng lượng cực đại nhận được từ mẫu pin này nếu hiệu suất của bộ thu là 65%.
b) Vấn đề chính khi sử dụng chất vòng 4 cạnh này là việc lựa chọn các hệ xúc tác để làm tăng tốc độ phản
ứng tỏa nhiệt A → B tại nhiệt độ sử dụng pin. Khi không có xúc tác, phản ứng đồng phân hóa nhiệt của A ở
nhiệt độ phòng gần như không xảy ra mà chỉ bắt đầu ở các nhiệt độ cao hơn (chu kì bán chuyển hóa ở 160oC
là 2,58 giờ; còn ở 200oC là 5,14 phút). Tính năng lượng hoạt hóa của phản ứng đồng phân hóa A → B nếu
phản ứng này diễn ra theo quy
luật động học bậc nhất.
c) Sau bao lâu bộ thu có thể tiếp nhận 90% năng lượng tích lũy trong hợp chất A ở 25oC nếu không có
xúc tác.
4.2. Tốc độ phản ứng: NO2(g) + CO(g) → NO(g) + CO2(g) chỉ phụ thuộc vào nồng độ của nitrogen dioxide ở
nhiệt độ dưới 225oC. Ở nhiệt độ dưới 225oC, thu được các dữ liệu sau đây. Xác định biểu thức động học dạng
tích phân, dạng vi phân và giá trị hằng số tốc độ ở nhiệt độ này. Tính [NO2] ở 2,70.104 giây sau khi phản ứng
bắt đầu.

Thời gian
0 1,2.103 3,00.103 4,5.103 9,00.103 1,8.104
(s)
[NO2]
0,500 0,444 0,381 0,340 0,250 0,174
(mol/L)

Câu 4 Nội dung chính cần đạt Điểm


a)
0,5
Emax = 10 . 0,85 . 0,65 . 92,5 = 511,0625 kJ
b) Đối với phản ứng bậc nhất: k = ln2/t1/2
Sử dụng phương trình Arrhenius:
4.1 k E  1 1 t1/2(2) Ea  1 1 
ln 1  a .    → ln  .   0,5
k 2 R  T2 T1  t1/2(1) R  T2 T1 
Thay các giá trị: t1/2(1) = 154,8 phút; t1/2(2) = 5,14 phút; T1 = 433K; T2 = 473K
ta được Ea ≈ 144,953 kJ/mol.
c) k433 = ln2/ t1/2(433) = 4,478.10-3 (phút-1)
k E  1 1 
ln 433  a .    → k298 = 5,360.10 (phút )
-11 -1
k 298 R  298 433  0,5
o
C
ln  kt → t = ln10/k = 4,296.1010 phút
C
Giả sử phản ứng có bậc 1 và 2
Thời gian (s) [NO2] (mol/L) ln[NO2] 1/[NO2]
0 0,500 -0,693 2,00
3
1,2.10 0,444 -0,812 2,25
3,00.103 0,381 -0,965 2,62
3
4,5.10 0,340 -1,079 2,94
9,00.103 0,250 -1,386 4,00 0,25
4
1,80.10 0,174 -1,749 5,75

4.2

Đồ thị 1/[NO2] – thời gian là đường tuyến tính, vậy phản ứng có bậc 2 theo
NO2.
Biểu thức động học dạng vi phân và tích phân lần lượt là: 0,25
v = k.[NO2]2 và 1/[NO2] = kt + 1/[NO2]o

y 5, 75  2, 00
Độ dốc = k =   2, 083.104 (L.mol-1.s-1) 0,5
x 1,80.104  0
[NO2] ở 2,70.104 giây sau khi phản ứng bắt đầu:
Thế k = 2,083.10-4 (L.mol-1.s-1); t = 2,7.104 giây, [NO2]o = 0,500M vào
1/[NO2] = kt + 1/[NO2]o → [NO2] = 0,131 M

Câu 5 (2,5 điểm) Cân bằng acid – base và cân bằng ít tan.
5.1. Dung dịch A là hỗn hợp gồm CH3COOH 0,1M; HCOOH 0,2M; H2SO4 0,01M. Thêm 0,58 mol NH3 vào
1 lít dung dịch A, được dung dịch B.
a) Tính pH của dung dịch B. Giả sử thể tích dung dịch không thay đổi.
b) Tính V ml dung dịch NaOH 0,1M cần cho vào 25 ml dung dịch A để thu được dung dịch có pH = 4,76.
Cho biết: pK a ( CH COOH) = 4,76; pK a ( HCOOH ) = 3,75; pK a  = 9,24; pK a   2
3 ( NH ) ( HSO4 )
4

5.2. Một trong các phương pháp để tách loại Cr (VI) trong nước thải của quá trình
mạ điện là khử Cr (VI) về Cr (III) trong môi trường acid, sau đó điều chỉnh pH bằng kiềm để kết tủa Cr(OH)3.
Nếu nồng độ ban đầu Cr3+ trong nước thải (sau khi đã khử Cr (VI) về Cr (III) là 10-3M). Khi tăng pH của dung
dịch (coi thể tích dung dịch không đổi), ban
đầu sẽ tạo thành kết tủa Cr(OH)3 có tích số tan bằng 10-30, sau đó kết tủa Cr(OH)3 sẽ tan ra do tạo thành ion
Cr(OH)4- theo phản ứng:
Cr(OH)3 + OH-   Cr(OH)4- có pK = 0,4.
Giả thiết Cr(III) chỉ tồn tại ở 3 dạng: dạng tan là Cr3+ và Cr(OH)4-; dạng kết tủa là Cr(OH)3. Hãy xác định:
a) pH của dung dịch khi bắt đầu xuất hiện kết tủa Cr(OH)3.
b) pH của dung dịch khi kết tủa Cr(OH)3 tan hoàn toàn thành Cr(OH)4-

Câu 5 Nội dung chính cần đạt Điểm


a) Do NH3 rất dư so với các axit trên:
2NH3 + H2SO4  (NH4)2SO4 0,25
0,02 0,01 0,01
NH3 + HCOOH  HCOONH4
0,2 0,2 0,2
NH3 + CH3COOH  CH3COONH4
0,1 0,1 0,1

Thành phần giới hạn: NH3 0,26M; NH 4 0,32M; CH3COO- 0,1M; HCOO- 0,25
2
0,2M; SO 0,01M 4
Cân bằng:
NH 4 NH3 + H+ (1) K1 = 10-9,24
H 2O H  + OH  (2) K2 = 10-14

NH3 + H2O NH 4 + OH- (3) K3 = 10-4,67
CH3COO- + H2O CH3COOH + OH- (4) K4 = 10-9,24 0,25
HCOO- + H2O HCOOH + OH- (5) K5 = 10-10,25
SO 24 + H 2O HSO 4 + OH  (6) K6 = 10-12
So sánh (1) và (2) thấy (1) là chủ yếu.
So sánh (3), (4), (5), (6) thấy (3) là chủ yếu.
5.1 Vậy cân bằng (1) và (3) là chủ yếu hay có thể coi dung dịch là một hệ đệm
gồm
NH 4 0,32M và NH3 0,26M
Cb 0, 26
pH  pK a  lg  9, 24  lg.  9,15 . Thỏa mãn điều kiện cân bằng 0,25
Ca 0, 32
của hệ đệm
 4,76
b) Khi cho NaOH vào dung dịch A để có pH = 4,67  [H ]  10
 102
%HSO bÞ trung hßa: 1 = 2  100%
10  104,76
4

103,75
%HCOOH bÞ trung hßa:  2  3,75  91,098% 0,5
10  104,76
104,76
%CH 3COOH bÞ trung hßa: 3  4,76  50%
10  104,76
25.(2C H 2SO4  0, 91098.C HCOOH  0, 5.CCH3COOH )  V.0,1  V  63, 049 ml
1030
a) [OH-] = 3
3
 109 M → [H+] = 10-5 M → pH = 5 0,5
10
b) Kết tủa Cr(OH)3 tan hoàn toàn thành Cr(OH)4-, khi đó có thể xem
5.2 [Cr(OH)4-] = [Cr3+] = 10-3 M
[Cr(OH) 4 ] 103 0,5
→ [OH ] =
-
 0,4  2, 51.103
K 10
pH = 11,4.

Câu 6: (2,5 điểm) Phản ứng oxi hóa - khử. Pin điện (không liên quan đến phức chất).
6.1. Cho: E oMnO ,H  / Mn 2 = 1,51 V; EoMnO2 ,H / MnO = 2,26 V; E oMnO ,H  / Mn 2 = 1,23 V.
4 4 2 2
a) Tính EoMnO ,H và E oMnO / MnO2 .
4 2O / MnO 2 4 4

b) Nhận xét về khả năng oxi hóa của ion MnO 4 trong môi trường acid, trung tính và base. Giải thích.
6.2. Điện cực loại II là điện cực tạo bởi kim loại được bao phủ bởi muối ít tan của kim loại đó, nhúng vào
dung dịch muối tan chứa anion của muối ít tan. Ví dụ như điện cực bạc/bạc clorua (Ag, AgCl/Cl-) và điện cực
calomel (Hg, Hg2Cl2/Cl-). Suất điện động của một tế bào điện hóa: (-) Ag,AgCl/KCl/Hg2Cl2/Hg (+) là E0 =
0,0455 V ở T = 298 K. Hệ số nhiệt độ của tế bào này là: dE0/dT = 3,38.10-4 V K-1.
a) Cho biết phương trình phản ứng xảy ra ở cả hai điện cực và phản ứng tổng cộng.
b) Tính năng lượng tự do Gibbs (ΔG0) cho quá trình diễn ra trong pin điện ở 298 K. Cho biết ý nghĩa dấu
của ΔGo.
c) Tính biến thiên enthalpy cho quá trình ở 298 K, biết rằng ΔS = nFΔE/ΔT.
d) Biết rằng thế chuẩn của Ag/Ag+ là E0 = 0,799 V và tích số tan của AgCl là Ksp = 1,73.10-10, tính giá trị
thế điện cực chuẩn của điện cực bạc/bạc clorua. Thiết lập phương trình cho biết sự phụ thuộc giữa E0(Ag/Ag+)
và E0(Ag, AgCl/Cl-).

Câu 6 Nội dung Điểm


6.1. a) Tính Eo  và E oMnO / MnO2 : 0,5
MnO ,H 4 2O / MnO 2 4 4
5.1,51
MnO 4 +
+ 8H + 5e 2+
Mn + 4H2O K1 = 10 0,0592

2.1,23
Mn 2+
+ 2H2O MnO2  + 4H + 2e +
K 2 1 = 10 0,0592

2.2,26
MnO2  + 2H2O MnO 24 +
+ 4H + 2e K 31 = 10 0,0592

Eo
MnO 2
4 / MnO4
0,5
MnO 4 + 1e MnO 24 K4 = 10 0,0592

K4 = K1. K 21 . K 31  E oMnO / MnO2 = 5.1,51 – 2.1,23 – 2.2,26 = 0,57 V


4 4
5.1,51
MnO 4 +
+ 8H + 5e 2+
Mn + 4H2O K1 = 10 0,0592

2.1,23
Mn 2+
+ 2H2O MnO2  + 4H + 2e +
K 2 1 = 10 0,0592

4 H2 O H+ + OH  K W = 1014
3Eo
MnO
4 / MnO2

MnO 4 + 2H2O + 3e MnO2  + 4 OH K5 = 10 0,0592
5.1,51 - 2.1,23 - 14.4.0,0592
K5 = K1. K 21 . K 4w  EoMnO ,H O / MnO = = 0,59 V.
4 2 2 3

E oMnO ,H  / Mn 2 > EoMnO ,H > E oMnO / MnO2


4 4 2O / MnO 2 4 4

 Khả năng oxi hóa của MnO 4 mạnh nhất trong môi trường acid và yếu nhất 0,5
trong môi trường base, bởi vì:
0, 0592 [MnO-4 ].[H + ]8
E MnO ,H  / Mn 2 = E oMnO ,H  / Mn 2  lg
4 4 5 [Mn 2+ ]
Do đó khi pH tăng, [H+] giảm, tính oxi hóa của MnO 4 giảm.
6.2. 1
a) Sự khử (điện cực calomen): Hg 2Cl2  1e  Hg  Cl 
2
0,25
Sự oxi hóa (điện cực Ag/AgCl): Ag  Cl  AgCl  1e
1
Phản ứng tổng cộng: Ag  Hg 2Cl2 
 Hg  AgCl
2

b) Năng lượng tự do Gibbs cho phản ứng xảy ra ở trên:


∆𝐺 0 = −𝑛𝐹𝐸 0 = −96497.0,0455 = −4,39 (kJ mol−1 ) 0,25
Vì ΔGo âm, phản ứng tự xảy ra.
c) Sự thay đổi của entanpy có liên hệ với phương trình Gibbs-Helmholtz:
∆𝐸 0 ∆𝐸 0
∆𝐻 = ∆𝐺 + 𝑇∆𝑆 = 𝑛𝐹𝐸 = 𝑇𝑛𝐹 ( ∆𝑡 ) = −𝑛𝐹 (𝐸 − 𝑇 ( ∆𝑡 )) 0,25
= −96497.(0,0455 – 298.3,38.10−4) = 5,36 (kJ mol−1 )
d) Đối với cặp Ag + |Ag E = 𝐸 0 + 0,0592.log[Ag + ]
K sp
Đối với cặp AgCl |𝐴𝑔, Cl− , [Ag+] được xác định bởi: [Ag+] =
[Cl ] 0,25
𝐸 0 (AgCl|Ag, Cl− ) = 𝐸 0 (Ag|Ag + ) + 0,0592. log 𝐾𝑠𝑝 = 0,799 − 0,577 = 0,222 (V)

Câu 7: (2,5 điểm) Halogen. Oxygen - Sulfua.


7.1. Xác định các chất A, B, C, D, E và viết phương trình hóa học của các phản ứng xảy ra theo sơ đồ:

7.2. Nung hỗn hợp A gồm sắt và lưu huỳnh sau một thời gian được hỗn hợp rắn B. Cho B tác dụng với dung
dịch HCl dư, thu được V1 lít hỗn hợp khí C. Tỉ khối của C so với hiđro bằng 10,6. Nếu đốt cháy hoàn toàn B
thành Fe2O3 và SO2 cần V2 lít khí oxi.
a) Xác định tỉ lệ V1 và V2 (đo ở cùng điều kiện).
b) Tính hàm lượng phần trăm các chất trong B theo V1 và V2.
c) Tính hiệu suất thấp nhất của phản ứng nung ở trên.
d) Nếu hiệu suất của phản ứng nung trên là 75%, tính hàm lượng phần trăm các chất trong hỗn hợp B.

Câu 7 Nội dung Điểm


7.1. Xác định các chất: A là KI, B là HIO 3, C là I2O5, D là KIO3, E là HI

(1) 2KI + KNO3 + H2SO4   I2 + KNO2 + H2SO4 + H2O 0,5


(2) 3I2 + 10HNO3 
 6HIO3 + 10NO + 2H2O
to
(3) 3I2 + 6KOH  5KI + KIO3 + 3H2O

(4) HIO3 + KOH 


 KIO3 + H2O
200 C o 0,5
(5) 2HIO3  I2O5 + H2O
o
t
(6) I2O5 + 5CO  I2 + 5CO2
(7) 2I2 + N2H4 
 4HI + N2
(8) HI + KOH 
 KI + H2O.
7.2. • Nung hỗn hợp A gồm sắt và lưu huỳnh:
o
Fe + S  t
 FeS (1)
 Thành phần B gồm có FeS, Fe và có thể có S
FeS + 2HCl   FeCl2 + H2S  (2)
Fe + 2HCl   FeCl2 + H2  (3)
Vậy trong C có H2S và H2 0,5
Gọi x là % số mol của H2 trong hỗn hợp C
2x  34.(100  x)
Ta có = 10,6. 2 = 21,2
100
 x = 40%
Vậy trong C: %n H 2 = 40%; %n H 2S = 60%
a) Đốt cháy B:
o
4FeS + 7O2 
t
 2Fe2O3 + 4SO2 (4)
o
4Fe + 3O2 
t
 2Fe2O3 (5)
o
S + O2  t
 SO2 (6)
Thể tích O2 đốt cháy FeS là: (3V1/5).(7/4) = 21V1/20. 0,25
Thể tích O2 đốt cháy Fe là: (2V1/5) . (3/4) = 6V1/20.
Tổng thể tích O2 đốt cháy FeS và Fe là: 21V1/20 + 6V1/20 = 27V1/20
Thể tích O2 đốt cháy S là: V2 – (27V1/20) = V2 – 1,35 V1
Vậy V2 ≥ 1,35. V1
b)
3V1
.88.100
5 5280V1 165V1
%FeS    %
3V1
.88 
2V1
.56  32(V2  1,35V1 ) 75, 2V1  32(V2  1,35V1 ) V2  V1
5 5
2V1 0,25
.56.100
70V1
%Fe  5  %
32(V2  V1 ) V2  V1
32(V2  1,35V1 ) x100 100V2  135V1 )
%S   %
32(V2  V1 ) V2  V1
c) Nếu dư S so với Fe thì tính hiệu suất phản ứng theo Fe
Trường hợp này H = 60%. Nếu dư Fe so với S tính hiệu suất phản ứng theo S
Trường hợp này H > 60%. Vậy hiệu suất thấp nhất của phản ứng nung trên là 60%. 0,25

d) Nếu H = 75% có nghĩa là nFeS = 3ns dư. nFeS tỉ lệ 3V1/5. Vậy nS tỉ lệ với V1/5
5280V1 5280V1
% FeS    64,7%
32V1 81,6V1
75,2V1 
5
0,25
2240V1
% Fe   27,45%
81,6V1
%S = 100 - (64,7+27,45) = 7,85%.

Câu 8 (2,5 điểm) Đại cương hữu cơ (Quan hệ giữa cấu trúc và tính chất)
8.1. Xem xét hợp chất hữu cơ dưới đây:
a) Hợp chất hữu cơ trên có bao nhiêu đồng phân quang học. Biểu diễn theo
công thức Fisher của từng đồng phân trên.
b) Chỉ ra các loại liên kết hiđro nội phân tử có thể có của hợp chất trên, đồng
thời chỉ ra liên kết bền nhất, kém bền nhất.
8.2.
a) Cho 3 chất sau đây: acid một nấc carboxylic
(1) ; picric acid (2) và acid hai nấc styphnic (3). Chất
đầu tiên được dùng để khử trùng ở các cơ sở y tế, và hai
chất sau
được dùng làm chất nổ. Picric acid có thể được tạo
thành từ carboxylic acid
bằng cách nitro hóa. Còn styphnic acid là sản phẩm tạo
thành khi thay thế
hydrogen trong picric acid. Nhóm chức nào quyết định tính acid của các acid (1); (2); (3). Chất nào có tính
acid mạnh nhất ? Giải thích ?
b) Ngoài buta-1,3-diene, vẫn còn có một số hydrocarbon khác cùng công thức C4H6. Xác định công
thức cấu tạo của chúng. Những hợp chất này có tính quang hoạt không? Ở thời điểm hiện tại, tất cả các đồng
phân C4H6 có thể có đều đã được tổng hợp và nghiên cứu chi tiết. Dưới đây là bảng dữ liệu về một số tính chất
của chúng. Hãy bổ sung vào các ô trống.

Số loại proton Số loại carbon Sự tồn tại trục Sự tồn tại mặt
STT Cấu tạo
tương đương tương đương đối xứng phẳng đối xứng
1 3 2 Có Có
2 3 4 Không Có
3 3 4
4 3 3
5 2 3
6 2 2
7
8 1

Câu 8 Nội dung chính cần đạt Điểm


a)

0,5

b)

8.1 0,5

I bền nhất, IV kém bền nhất.


a) Nhóm phenolic hydroxyl quyết định tính acid của các acid này. Acid 2
mạnh hơn do có sự bền hóa bổ sung bởi các nhóm nitro với anion tạo thành 0,5
trong quá trình phân li acid.
b) Các đồng phân cấu tạo có công thức phân tử C4H6

8.2 0,5

Không có hợp chất nào có tính quang hoạt.


0,5

ĐỀ SỐ 6
Cho biết: Hằng số Plank h = 6,626.10-34 J.s; Hằng số Faraday F = 96485 C.mol-1;
Tốc độ ánh sáng c = 3.108 m.s-1; Số Avogadro NA = 6,022.1023 mol-1; R = 8,314 J.mol-1.K-1;
1eV = 1,602.10-19 J; 1uc2 = 931,5 MeV
Bài 1. (2,5 điểm)
1.1. Ứng với các tiểu phân gồm một hạt nhân và 2 electron như: He, Li+, Be2+,…, biểu thức liên
hệ giữa năng lượng liên kết của electron với hạt nhân được biểu diễn dưới dạng công thức sau:
(Z  )2
En   13.6  (eV)
n2
Với: n là số lượng tử chính; σ = 0.3; Z là số đơn vị điện tích hạt nhân.
a) Xác định năng lượng liên kết của các electron với hạt nhân trong ion Be2+ theo eV ở trạng thái
cơ bản.
b) Xác định năng lượng (theo eV) để tách hoàn toàn 1electron ra khỏi trường lực của hạt nhân
Be2+ ở trạng thái cơ bản.
1.2. Nhiên liệu hạt nhân là vật liệu được sử dụng trong các nhà máy điện hạt nhân để sản xuất nhiệt
năng cho tuabin. Hầu hết nhiên liệu hạt nhân chứa các nguyên tố lý tưởng cho phản ứng phân hạch có thể trải
qua quá trình phân hạch hạt nhân và duy trì chuỗi tái hoạt động. Ba đồng vị phân hạch có liên quan nhất là
233
U, 235U và 239Pu. Đồng vị Plutonium-239 được sản xuất trong các nhà máy điện hạt nhân vận hành bằng
hỗn hợp giàu đồng vị 238U, bằng cách bắt một netron.

a) Xác định đồng vị A và B


Một quy trình làm giàu là phức tạp đối với nhiên liệu hạt nhân dựa trên 235U. Mặt khác, 233U có thể được sản
xuất trong lò phản ứng bằng cách thu neutron từ nguyên tố ổn định xuất hiện trong tự nhiên. Sản phẩm theo
con đường phản ứng tương tự như đối với sản xuất 239Pu.
b) Cho biết nguyên tố nào được thêm vào lò phản ứng hạt nhân để tạo ra 233U. Viết các sản phẩm
của quá trình tổng hợp ra 233U.
Đồng vị 40K (0,012% trong tự nhiên) với một số lẻ proton và neutron trong hạt nhân, dẫn đến tính chất phân
rã đặc biệt. 40K phân rã theo 𝛽− (𝜆𝑏 = 4,962×10−10 (năm-1)) tạo thành 40Ca (năng lượng giải phóng 1,32 MeV)
và hấp thụ electron thành 40Ar (trạng thái cơ bản, năng lượng giải phóng 1,51MeV). Ngoài ra, quá trình hấp
thụ electron còn tạo ra 40Ar ở trạng thái kích thích (năng lượng giải phóng 0,05MeV). Bằng phát xạ 𝛾, Ar từ
trạng thái kích thích chuyển về trạng thái cơ bản. Hằng số phân rã đối với toàn bộ quá trình hấp thụ electron
được xác định là 𝜆𝑒 = 0,581×10−10 (năm-1)
c) Vẽ giản đồ năng lượng phân ra của 40K (Trục tung là mức năng lượng, trục hoành là điện tích
hạt nhân và mức không là năng lượng của Ar ở trạng thái cơ bản).
Thành phần đồng vị của Ar trong đá không phải là hằng số mà là hàm số của hàm lượng K, tuổi và lịch sử của
chúng. Với điều kiện mẫu đá bắt đầu hình thành không chứa 40Ar và từ lúc hình thành đến nay mẫu là hệ khép
kín đối với K và Ar , tỷ lệ K-40/Ar-40 có thể cho biết tuổi của đá.
Phân tích 1kg đá núi lửa cho thấy tổng hàm lượng Potassium là 0,14kg. Lượng 40Ar được xác định là 7,638.10−4
mg.
d) Xác định tuổi của mẫu vật này.
Câu Ý Nội dung Điểm
1.1 a Ta có: Be2+ 1s2 => n = 1. 0,25
(4  0,3) 2
EBe2  2[13.6  (eV )]   372,368 eV
12
1e
b Ta có: Be2  I3
Be3 0,5
42
Ta có: EBe3   13.6  (eV )   217, 6 eV
12
I 3  EBe3  EBe2   217, 6 eV  (372,368 eV )  154, 768 eV
1.2 a 0,25

b 0,5

c 0,5

d 0,5
Bài 2. (2,5 điểm)
2.1. Selenium là nguyên tố cần thiết cho con người. Tính chất hóa học của nó giống với Sulfur và
Tellerium, với một vài điểm tương đồng với Arsenic. Hợp chất A với thành phần nguyên tố CFNOSe.
a) Vẽ công thức Lewis của A với các gợi ý sau về cấu tạo của A: Se liên kết với ba trong bốn
nguyên tử còn lại; O và N ở trạng thái oxi hóa thấp nhất; có hai liên kết bội; không có điện tích hình thức.
b) Cho biết góc liên kết nhỏ nhất trong phân tử A.
Kể từ khi phát hiện ban đầu về Xe[PtF6] bởi N.Bartlettin1962, nhiều hợp chất khí của khí hiếm đã
được biết đến. Do đó, Xe hình thành liên kết không chỉ với các nguyên tố có độ âm điện lớn nhất là F và O
mà còn với các nguyên tố có độ âm điện nhỏ hơn như C.
c) Vẽ cấu trúc hình học cho các phân tử sau: XeF2; XeF4; XeOF2; XeO3; XeO4; [XeF2(C6F5)]+
2.2. Tinh thể BaTiO3 được tạo từ các ion Ba2+, Ti4+ và O2-. Các ion Ba2+ và Ti4+ tạo thành mạng
lưới lập phương tâm khối, trong đó ion Ba2+ chiếm vị trí các đỉnh và ion Ti4+ chiếm vị trí tâm hình lập phương.
Các ion O2- phân bố trên tất cả các mặt của hình lập phương.
a) Biểu diễn cấu trúc của một ô mạng cơ sở và cho biết phối trí của các ion trong mạng tinh thể.
b) Khối lượng riêng của BaTiO3 là 6,02 g/cm3 và bán kính của O2- là 1,26 Å. Xác định bán kính
của các ion còn lại trong mạng tinh thể.
Cho biết: M (BaTiO3) = 233 g/mol.
Câu Ý Nội dung Điểm
2.1 a 0,5

b Góc C-Se-F nhỏ nhất do liên kết Se=O là liên kết đôi làm tăng lực đẩy các cặp e liên 0,5
kết, dẫn đến tăng góc liên kết các góc chứa Se=O

Thực nghiệm:
c 0,5

Đường thẳng Vuông phẳng Chữ T

Chóp đáy vuông Chóp tam giác Chữ T


2.2 a Cấu trúc tinh thể BaTiO3 0,5

Số phối trí của Ti4+ = 6


Số phối trí của Ba2+ = 12
Số phối trí của O2- = 6.
b Khối lượng riêng của tinh thể là 6,02 g/cm3 và bán kính của O2- là 1,40Ao. 0,5
233,2 g
d  6,02 g.cm 3  a  4,01.108 cm  4,01 Ao
6,022.10 a23 3

Ta có: 2r(Ti4+) + 2r(O2-) = a  r(Ti4+) = 0,745 Ao


Mặt khác ta có, 2r(Ba2+) + 2r(O2-) = a 2 = 5,67Ao  r(Ba2+) = 1,575 Ao

Bài 3. (2,5 điểm)


3.1. Tính năng lượng mạng lưới tinh thể BaCl2 từ 2 tổ hợp dữ kiện sau:
(1) - Enthalpy tạo thành của BaCl2 tinh thể: - 859,41 kJ/mol
- Enthalpy phân li của Cl2: 238,26 kJ/mol
- Enthalpy thăng hoa của Ba: 192,28 kJ/mol
- Năng lượng ion hoá thứ nhất của Ba: 500,76 kJ/mol
- Năng lượng ion hoá thứ hai của Ba: 961,40 kJ/mol
- Ái lực electron của Cl : - 363,66 kJ/mol

(2) - Enthalpy của quá trình hoà tan 1 mol BaCl2 vào  mol H2O là: -10,16kJ/mol.
- Nhiệt hiđrat hoá ion Ba2+ : - 1344 kJ/mol
- Nhiệt hydrate hoá ion Cl- : - 363 kJ/mol
Trong các kết quả thu được, kết quả nào đáng tin cậy hơn.
3.2.
a) Với enthalpy và entropy không đổi, xây dựng biểu thức sự phụ thuộc tuyến tính của hằng số
cần bằng phản ứng (K) là một hàm của nhiệt độ (T)
Cho phản ứng: CH4(g)⇌ C(s)+2H2(g); H = +74,9kJ. Ở 5000C hằng số cân bằng của phản ứng Kp=0,41
b) Tính Kp ở 8500C (Giả thiết rằng H không đổi trong khoảng nhiệt độ trên).
c) Trong một bình chân không dung tích 50 lít giữ ở 8500C, người ta cho vào 1 mol CH4. Xác
định độ phân ly  của CH4 cũng như áp suất của hỗn hợp khí ở thời điểm cân bằng. (Giả thiết các khí là lý
tưởng).

Câu Ý Nội dung Điểm


3.1 o 0,5
HS(BaCl , tt)
2
Ba(r) + Cl2(k) BaCl2 (tt)
Hth(Ba) Hpl(Cl2) Uml
I1(Ba) + I2(Ba) 2+
Ba(k) + 2Cl (k) Ba + 2Cl-
2. ACl
o
Uml = H - Hth (Ba) - Hpl(Cl ) - I1(Ba) - I2(Ba) - 2ACl
S(BaCl2, tt) 2

= - 859,41 - 192,28 - 238,26 - 500,76 - 961,40 + 2 .363,66


0,5
= - 2024,79 (kJ/mol)
 Hht(BaCl2) -
2) BaCl2 (tt) Ba2+
(aq) + 2Cl(aq)

- Uml O
+ H 2 H
1 H2
0,25
Ba2+ + 2Cl-
Uml =  H1 + H2 - Hht(BaCl2)
= -1344 - 2.363 + 10,16 = -2059,84 (kJ/mol)

Kết quả 1) đáng tin cậy hơn, kết quả tính theo mô hình 2) chỉ là gần đúng do mô hình
này không mô tả hết các quá trình diễn ra trong dung dịch, các ion nhất là cation ít nhiều
còn có tương tác lẫn nhau hoặc tương tác với H2O.
3.2 a 0,25

b KP (1123K) = 15,5 0,25


c CH4(k)⇌ C(r)+2H2 (k) KP (1123K) = 15,5 0,5
1
 2
1-  n tổng = 1+
KP (1123K) = 15,5= PH2 /PCH4 = 4 /(1-2).Phệ ; Phệ= nhệRT/V=(1+)0,082.1123/50
2 2

= 0,74 và áp suất toàn phần Phệ=3,2 atm


Bài 4. (2,5 điểm)
Để có thể phân hủy các phân tử H2O2 với một tốc độ phản ứng đáng kể, người ta cần các nhiệt độ
cao và một chất xúc tác, ví dụ như các ion iodua trong dung dịch trung tính.
Bảng dưới đây ghi lại các số liệu rhu được qua thực nghiệm về tốc độ thoát oxyen từ một dung
dịch H2O2. Để đạt mục đích đó người ta trộn lẫn dung dịch H2O2 3% (30g H2O2/1L dung dịch), dung dịch KI
(C = 0,1mol.L-1) với nước.
Thí nghiệm mL dd H2O2 mL dd KI mL H2O v(O2)* mL/phút
1 50 100 150 8,8
2 100 100 100 17
3 200 100 0 35
4 100 50 150 8,5
5 100 200 0 33
*: Tại 298K và 1,013.105Pa
a) Bạn hãy xác định bậc phản ứng đối với H2O2 và đối với chất xúc tác.
b) Bạn hãy viết phương trình tỷ lượng đối với phản ứng. Và hãy viết biểu thức tốc độ phản ứng.
c) Bạn hãy tính nồng độ H2O2 (mol.L-1) khi bắt đầu thí nghiệm và sau 4 phút đối với thí ngiệm 4.
d) Cơ chế được xem như là một chuỗi hai phản ứng.
(1) H2O2 + I- → H2O + IO- (2) IO- + H2O2 → O2 + I- + H2O
Hãy cho biết hai phản ứng này xảy ra với tốc độ như nhau hay khác nhau. Một hay cả hai phản
ứng nói trên quyết định tốc độ phản ứng giải phóng oxy.
e) Bạn hãy giả thiết rằng phản ứng tuân theo định luật Arrhenius về mối liên hệ giữa năng lượng
hoạt hóa EA và hằng số tốc độ k
Bạn hãy tính các tốc độ phản ứng tương đối khi có mặt các chất xúc tác khác nhau so với chất xúc tác
đầu ở 22oC
Số TT Chất xúc tác EA(kJ.mol-1) Tốc độ tương đối
1 Bề mặt bình phản ứng 73,3
2 I- 56,2
2+ 3+
3 Fe /Fe 42,4
4 Xúc tác enzym 1,74
Giữa thế kỷ trước người ta đã tiên hành những phép đo động học rất chính xác về phản ứng giữa
H2O2 với HI trong khoảng nhiệt độ từ 0 – 50oC. Qua đó người ta thu được các mối liên hệ sau đây:
Nếu tốc độ tương đối tại 0,0oC bằng 1,00 thì nó sẽ là 8,27 tại 25oC và 49,3 tại 50oC.
Với những số liệu đó người ta đã có thể tính được điểm không tuyệt đối tức là nhiệt độ mà ở đó
không thể xảy ra một phản ứng nào nữa. Bạn không cần thiết phải làm việc đó.
Bạn hãy dùng phương trình Arrhenius để cùng với các số liệu đã cho để tính ra điểm không tuyệt
đối

Câu Ý Nội dung Điểm


a Theo các thí nghiệm 1,2 và 3 thì khi tăng gấp đôi nồng độ của H2O2 và giữ nguyên
nồng độ của I- thì tốc độ phản ứng tăng gấp đôi, điều đó có nghĩa là phản ứng tỉ lệ
thuận với nồng độ H2O2. Nói một cách khác phản ứng là bậc 1 đối với H2O2.
Từ các thí nghiệm 2, 4, 5 ta nhận thấy tốc độ phản ứng tỉ lệ thuận với [I-].
Điều đó có nghĩa phản ứng cũng là bậc 1 đối với I-.
b 2H2O2 → 2H2O + O2; v = kC(H2O2).C(I-)
c Nồng độ ban đầu:Khi pha loãng lên 3 lần thì Co=1%, nghĩa là 10g H2O2/L;
10 g / L =0,294mol/L.
Co 
34 g / mol
Phản ứng diễn ra chậm tới mức mà trong thời gian ngắn có thể bỏ qua sự giảm nồng
độ.
Sau 4 phút sẽ hình thành 4.8,5mL = 34mL O2; Khi đó; n(O2) = 1,390.10-3 mol
Lúc ban đầu có no = 3/34 mol H2O2; sau 4 phút chỉ còn: n4 = no – 2n(O2) = 0,0854
mol
Với C4 = n4/0,3L ta có C4 = 0,285mol.L-1
Nếu tính ra nồng độ sau 1 phút và coi nồng độ đó là Co đối với phút thứ hai v.v… thì
sau 4 phút người ta cũng đi đến cùng kết qủa C4.
d Đối với trường hợp bước thứ nhất quyết định tốc độ thì hằng số của bước thứ nhất k1
nhỏ hơn hằng số tốc độ của bước thứ hai k2. Tốc độ phản ứng tổng hợp bằng tốc độ
phản ứng của bước thứ nhất và như vậy là v = kC(H2O2).C(I-). Phản ứng (1)
chậm và quyết định tốc độ.; Phản ứng (2) nhanh hơn
(Nếu bước thứ hai quyết định tốc độ thì k2 < k1. Như vậy bắt buộc bước
thứ nhất phải là thuận nghịch, điều đó cũng có nghĩa là k2 < k1. Tốc độ phản ứng tổng
hợp lại được quyết định bởi tốc độ của bước chậm nhất v = kC(H2O2).C(IO-). Nhưng
C(IO-) không được biết. Do phản ứng cân bằng trước nó cho nên C(IO-) có thể biểu
thị qua nồng độ của H2O2 và I-: C(IO-) = k1/k-1.C(H2O2).C(I-)
Khác với quy luật đã quan sát được ta có:v = k1k2 /k-1.C(H2O2)2.C(I-)
e Vì rằng trong tất cả các thí nghiệm, nồng độ của các chất tham gia bằng nhau cho nên
quan hệ giữa các tốc độ phản ứng giống như quan hệ giữa các hằng số tốc độ tương
ứng. Khi đó v2/v1 = k2/k1. Nếu đặt v1 bằng tốc độ tương đối là 1 thì v2 = k2/k1.;
k1 = Ae-73300(8,314.295)
k2 = Ae-56200(8,314.295) v2(tđ) = k2/k1 = 1,07.103.
k3 = Ae-42400(8,314.295) v3(tđ) = k3/k1 = 2,96.105.
k4 = Ae -1750(8,314.295)
v4(tđ) = k4/k1 = 4,69.1012.
* Gọi x là nhiệt độ của 0oC tính bằn Kelvin thì khi đó –x là nhiệt độ của điểm không
tuyệt đối tính bằng oC.
vo = ko.C(H2O2).C(I-) k25 = 8,27k0; k50 = 49,3k0 (1)
v25 = k25.C(H2O2).C(I-) ; v50 = k50. C(H2O2).C(I-)
Theo Arrhenius thì k0 = Ae-Ea/RT hoặc đơn giản hơn là: k0 = Ae-B/T.
Theo (1) ta có: Ae-B/(x+25) = 8,27Ae-B/x ; Ae-B/(x+50) = 49,3Ae-
B/x

Logarit hóa hai biểu thức trên và giải hệ phương trình ta thu được gía trị x =
272,7 và như vậy giá trị của điểm không tuyệt đối là –272,7oC.

Bài 5.
5.1. Dung dịch X chứa H3PO4 và H2SO4 0,010M có pHX = 1,75.
a) Tính nồng độ mol/l của H3PO4 trong dung dịch X.
b) Thêm một lượng HCOOH vào dung dịch X, thu được dung dịch Y. Xác định nồng độ HCOOH có
trong dung dịch Y sao cho độ điện li của H3PO4 giảm 20% so với trước khi cho vào.
(Coi thể tích dung dịch không đổi khi thêm HCOOH).
5.2. Dung dịch bão hòa H2S có nồng độ 0, 100 M. Hằng số acid của H2S: K1 = 1,0.10-7 và K2 = 1,3.10-
13
.
a) Tính nồng độ ion sunfua trong dung dịch H2S 0, 100 M khi điều chỉnh pH = 2,0.
b) Một dung dịch A chứa các cation Mn2+, Co2+, và Ag + với nồng độ ban đầu của mỗi ion
đều bằng 0, 010 M. Hoà tan H2S vào A đến bão hoà và điều chỉnh pH = 2, 0 thì ion nào tạo
kết tủa.
Cho: TMnS = 2,5 x 10-10 ; TCoS = 4,0 x 10 – 21 ; TAg2S = 6,3 x 10-50

Câu Ý Nội dung Điểm


5.1 a Tính nồng độ mol/l của H3PO4 trong dung dịch X.
2 101,99
[SO4 ]  0, 01 1,75 1,99
 3, 652.103 M 0,5
10  10
 102,15
[H 2 PO ]  c 1,75  0, 285c (M)
 102,15
4
10
Áp dụng định luật bảo toàn điện tích, ta có: 0,25
[H+] = [H2PO4-] + 2[SO42-] + [HSO4-]
10-1,75 = 0,285c + 2.3,652.10-3 + (0,01 - 3,652.10-3)
c = 1,45.10-2 (M)
b Độ điện ly của H3PO4 trước khi có HCOOH là
[H PO ] 102,15
1 = 2 4  1,75  0, 285 = 28,5%
c 10  102,15
Độ điện ly sau khi thêm HCOOH là 28,5% x 80% = 22,8% 0,5
Nồng độ [H+] sau khi thêm HCOOH là:
[H PO ] 1  0, 228
[H  ] = K a1 3 4  102,15  2,397.102 (M)
[H 2 PO4 ] 0, 228
Gọi a là nồng độ HCOOH ban đầu trong dung dịch A:
103,75
[HCOO ]  a  7,364.103 a (M)
2,397.102  103,75
101,99
[SO24 ]  0, 01  2,992.103 M
2,397.102  101,99
Áp dụng định luật bảo toàn điện tích, ta có:
[H+] = [H2PO4-] + 2[SO42-] + [HSO4-] + [HCOO-] 0,25
-2 -2 -3 -3 -
2,397.10 = 22,8%.1,45.10 + 2.2,992.10 + (0,01 - 2,992.10 ) + 7,364.10
3
a
a = 1,042M
5.2 a Tính nồng độ ion S2–trong dung dịch HtS 0,100 M; pH = 2,0. 0,5
CH2S = [H2S] = 0,1 M H2S (k) ⇌H2S (aq)
[H2S] = 10-1 H2S (aq) ⇌H+ + HS – K1 = 1,0 x 10-7
-2 + 2-
[H+] = 10 HS ⇌ H + S K2 = 1,3 x 10-13
2
 H    S 2  
H2S (aq) ⇌2H+ + S2- K= = Kl.
 H2 S 
K2
[S2- ] = 1,3 x 10-20 x  H 2 S  = 1,3 x 10-20 x 10 1 = 1,3 x 10-17 (M)
 10 
2
 H  
2
2

b [Mn2+] [S2- ] = 10-2 x 1,3 x 10-17 = 1,3 x 10-19 < TMnS = 2,5 x 10-10 không có kết 0,5
tủa
[Co2+] [ S2- ] = 10-2 x 1,3 x 10-17 = 1,3 x 10-19 > TCoS = 4,0 x 10-21 tạo kết tủa CoS
[Ag+]2[S2- ] = (10-2)2x 1,3 x 10-17 = 1,3 x 10–21 > TAg2S = 6,3 x 10-50 tạo kết tủa Ag2S

Bài 6. (2,5 điểm)


Để loại trừ các ion NO3- trong nước (các ion NO3- có mặt trong nước xuất phát từ phân bón) có
thể khử nó thành NO2- bằng cách cho đi qua lưới có chứa bột Cd.
a) Viết nửa phản ứng của hai cặp NO3-/HNO2 và HNO2/NO trong môi trường acid. Chứng minh
rằng HNO2 bị phân hủy trong môi trường pH = 0 đến 6.
b) Ở pH = 7, nồng độ NO3- là 10-2M. Viết phản ứng giữa Cd và NO3-. Hỏi NO3- có bị khử hoàn
toàn ở 25 C trong điều kiện này không? Tính nồng độ NO3- còn lại trong nước khi cân bằng.
o

c) Tính thế khử (thế oxy hóa - khử) chuẩn của cặp NO3-/NO2- ở pH = 14 và 25oC
Cho biết các số liệu sau ở 25oC:
Eo(NO3-/HNO2) = 0,94V; Eo(HNO2/NO) = 0,98V;
Eo(Cd2+/Cd) = -0,40V; Ka(HNO2) = 5.10-4; Ks(Cd(OH)2) = 1,2.10-14.
Câu Ý Nội dung Điểm
6 a NO3- + 3H+ + 2e  HNO2 + H2O; Eo = 0,94V 1
HNO2 + H + e  NO + H2O;
+ o
E = 0,98V
Ở pH = 0 thì Eo(HNO2/NO) > Eo(NO3-/HNO2) nên HNO2 bị phân hủy theo phản
ứng:
3HNO2  NO3- + 2NO + H+ + H2O
Ở pH = 6 thì: Eo(NO3-/HNO2) = 0,94 + 0,059/2(lg10-6)
Eo(HNO2/NO) = 0,98 + 0,059lg10-6
E (HNO2/NO) vẫn lớn hơn Eo(NO3-/HNO2) nên HNO2 vẫn không bền
o

b Cd + NO3- + H2O ⇌ Cd2++ NO2- + 2OH- 1


Giả thiết phản ứng là hoàn toàn thì [Cd2+] = [NO3-]bđ = 10-2M
Ở pH = 7 thì [Cd2+] = Ks/[OH-]2 = 1,2M. Nồng độ Cd2+ sau phản ứng nhỏ hơn
nhiều so với 1,2M nên không có kết tủa Cd(OH)2.
Để tính [NO3-] khi cân bằng cân tính hằng số cân bằng K của phản ứng trên:
Cd + NO3- + H2O + 3H+  K
Cd2+ + NO2- + 2OH- + 3H+

K1 K1

Cd2+ + HNO2 + 2H2O  Cd2+ + H+ + NO2- + 2H2O


K2

K = K1.K2.K3.
2(0,94  0,40)
lg K 1   45,42  K 1  2,65.10 45
0,059
K  2,65.10 45.5.10  4.(10 14 ) 2  1,325.1014
Hằng số K rẩt lớn nên phản ứng gần như hoàn toàn. Ở pH = 7 ta có:
Cd + NO3- + H2O ⇌ Cd2+ + NO2- + 2OH-
Nđcb: (10-2 – x) =  x = 10-2 x = 10-2 10-7
Như vậy ta có: 1,325.1014 
10 2.10 2.(10 7 ) 2

 
   NO3  7,55.10 33 M

c o
2( E NO 
/ NO 
 0,40) 0,5
lg K1  3 2
 E NO
o

/ NO 
 0,017V
0,059 3 2

Bài 7. (2,5 điểm)


Acid B (pKa = 0,77) có thể được điều chế đễ dàng trong phòng thí nghiệm bằng cách đun nóng
một trong các acid: X (pKa = –1,74), Y (pKa = –1) hoặc Z (nhị phân, pKa = 11,65) với đơn chất A - chất rắn
tại STP. Mỗi phản ứng này tạo ra một khí và dung dịch. Sau phản ứng hoàn toàn, dung dịch được làm bay hơi
cẩn thận để tạo ra sản phẩm cuối cùng. Đun nóng đến 110°C, sản phẩm cuối cùng nóng chảy và sự phân hủy
để tạo ra chất C chứa 74,69% khối lượng của nguyên tố A. Đun nóng C trong không khí ở 220°C dẫn đến mất
1,77% khối lượng C và tạo ra chất rắn D. Khi nung D trong không khí đến 300oC thì bị phân hủy hoàn toàn
không có cặn rắn.
Nung m gam muối bari khan của acid B trong không khí thì thấy giảm khối lượng và tạo ra muối E
- không thể thu được trong dung dịch nước. Khi đun nóng 6,00 g B với lượng dư sulfuric acid đặc thu được
một hợp chất F và 382 cm3 (STP) khí có tỉ khối so với không khí > 1. Hợp chất F chứa 66,46% (theo khối
lượng) nguyên tố A. Khi sau khi tách và làm khô F được tráng chậm bằng nước lạnh trên phễu lọc thuỷ tinh,
nó biến thành chất G có thành phần định tính giống chất D nhưng với lượng oxy ít hơn 1,190 lần (theo khối
lượng).
a) Xác định các chất A, B,C, D, X, Y, và Z.
b) Viết phương trình hóa học của 3 phương pháp điều chế acid B.
c) Viết muối E, F và viết phương trình điều chế nó.
d) Xác định chất G và viết các phản ứng tương ứng.
e) Trong nước, ở điều kiện thường, acid B phản ứng với acid H để tạo các acid mới. Viết phương
trình hóa học.

Câu Ý Nội dung Điểm


7 a A=I2; B=HIO3; C=3I2O5.H2O (hoặc I2O5.HIO3); D=I2O5 0,75
X=HNO3; Y=HClO3; Z=H2O2
b 0,5

c 0,5

E là Ba5(IO6)2
d G là IO2 0,5

e 0,25

H là H2SO3

Bài 8. (2,5 điểm)


1. Tinh dầu bạc hà có chứa (-)-menthol. Gọi tên (-)-menthol theo danh pháp IUPAC. Biểu diễn hai cấu dạng
ghế của (-)-menthol. Cho biết trong hai cấu dạng trên, cấu dạng nào bền hơn?

2. So sánh giá trị moment lưỡng cực của hai hợp chất sau và giải thích:

3. So sánh các tính chất của các cặp chất sau và giải thích ngắn gọn:
a) So sánh nhiệt độ sôi của propanal và acetone.
b) So sánh độ tan trong nước của cyclohexanol và hexan-1-ol.
4. So sánh độ bền của hai phân tử phosphine oxide (B1) và amine oxide (B2) sau, giải thích ngắn
gọn:

5. So sánh tính acid của các nguyên tử hydrogen được đánh dấu (nguyên tử hydrogen được in đậm)
trong phân tử sau đây và giải thích:

Câu Ý Nội dung Điểm


1 (1R, 2S, 5R)-2-isopropyl-5-methyl cyclohexanol. 0,5
Hai cấu dạng ghế:
Cấu dạng (1) kém bền hơn cấu dạng (2) do các nhóm thế của cấu dạng hai đều ở liên
kết e trong khi cấu dạng (1) các nhóm thế ở dạng liên kết a.
2 A1 có  = 3,90D; A2 có  = 3,39D nên giá trị moment lưỡng cực của A1 > A2.
Hiệu ứng không gian loại II của hai nhóm methyl làm cho nhóm -NO2 trong hợp chất
A2 không liên hợp được với vòng benzene. Điều đó dẫn đến A2 chỉ có hiệu ứng -I
của nhóm -NO2; +I, +H của các nhóm methyl (ở vị trí đối xứng nên bị triệt tiêu).
Trong khi đó A1 có hiệu ứng -I và -C của nhóm -NO2 làm moment lưỡng cực của
phân tử lớn hơn.
3 a. Nhiệt độ sôi của propanal thấp hơn so với acetone. 0,25
Acetone có 2 nhóm methyl đẩy electron (+I, +H) trong khi propanal của có một nhóm
đẩy electron nên moment lưỡng cực của acetone lớn hơn, dẫn đến nhiệt độ sôi của
acetone cao hơn.

b. Độ tan trong nước của cyclohexanol cao hơn so với hexan-1-ol.


Nhóm thế cyclohexyl có hình dạng gọn hơn so với nhóm hexan-1-yl nên nhóm -OH
trong cyclohexanol ít bị cản trở không gian khi hình thành liên kết hydrogen hơn. 0,25
Bên cạnh đó, dạng mạch hở, gốc hydrocarbon khi quay cấu dạng cũng cản trở sự hình
thành liên kết hydrogen, gốc hydrocarbon mạch vòng thì sự quay cấu dạng diễn ra
khó khăn hơn.
4 Độ bền của B1 cao hơn so với B2 do ở B1 có liên kết pi được hình thành giữa cặp 0,5
electron không liên kết của nguyên tử oxi và orbital 3d trống của nguyên tử
phosphorus. Trong khi đó nguyên tử nitrogen trong phân tử B2 không có orbital 3d
trống như vậy nên không hình thành được liên kết pi. Liên kết pi giúp phân tử bền
vững hơn (tăng số liên kết và giảm điện tích hình thức ở mỗi nguyên tử).
5 Tính acid của nguyên tử hydrogen: (6) > (1) > (4) > (5) > (3) > (2). 0,5
H (6) là hydrogen của nhóm -SO3H có tính acid mạnh do được hai nhóm S=O gây
hiệu ứng -C, -I giải tỏa điện tích âm.
H (1) là hydrogen của nhóm -CO2H có tính acid mạnh thứ hai do có một nhóm C=O
gây hiệu ứng -C, -I (yếu hơn hai nhóm S=O) giải tỏa điện tích âm.
H (4) là hydrogen của nhóm -OH phenol có tính acid mạnh thứ ba do có nhóm phenyl
gây hiệu ứng -C (yếu hơn nhóm C=O) giải tỏa điện tích âm.
H (5) là hydrogen α của nhóm carbonyl có tính acid mạnh thứ ba do có C=O gây hiệu
ứng -C giải tỏa điện tích âm. H (1) mạnh hơn H (5) do H (1) là nhóm -OH còn H(5) là
nhóm C-H, mức độ phân cực H(5) kém hơn nhiều (nguyên tử carbon độ âm điện nhỏ
hơn oxygen).
H (3) là hydrogen α của nhóm phenyl gây hiệu ứng -C, giải tỏa điện tích âm.
H (2) là hydrogen của gốc alkyl nên không được giải tỏa điện tích âm.
ĐỀ SỐ 7
Câu 1 (2,5 điểm): Cấu tạo nguyên tử. Phản ứng hạt nhân. Định luật tuần hoàn
Mô hình Bohr được sử dụng để tính năng lượng cho các hệ một hạt nhân và một electron:
Z2
En   RH 2
n
Với RH là hằng số Rydberg, RH = 2,179.10-18 J; Z là điện tích hạt nhân và n là số lượng tử
Sử dụng mô hình Bohr này hãy:
1. Tính năng lượng electron nằm trên orbital 2p trong nguyên tử hydrogen
2. Tính năng lượng cần thiết để kích thích electron từ orbital 1s lên orbital 2p trong nguyên tử hydrogen.
3. Tính năng lượng ion hóa thứ hai của nguyên tử helium (He).
4. Tính tần số ứng với vạch  trong dải Lyman của nguyên tử hydrogen (ứng với bước chuyển khi
electron chuyển từ 2p về 1s)
5. Supernova E0102 -72 là một hành tinh cách trái đất khoảng hai trăm nghì năm ánh sáng, người ta tin
rằng hành tinh này có lượng oxygen gấp hàng tỉ lần trên trái đất. Nhiệt độ tại đó rất cao, cỡ hàng triệu Kelvin,
các nguyên tử oxygen bị ion hóa thành O7+. Hãy tính tần số của bức xạ tương ứng với bước chuyển  trong
dải Lyman cho ion O7+.
6. Một nguyên tố khác tồn tại trên Supernova E0102 -72 có hàm lượng lớn hơn oxygen, tần số bức xạ tương
ứng với bước chuyển  trong dải Lyman   2, 47.1017 Hz . Hãy xác định nguyên tố này.
Câu 1 Hướng dẫn giải Điểm
1 Electron nằm trên orbital 2p, nên n=2
12 0,25
E2  2,179.1018 2  5, 45.1019 J
2
2 12 12 0,5
E  E2  E1  2,179.1018 ( 2  2 )  1, 634.1018 J
2 1
3 Năng lượng ion hóa thứ hai của Helium là năng lượng cần thiết để tách electron ra
khỏi hệ He+ ở trạng thái cơ bản
22 0,5
I 2, He   E1, He  (2,179.1018 2 )  8, 716.1018 J
1
4  1, 634.10 18
  34
 2, 47.1015 Hz 0,25
h 6, 626.10
5 Bước chuyển  của O7+ có năng lượng là
82 82
E  E2  E1  2,179.1018 ( 2  2 )  1, 046.1016 J 0,25
2 1
Tần số bức xạ của vạch 
 1, 046.1016
   1, 479.1017 Hz 0,25
h 6, 626.1034
6 Gọi số điện tích hạt nhân của nguyên tố cần tìm là Z, ta có:
Z2 Z2
6, 626.1034.2, 47.1017  2,179.1018 ( 2  2 )  Z  10 0,5
2 1
Nguyên tố cần tìm là Ne

Câu 2 (2,5 điểm): Cấu tạo phân tử. Tinh thể


1.
a) Hãy vẽ các cấu trúc Lewis của dinitrogen oxide (N2O)
b) Tính điện tích hình thức trên mỗi nguyên tử, từ đó chỉ ra cấu trúc hợp lí nhất.
c) Thực nghiệm độ dài các liên kết sau đây:
N-N N=N N≡N N-O N=O
167 pm 120 pm 110 pm 147 pm 115 pm
Độ dài liên kết N – N trong N2O là 112 pm và độ dài liên kết N – O là 119 pm. Kết quả thu được ở câu
(b) có phù hợp với số liệu thực nghiệm không?
2. Muối fluoride (F-) của kim loại R có cấu trúc lập phương với hằng số mạng a = 0,62 nm, trong đó các
ion kim loại (Rn+) nằm tại các vị trí nút mạng của hình lập phương tâm diện, còn các ion fluoride (F‒) chiếm
tất cả các hốc tứ diện. Khối lượng riêng của muối fluoride là 4,89 g/cm3.
a) Vẽ cấu trúc tế bào đơn vị (ô mạng cơ sở) của mạng tinh thể fluoride?
b) Xác định công thức phân tử tổng quát của muối?
c) Xác định kim loại R? Cho NA = 6,023.1023; MF = 19 g/mol.

Câu 2 Hướng dẫn giải Điểm


1 a) N2O có 3 cấu trúc cộng hưởng chính:
0,5
Câu 2 Hướng dẫn giải Điểm
b) Điện tích hình thức trên mỗi cấu trúc
0,25

- Cấu trúc (a) và (b) có điện tích hình thức “tối đa” là 2, còn cấu trúc (c) có điện tích 0,25
hình thức “tối đa” là 4 nên cấu trúc (a) và (b) hợp lí hơn cấu trúc (c). Tuy nhiên, trong
cấu trúc (b) điện tích hình thức -1 trên O là nguyên tử có độ âm điện lớn hơn N nên
cấu trúc (b) là cấu trúc hợp lí nhất.
c) Trong N2O độ dài liên kết N – N là 112 pm nằm nữa liên kết đôi (120 pm) và liên
kết ba (110 pm). Độ dài liên kết N – O là 119 pm nằm giữa liên kết đơn (147 pm) và 0,25
liên kết đôi (115 pm), không có bằng chứng về liên kết ba N – O. Vậy các kết luận ở
b) cấu trúc cộng hưởng (a) và (b) là hợp lí nhất là phù hợp với thực nghiệm.

2 Ô mạng cơ sở:

0,25

Trong một ô mạng:


1 1
- Số ion Rn+: 8   6   4
8 2
- Số ion F‒: 8  1  8 0,25
-Để đảm bảo về mặt trung hòa điện tích thì: 4×n = 8×1  n = 2
 ion kim loại là R2+
Vậy trong 1 ô mạng cơ sở có 4 phân tử oxit có dạng RF2. 0,25
Khối lượng riêng fluoride tính theo công thức:
M RF2
4
6, 023.10 23
D=
a3
D×a 3  6, 023.1023 4,89  (0, 620.107 )  6, 023.10 23 0,25
 M RF2    175, 48
4 4
 M R  175, 48  19  2  137, 48 (g/mol)
Vậy kim loại R là Ba
0,25
Muối fluoride là BaF2.

Câu 3 (2,5 điểm): Nhiệt hóa học. Cân bằng trong pha khí
1. Khí methane (CH4) thường được sử dụng làm khí đốt để cung cấp nhiệt cho mục đích dân dụng và
một số mục đích công nghiệp. Tính nhiệt độ lớn nhất (theo độ Kelvin) mà ngọn lửa có thể đạt được khi đốt
cháy methane bằng oxy không khí ở 1,0 atm. Biết methane và không khí có nhiệt độ ban đầu đều là 298 K,
không khí được lấy để lượng oxygen phản ứng vừa đủ với methane.
2. Thạch cao nung (CaSO4.1/2H2O) được sản xuất bằng cách đề hiđrat hóa một phần thạch cao sống
(CaSO4.2H2O) trong lò nung ở 400K:
1 3
CaSO4 .2 H 2O( r )  CaSO4 . H 2O( r )  H 2O( k ) (*)
2 2
a) Tính  r H 0 (theo kJ.mol-1) của phản ứng (*) tại 298K và 400K.
b) Năng lượng cần thiết để thực hiện phản ứng (*) được cung cấp bởi quá trình đốt cháy khí methane
bằng oxygen không khí. Tính khối lượng thạch cao nung (theo kg) thu được khi sử dụng 1,00 kg khí methane
làm nhiên liệu cho quá trình nung thạch cao.
Biết không khí được lấy vừa đủ cho phản ứng đốt cháy, thạch cao sống và hỗn hợp khí được đưa vào lò nung
ở 298K, thạch cao nung và khí đi ra khỏi lò nung có nhiệt độ 400K, hiệu suất của lò nung là 80,0%.
Cho biết
+ Không khí chỉ gồm N2 và O2 có tỉ lệ số mol tương ứng là 4:1.
+ Sản phẩm đốt cháy methane là CO2 và hơi nước.
+ Các giá trị nhiệt dung không phụ thuộc vào nhiệt độ trong khoảng nhiệt độ nghiên cứu.
+ Nhiệt hình thành chuẩn (  f H 298
0
) và nhiệt dung đẳng áp ( C p0 ) của các chất ở 298 K được cho trong
bảng sau:
H2O(k) O2(k) N2(k) CO2(k) CH4(k) CaSO4.2H2O (r) CaSO4.1/2H2O(r)

 f H 298
0

(kJ.mol-1) -241,82 0 0 -393,51 -74,81 -2021,00 -1575,00


C p0
(J.K-1.mol-1) 33,58 29,36 29,13 37,11 35,31 186,00 120,00

Câu 3 Hướng dẫn trả lời Điểm


CH4 (k) + 2O2 (k)  CO2 (k) + 2H2O (k)
Coi quá trình cháy nhanh tới mức chưa có trao đổi nhiệt với môi trường bên ngoài,
tức là quá trình cháy là đoạn nhiệt. Ta có sơ đồ
1
Ban đầu Kết thúc
Q = H =0
CH4(k): a mol CO2(k): a mol
0,5
O2 (k): 2a mol H2O (k): 2a mol

N2(k): 8a mol Kết thúc N2(k): 8a mol

T1 = 298K, P =1
Hatm
1
CO2(k): a mol T1 
=HTmax
2 , P =1 atm

H2O (k): 2a mol

N2(k): 8a mol
Theo định luật Hess: H1 + H 2 = 0
H1 =  r H 2980
 ( f  0298(CO2T)1= 2298K  f ,0298(
P =1 atm   0
H 2Ok ) f 298( CH 4 )  2 f  298( O2 )
0

= -393,51 + 2.(-241,82) – (-74,81) – 2.(0) = -802,34 (kJ.mol-1)


Ta có:
a  r H 298
0
+ a (C p0 (CO2 )  2C p0 ( H 2O ) k  8C p0 ( N 2 ) )(Tmax  298) =0

 Tmax = 298  ( r  0298 ) = 298  802,34.103


 2676, 64 K 0,5
(C p (CO2 )  2C p ( H 2O )  8C p ( N2 ) 37,11  2.33,58  8.29,13
Nhiệt độ lớn nhất mà ngọn lửa có thể đạt được là 2676,64 K
2 1 3
a) Entanpi chuẩn của phản ứng: CaSO4 .2 H 2O( r )  CaSO4 . H 2O( r )  H 2O( k ) (*)
2 2
3
 r H 298
0
  f  0298(CaSO4 .1/2 H 2O )   f  0298( H 2O )   f  0298(CaSO4 .2 H 2O )
2
= -1575,0 + 3/2.(-241,82) – (-2021,0) = 83,27 (kJ.mol-1)
* Tính  r  0400
 r  0400 =  r  0298 + C p (400  298).103
3
C p  C p (CaSO4 .1/2 H 2O )  C p ( H 2O )  C p (CaSO4 .2 H 2O ) =120,0 + 3/2.33,58 – 186,0 = -15,63
2
(J.mol-1.K-1)
 r  0400 =  r  0298 + C p (400  298).103 = 83,27 -15,63 (400 – 298) .10-3 = 81,68
0,5
(kJ.mol-1)
b)
Xét chu trình sau cho 1 mol CaSO4.1/2H2O

 
0
[CaSO4.2H2O(r)]298 [CaSO4.1/2H2O(r) + 3/2H2O(k)]400 K
1  2
[CaSO4.2H2O (r)]400 K
  1   2  C p ( CaSO .2 H O (400  298)   0400
0
4 2

= 186.10-3 (400-298) + 81,68 = 100,65 (kJ)


* Xét chu trình sau cho 1 mol CH4

[CH4 (k) + 2O2 (k) + 8N2(k)]298   [CO2(k) + 2H2O(k) + 8N2 (k)]400 K
 3  4
[CO2(k) + 2H2O(k) + 8N2 (k)]298 K
   3   4
 3 =  r H 298
0
 ( f  0298(CO2 )  2 f  0298( H 2Ok )   f  0298(CH 4 )  2 f  0298(O2 )
= -393,51 + 2.(-241,82) – (-74,81) – 2.(0) = -802,34 (kJ.mol-1)
 4 = (C p0 (CO2 )  2C p0 ( H 2O ) k  8C p0 ( N 2 ) )(400  298) = (37,11 + 2.29,36 + 8.29,13).10-
3
(400-298) = 34,41 (kJ)
   3   4 = -802,34 + 34,41 = -767,93 (kJ)
- Nhiệt tỏa ra khi đốt cháy 1,0 kg khí metan là:
m 1000
Q   .  767,93.  4,8.104 (kJ)
16 16
- Khối lượng thạch cao nung thu được khi sử dụng 1,0 kg metan làm nhiên liệu là:
Q 4,8.104
m = 145.0,8. = 145.0,8.  55,32.103 (gam) = 55,32 (kg) 1,0
 100, 65

Câu 4 (2,5 điểm): Động hóa học (không có cơ chế)


Cho phản ứng: SO2Cl2  SO2 + Cl2
Người ta tiến hành nung nóng 0,1 mol SO2Cl2 ở 600K trong bình phản ứng có dung tích 1 lít và đo áp
suất của hỗn hợp các chất trong bình thì thu được các số liệu thực nghiệm sau:
T (giờ) 0 1 2 4 8
P (atm) 4,92 5,67 6,31 7,31 8,54
1. Xác định bậc của phản ứng.
2. Tính hằng số tốc độ và thời gian bán phản ứng ở 600K.
3. Tính áp suất trong bình sau khi tiến hành phản ứng 24 giờ.
4. Nếu tiến hành phản ứng với cùng lượng SO2Cl2 trong bình trên ở 620K thì sau 2 giờ, áp suất trong
bình là 9,12. Tính hệ số nhiệt của phản ứng.
Câu 4 Hướng dẫn giải Điểm
1 P0
1
Giả sử phản ứng là bậc 1  Phương trình động học k = ln 0,25
t P
(P0 là áp suất của SO2Cl2ở thời điểm ban đầu, t là áp suất của SO2Cl2 tại thời điểm t)
SO2Cl2  SO2 + Cl2
t=0 Po 0 0
phản ứng x x x (atm)
t Po – x x x
 Phỗn hợp = Po + x ; P = Po – x = 2Po – Phh. Ta có bảng số liệu sau :
t(h) 0 1 2 4 8 0,25
Phh (atm) 4,92 5,67 6,31 7,31 8,54
P (atm) 4,92 4,17 3,53 2,53 1,30
Thế các giá trị vào phương trình động học, ta có :
1 4,92 1 4,92
k1  ln  0,1654h 1 k2  ln  0,1660h 1
1 4,17 2 3,53 0,25
1 4,92 1 4,92
k3  ln  0,1663h 1 k4  ln  0,1664h 1
4 2,53 8 1,3
Vì k1 ≈ k2 ≈ k3 ≈ k4  Phản ứng trên là bậc 1 0,25
2 k k k k
k  1 2 3 4  0,1660h 1
4
ln 0,6931 0,25
t1  2   4,1753h
2
k 0,1660
3 t = 24h 0,5
P = Po.e-kt = 4,92.e-0,166.24 = 0,093 atm = Po – x  x = 4,827 atm
Vậy áp suất trong bình: Phh = Po + x = 9,747 atm
4
Ở 620k:
nRT 0,1.0,082.620
Po    5,084atm; P  1,048atm
V 1
1 5,084
k  ln  0,7895h 1 0,5
2 1,048

  620  600    K 620  2,181


k620
Ta có : 0,25
k600 10 K 600

Câu 5 (2,5 điểm): Cân bằng acid – base và cân bằng ít tan
Lactic acid (CH3CHOHCOOH) được hình thành trong các cơ bắp trong quá trình hoạt động cường độ
cao (trao đổi chất kị khí). Lactic acid là đơn acid (kí hiệu HL) có hằng số phân li acid là KHL = 1,38.10-4.
Trong máu, lactic acid được trung hòa bằng phản ứng với ion HCO3-. Các hằng số phân li acid của H2CO3
là: Ka1= 4,47.10-7 và Ka2= 4,68.10-11.
1. Tính pH trong dung dịch HL có nồng độ 3,00.10-3 M.
2. Tính giá trị của hằng số cân bằng của phản ứng giữa lactic acid và ion HCO3-.
3. Thêm 3,00.10-3 mol lactic acid (HL) vào 1 lít dung dịch NaHCO3 0,024 M (bỏ qua sự thay đổi thể
tích khi thêm HL vào dung dịch, HL trung hòa hoàn toàn).
a) Tính giá trị pH trong dung dịch NaHCO3 trước khi HL được thêm vào.
b) Tính giá trị pH trong dung dịch sau khi thêm HL.
4. Trong máu của một người đang có pH = 7,40 và [HCO3-] = 0,022M. Khi người này hoạt động với
cường độ cao thì hình thành thêm lactic acid (HL) làm cho pH trong máu giảm xuống và có giá trị là 7. Tính
số mol lactic acid (HL) đã được hình thành trong 1 lít máu của người này.
Hướng dẫn
Câu 4 Hướng dẫn trả lời Điểm
HL H+ + L- KHL = 1,38.10-4
H2O H+ + OH- Kw = 1,0.10-14
1 Do KHL. CHL >> KW  bỏ qua sự phân li của H2O
HL H+ + L- KHL = 1,38.10-4
-3
C0 3,00.10
[ ] 3,00.10-3 – x x x
2
x
 1,38.104
3, 00.103  x
 x = 5,78.10-4 M  [H+] = x = 5,78.10-4 M  pH = 3,24 0,5
2 HL + HCO3- -
H2CO3 + L K = ?
K = KHL.Ka1-1 = 1,38.10-4 .(4,47.10-7)-1 = 308,72 = 102,49 0,5

3 a) pH của dung dịch NaHCO3 trước khi thêm HL:


HCO3- lưỡng tính 0,5
pK a1  pK a 2
pH   8,34
2
b) pH của dung dịch NaHCO3 sau khi thêm HL
HL + HCO3- H2CO3 + L-
Ban đầu 0,003 0,024
Sau - 0,021 0,003 0,003
TPGH: HCO3-, H2CO3, L-
0, 021
Hệ đệm: pH sơ bộ  pK a1  lg  6,35  0,85  7, 2  7
0, 003
Bảo toàn điện tích:
[H+] + [Na+] = [L-] + [ HCO3-] + 2[CO32-] + [OH-]  [L-] + [ HCO3-] + [OH-]
(pH  7,2 nên [CO32-]<< [ HCO3-] )
 [H+] = 6,38.10-8 M  pH  7,2 0,5

4 Máu có pH = 7,40  [H+] = 4,0.10-8 M; [HCO3-] = 0,022M


[ H  ][ HCO3 ] 4, 0.108.0, 022
K a1   [ H 2CO3 ]   0, 00197 M
[ H 2CO3 ] 4, 47.107
 [HCO3-] + [H2CO3] = 0,022 + 0,00197 = 0,02397  0,024 M (1)
Máu có pH = 7
[HCO3 ] K
 a1  4, 47 hay [HCO3-] = 4,47.[H2CO3]
[H 2CO3 ] [H ]
Từ (1)  [HCO3-] = 0,0196 M và [H2CO3] = 0,0044 M
Số mol HL thêm vào máu = n( HCO3 )  0, 022  0, 0196  2, 4.103 mol.
0,5

Câu 6 (2,5 điểm): Phản ứng oxi hóa – khử. Pin điện (không liên quan đến phức chất)
Thế E của pin: Hg(l) Hg2Br2(r) KBr 0,1M ‖ KCl 0,1M Hg2Cl2 (r) Hg(l), phụ thuộc tuyến tính theo nhiệt độ:
E = a – bT, trong đó a = 0,1318 V và b = 1,58.10-5 V.K-1. Tại nhiệt độ xác định, thế chuẩn EHg
0
2
/ Hg
 0, 799
2

V và thế điện cực calomen trong dung dịch KCl 0,1M ECl  , Hg Cl  0,3335 V .
2 2 / Hg

1. Viết các nửa phản ứng trên các điện cực và phản ứng tổng cộng ứng với hai electron trao đổi.
2. Tính hiệu ứng nhiệt chuẩn  r H 0 của phản ứng trong pin và tích số tan của Hg2Br2 tại 250C.
Câu 4 Hướng dẫn trả lời Điểm
Các nửa phản ứng trong pin
Catot: Hg2Cl2 + 2e →2Hg + 2Cl-
Anot: 2Hg + 2Br- →Hg2Br2 + 2e
1 Phản ứng trong pin: Hg2Cl2 + 2Br- →Hg2Br2 + 2Cl- 1,0
2 Thế của pin được tính:
RT [Br  ]2
EE  0
ln  2
 E 0  E 0  a  bT
nF [Cl ]
Năng lượng Gibbs chuẩn của phản ứng:
 r G 0  nFE 0  2.96485.(0,1318  1,58.10 5.298) = -24,525.10-3 J.mol-1 0,5
Từ biểu thức:
 H 0 r S 0 dE 0  r S 0
E0   r  T   b   r S 0  bnF
nF nF dT nF
  r S 0  1,58.105.2.96485  3, 05 J.K-1.mol-1
  r H 0   r G 0  T . r S 0  24,525.103  298.(3, 05)  24, 434.103 J.mol-1
Từ biểu thức E = E+ - E- → E- = E+ - E
EBr  , Hg Br / Hg  0,3335  (0,1318  1,58.105.298)  0, 2064V
2 2
0,5
0, 0592 0, 0592 K
EBr  , Hg Br / Hg  E
0

Hg 22 / Hg
lg[ Hg 22 ]  EHg
0
2  lg
2 2
2 2 / Hg
2 [ Br  ]2
0, 0592 K
 0, 2064  0, 799  lg 2  K  9,544.1023 0,5
2 0,1

Câu 7 (2,5 điểm): Halogen. Oxygen – Sulfur


Cho sơ đồ chuyển hóa các chất từ sulfur (S) như sau:

Khi thủy phân hoàn toàn các chất C, E và G, thu được các dung dịch acid và không thấy khí thoát ra.
Thêm dung dịch Ba(NO3)2 dư vào các dung dịch trên đều thu được kết tủa trắng X. Lọc kết tủa X, thêm tiếp
dung dịch AgNO3 dư vào nước lọc, lại thu được kết tủa trắng Y. Đối với chất E và G, tỉ lệ khối lượng kết tủa
 mX 
X và kết tủa Y   đều là 1,624; còn đối với chất C, tỉ lệ trên là 0,812.
 Y 
m
Cho dung dịch Ba(OH)2 dư vào dung dịch sau khi thủy phân của các chất D, H và K đều thấy có kết tủa
trắng tạo thành, không tan trong acid mạnh và có khí NH3 thoát ra. Hàm lượng phần trăm về khối lượng N và
S trong các chất sau lần lượt là: trong chất D là 29,16% và 33,33%; trong chất H là 14,43% và 32,99%; trong
chất K là 24,56% và 28,07%. Trong các chất D, H và K, mỗi phân tử chỉ chứa 1 nguyên tử sulfur.
1. Xác định công thức của các chất.
2. Viết phương trình hóa học của các phản ứng trong sơ đồ.
Hướng dẫn chấm
Câu 7 Nội dung 2,5 đ
1 Chất A là SO2, chất B là SO3. 0,5
- Xác định các chất C, G:
Khi thủy phân hoàn toàn các chất C, E và G thu được các dung dịch axit và
không thấy khí thoát ra. Thêm Ba(NO3)2 dư vào các dung dịch trên đều thu
được kết tủa trắng X → kết tủa X là BaSO4 → trong các chất này có S+6, không
có chứa C.
- Kết tủa Y là AgCl.
 mX 
Từ tỉ lệ   =1,624 → trong E, G tỉ lệ nS : nCl = 1:1
 mY 
Gọi công thức của G là SxOyClx ta có: 6x – 2y – x = 0 → 5x – 2y = 0
→ x:y = 2:5 → công thức của G là S2O5Cl2
Từ phản ứng SO3 + CCl4 → S2O5Cl2 + F → công thức của F là COCl2
 mX 
Chất C có S+6, tỉ lệ   =0,812 → trong C, tỉ lệ nS : nCl = 1:2 0,5
 mY 
→ công thức của C là SO2Cl2
- Xác định các chất D, E, H, K:
Dung dịch thủy phân của các chất D, H, K khi tác dụng với Ba(OH)2 đều thấy
có kết tủa trắng tạo thành và khí NH3 thoát ra → trong D, H, K có S+6 và có
N-3
- Hàm lượng N và S trong D là 29,16 và 33,33% → tỉ lệ nN : nS = 2: 1.
- Hàm lượng N và S trong H là 14,43 và 32,99% → tỉ lệ nN : nS = 1: 1.
- Hàm lượng N và S trong K là 24,56 và 28,07% → tỉ lệ nN : nS = 2: 1.
Gọi công thức của D là SN2OxHy:
32
MD= .100  96 ; trong D có S+6 và có N-3 → công thức của D là 0,5
33,33
SO2(NH2)2.
SO2Cl2 + 2NH3 → SO2(NH2)2 + 2HCl → L là HCl
Chất E có S+6, tỉ lệ nS : nCl = 1: 1; SO3 + HCl → E Suy ra công thức của E là
HSO3Cl.
Gọi công thức của H là SNOxHy:
32
MH = .100  97 ; trong H có S+6 và có N-3
32,99
→ Công thức của H là NH2SO3H.
Gọi công thức của K là SN2OxHy: 0,5
32
MK = .100  114 ; trong K có S+6 và có N-3
28,07
→ Công thức của K là NH2SO3NH4.
2 1. S + O2 → SO2
2. 2SO2 + O2 → 2SO3
3. SO2 + Cl2 → SO2Cl2
4. SO2Cl2 + 2NH3 → SO2(NH2)2 + 2HCl 0,5
5. 2SO3 + CCl4 → COCl2 + S2O5Cl2
6. SO3 + HCl → HSO3Cl
7. SO3 + NH3 → HSO3NH2
8. SO2 + 2NH3 → H2NSO3NH4

Câu 8 (2,5 điểm): Đại cương hữu cơ (quan hệ giữa cấu trúc và tính chất)
1. Dựa trên cấu trúc phân tử, so sánh và giải thích độ bền tương đối giữa cấu dạng xen kẽ và cấu dạng
che khuất của phân tử etan
2.
a) Vẽ cấu trúc của 1,3-diazole (imidazole, C3H4N2), anion imidazol-1-yl, cation imidazolyl, 1,3-oxazole
(oxazole, C3H3NO) và 1,3-thiazole (thiazole, C3H3NS). Cấu trúc nào thơm?
b) Sắp xếp imidazole, 1,3-oxazole and 1,3-thiazole theo chiều giảm dần nhiệt độ sôi, tonc, giải thích?
c) Sử dụng cấu trúc viết quá trình ion hóa imidazole, oxazole, and thiazole trong nước. Sắp xếp theo
chiều giảm dần tính bazo và giải thích?
Hướng dẫn chấm
1. (0,75 điểm) Phân tích cấu dạng xen kẽ và che khuất của etan:
Như trên giản đồ, cấu dạng xen kẽ của etan có sự tương tác bền hoá giữa cặp e trên liên kết C H với
orbital trống *C H. Trong khi đó, ở cấu dạng che khuất xuất hiện tương tác đẩy giữa 2 cặp e trên 2 liên
kết C H. Vì thế cấu dạng xen kẽ bền hơn cấu dạng che khuất.

2.
a) (0,75 điểm)
Cấu trúc Thơm hay không thơm

Imidazole (C3H4N2) thơm

Imidazol-1-yl anion (C3H3N2) thơm

Imidazolyl cation (C3H5N2) thơm

Oxazole (C3H3NO) thơm

Thiazole (C3H3NS) thơm


b) (0,5 điểm)
Tonc Imidazole > Thiazole > Oxazole
Imidazole lớn nhất do liên kết H liên phân tử. Thiazole lớn hơn oxazole
Giải thích
do khối lượng phân tử và độ phân cực lớn hơn.
Tos Imidazole > Thiazole > Oxazole
Imidazole lớn nhất do liên kết H liên phân tử. Thiazole lớn hơn oxazole
Giải thích
do khối lượng phân tử và độ phân cực lớn hơn.
c) (0,5 điểm)
Phản ứng ion hóa

Kb Imidazole > Thiazole > Oxazole


Giải thích Axit liên hợp của imidazole có sự giải tỏa đối xứng, tạo liên kết H mạnh hơn
với nước, do đó có lực bazo mạnh hơn oxadiazole và thiazole. O có độ âm
điện lớn hơn N và S, nên làm giảm mật độ e trên N ở oxazole, làm giảm độ
bền của axit liên hợp của oxazole nên lực bazo yếu nhất.
ĐỀ SỐ 8
Câu 1 (2,5 điểm): Cấu tạo nguyên tử. Phản ứng hạt nhân. Định luật tuần hoàn.
1.1. Mô hình hạt trong hộp thế một chiều được xây dựng để giải gần đúng bài toán năng lượng của hệ liên hợp
carbon mạch thẳng. Trong mô hình này, các electron π bất định xứ có thể di chuyển tự do trên khung carbon
của các liên kết liên hợp.
Chiều dài của hộp thế được tính gần đúng bằng công thức L = nC × 1,40 Å, trong đó nC là số lượng nguyên
tử carbon của mạch liên hợp. Nguyên lý Pauli được áp dụng khi các electron được lấp đầy các mức năng
n 2h 2
lượng. Mức năng lượng của hạt trong hộp một chiều được tính bằng biểu thức: E n =
8mL2
Áp dụng mô hình trên cho phân tử 1,3,5,7-octatetraene, hãy:
a) Vẽ sơ đồ mức năng lượng và điền các electron.
b) Tính năng lượng tổng cộng cho hệ liên hợp  nói trên theo J.
c) Xác định bước sóng (theo nm) tương ứng với bước chuyển năng lượng giữa HOMO và LUMO.
Cho biết: me = 9,10938215×10−31 kg; h = 6,6261×10–34J.s.
Câu Nội dung Điểm
1.1 a) Sơ đồ mức năng lượng:
E
0,5
b) Năng lượng cho từng mức:
h2 (6, 62.1034 ) 2
E1 = 2
 31 10 2
 4,8029.1020 J
8mL 8.9,1.10 .(8.1, 4.10 )
4h 2 4.(6, 62.1034 ) 2
E2 =   1,9211.1019 J
8mL2 8.9,1.1031.(8.1, 4.1010 ) 2
9h 2 9.(6, 62.1034 ) 2
E3 = 2
 31 10 2
 4,3226.1019 J
8mL 8.9,1.10 .(8.1, 4.10 )
16h 2 16.(6, 62.1034 ) 2
E4 = 2
 31 10 2
 7, 6846.1019 J
8mL 8.9,1.10 .(8.1, 4.10 )
25h 2 25.(6, 62.1034 ) 2
E5 = 2
 31 10 2
 1, 2007.1018 J
8mL 8.9,1.10 .(8.1, 4.10 )
0,5
Tổng năng lượng của hệ liên hợp π trên là:
ET = 2 * (E1 + E2 + E3 + E4) = 2,8817×10-18 (J)
(HS có thể áp dụng gộp công thức các mức khi tính ET, không nhất thiết phải tính
từng mức)
hc
c)    459,55.109 (m)  459,55nm
E5  E4 0,25
a) 1.2.
Câu Nội dung Điểm
1.2 a) Phương trình phản ứng phân rã: 210
84Po   206
82 Pb + 2 α
4

A 0,25
Z X là 206 82 Pb

b) Chu kỳ bán rã của Po:


t.ln 2 t.ln 2 t.ln 2 0,25
T1/2   
n     
ln  0  ln  n0  
m0

n  n0  nHe  ln  210 
 m0  VHe 
 210 22, 4  0,25
 
Thay t = 365 ngày; m0 = 2g; VHe = 0,179 l ta có T1/2 = 138,5 ngày
c) Tại thời điểm t mẫu chuẩn có chứa Pb và Po. Dựa vào đề bài ta có được:
mPb / t
 0, 4
mPb / t  mPo /t
0,25
m 0, 4 206.nPb / t n 70
 Pb / t    Pb / t 
mPo / t 0, 6 210.nPo / t nPo / t 103
T1/2  n0  T1/2  n  138,5  70  0,25
t  .ln  Po / t  .ln 1  Pb / t  .ln 1    103, 6 (ngày)
ln 2  nPo / t  ln 2  nPo / t  ln 2  103 
Câu 2 (2,5 điểm): Cấu tạo phân tử. Tinh thể.
2.1. a) Cả nitrogen và boron đều tạo hợp chất trifluoride. Năng lượng liên kết B-F trong BF3 là 646 kJ/mol và
của liên kết N-F trong NF3 chỉ là 280 kJ/mol. Hãy giải thích sự khác nhau về năng lượng liên kết trong hai
hợp chất.
b) Hãy vẽ các cấu trúc Lewis của dinitrogen oxide (N2O). Tính điện tích hình thức trên mỗi nguyên tử, từ đó
chỉ ra cấu trúc hợp lí nhất.
c) Thực nghiệm độ dài các liên kết sau đây:
N-N N=N N≡N N-O N=O
167 pm 120 pm 110 pm 147 pm 115 pm
Độ dài liên kết N – N trong N2O là 112 pm và độ dài liên kết N – O là 119 pm. Kết quả thu được ở câu b)
có phù hợp với số liệu thực nghiệm không?
Câu Nội dung Điểm
2.1 a) Năng lượng liên kết trong BF3 cao hơn NF3 nhiều do trong BF3 có thêm liên kết 
cho nhận giữa cặp electron hóa trị tự do của F và obitan p còn trống của B. 0,25

b) N2O có 3 cấu trúc cộng hưởng chính: 0,3


(Mỗi
CT
0,1)
Điện tích hình thức trên mỗi cấu trúc

0,3

- Cấu trúc (a) và (b) có điện tích hình thức “tối đa” là 2, còn cấu trúc (c) có điện tích
hình thức “tối đa” là 4 nên cấu trúc (a) và (b) hợp lí hơn cấu trúc (c). Tuy nhiên, trong
cấu trúc (b) điện tích hình thức -1 trên O là nguyên tử có độ âm điện lớn hơn N nên 0,15
cấu trúc (b) là cấu trúc hợp lí nhất.
c) Trong N2O độ dài liên kết N – N là 112 pm nằm nữa liên kết đôi (120 pm) và liên
kết ba (110 pm). Độ dài liên kết N – O là 119 pm nằm giữa liên kết đơn (147 pm) và
liên kết đôi (115 pm), không có bằng chứng về liên kết ba N – O. Vậy các kết luận ở 0,25
b) cấu trúc cộng hưởng (a) và (b) hợp lí là phù hợp với thực nghiệm.

2.2. Tính dẻo và dễ uốn cong của kim loại là những đặc tính cực kì quan trọng trong xây dựng hiện đại. Dạng
bền nhiệt động của thiếc kim loại ở 298K và áp suất thường là thiếc trắng. Loại thiếc này có các tính chất cơ
học điển hình của kim loại và vì vậy có thể sử dụng làm vật liệu xây dựng. Ở nhiệt độ thấp hơn, thiếc xám,
một loại thù hình của thiếc trắng lại bền nhiệt động hơn. Bởi vì thiếc xám giòn hơn nhiều so với thiếc trắng,
vì vậy các thành phần xây dựng bằng thiếc nếu để lâu ở nhiệt độ thấp sẽ trở nên hư hại, dễ gãy. Bởi vì sự hư
hại này tương tự như một loại bệnh, nên người ta gọi sự hư hại này là “bệnh dịch thiếc”.
a) Sử dụng bảng số liệu dưới đây, tính nhiệt độ tại đó thiếc xám cân bằng với thiếc trắng (tại áp suất 1 bar =
105 Pascal).
Chất ΔfH0 (kJ.mol-1) S0 (J.mol-1.K-1)
Thiếc xám -2,016 44,14
Thiếc trắng 0 51,18
b) Thiếc trắng có ô mạng cơ sở khá phức tạp, ở dạng bốn phương, a = b = 583,2 pm và c = 318,1 pm với 4
nguyên tử Sn trong 1 ô mạng cơ sở. Tính khối lượng riêng của thiếc trắng theo g/cm3.
c) Cho rằng thiếc xám có cấu trúc lập phương tâm mặt được gọi là cấu trúc
kim cương (hình bên):
Khảo sát một mẫu thiếc xám bằng phương pháp nhiễu xạ tia X (sử dụng
bức xạ Cu Kα,  = 154.18 pm). Góc phản xạ nhỏ nhất, gây bởi sự nhiễu xạ
từ họ các mặt phẳng (111), được quan sát thấy ở 2 = 23,74°. Tính khối lượng
riêng của thiếc xám theo g/cm3.

Câu Nội dung Điểm


2.2 a) Hai pha thiếc nằm cân bằng khi ΔG = 0 cho cân bằng Sn(trắng) 
0   Sn(xám)

0,25
Từ các giá trị ΔH và S đã cho ta tìm được T = 13,2 C
0 0 o

b) Thể tích 1 ô mạng cơ sở dạng bốn phương là:


V = a.b.c = 583,2 pm × 583,2 pm × 318,1 pm = 1,082 × 108 (pm3) = 1,082 × 10-
22
(cm3)
Theo bài ra, mỗi ô mạng cơ sở chứa 4 nguyên tử Sn trắng
n.M 4.118, 71 0,25
Khối lượng riêng của thiếc trắng: D    7, 29 (g/cm3 )
V .N A 1, 082.1022.6, 02.1023
c) Từ định luật Bragg: nλ = 2dsinθ
λ
Với góc phản xạ nhỏ nhất: n = 1  d   374,8 (pm) 0,125
2sinθ
Khoảng cách giữa các mặt (111) gần nhau nhất trong ô mạng lập phương là:
a a
d  với a là độ dài cạnh ô mạng cơ sở
h2  k 2  l 2 3 0,125
 a  d 3  374,8. 3  649,1 pm
Theo ô mạng cơ sở minh họa, có 8 nguyên tử Sn trong ô mạng cơ sở 0,25
Khối lượng riêng của thiếc xám:
n.M 8.118, 71 0,25
D  10 3 23
 5, 768 (g/cm3 )
V .N A (649,1.10 ) .6, 02.10

Câu 3 (2,5 điểm): Nhiệt hóa học. Cân bằng hóa học trong pha khí.
3.1. Nạp 0,01 mol but-1-yne vào một lò phản ứng có thể tích thay đổi được với V0 = 0,1 m3 chỉ chứa không
khí ở P = 1,0 atm và T = 298 K. Tiến hành đốt cháy hoàn toàn hydrocarbon này ở điều kiện đoạn nhiệt, đẳng
áp (là phản ứng duy nhất xảy ra trong điều kiện này). Sau khi đốt cháy hoàn toàn thì trong bình phản ứng chỉ
chứa carbon dioxide, hơi nước, nitrogen và oxygen.
a) Tính enthalpy chuẩn của phản ứng ở 298 K. Từ đó tính lượng nhiệt toả ra khi đốt cháy 0,01 mol but-1-
yne trong thí nghiệm trên?
b) Tính số mol các chất có trong bình phản ứng sau khi quá trình đốt cháy xảy ra hoàn toàn. Xem không
khí là hỗn hợp của oxygen và nitrogen với tỉ lệ 1:4.
c) Tính nhiệt độ cực đại trong bình sau phản ứng cháy. Biết:
C4H6(k) CO2(k) H2O(k) O2(k) N2(k)
Nhiệt tạo thành chuẩn ở 298K (kJ/mol) 165,2 -393,5 -241,8 - -
Nhiệt dung đẳng áp (J/mol.K) 13,5 46,6 41,2 32,3 27,6
Giả sử các giá trị nhiệt dung và nhiệt tạo thành không phụ thuộc nhiệt độ.
Câu Đáp án Điểm
3.1 a) Phản ứng xảy ra theo phương trình: C4H6(k) + 5,5 O2(k) → 4 CO2(k) + 3 H2O(k)
0,125
 r H o298 = 4 H so (CO2) + 3 H so (H2O) - H so (C4H6) = -2464,6 kJ/mol.
Ứng với 0,01 mol C4H6 thì nhiệt lượng tỏa ra sẽ là 24,646 kJ. 0,25
b) Tổng số mol khí trước khi nạp but-1-in vào bình: n = PV/RT = 4,090 mol
0,125
n(O2) = 4,090/5 = 0,818 mol, n(N2) = 3,272 mol
Sau khi đốt cháy:
n(N2) = 3,272 mol;
0,25
n(CO2) = 0,01 x 4 = 0,04 mol;
n(H2O) = 0,03 mol;
n(O2) dư = 0,818 – 0,01x5,5 = 0,763 mol.
c) Gọi Tx là nhiệt độ cực đại của bình sau khi quá trình đốt cháy xảy ra hoàn toàn. Do sự đốt
cháy là đoạn nhiệt nên không có sự trao đổi nhiệt với bên ngoài, tức ở đây Q = ∆H = 0.
Từ đó ta có chu trình sau:

0,25

Với H oT = H1o + H o2 + H 3o + H o4
0,125
= 0,04 x 46,6(Tx– 298)+0,03x 41,2(Tx–298) + 0,763 x 32,2(Tx – 298) + 3,272 x 27,6(Tx–298)
Theo chu trình Hess: 0,01. H opu (298K) + H oT = ∆H = 0
0,04 x 46,6(Tx – 298) + 0,03 x 41,2(Tx – 298) + 0,763 x 32,2(Tx – 298) + 3,272 x 27,6(Tx – 0,125
298) = - 0,01. H opu (298K) = 24646
0,25
→ Tx = 507K

3.2. Ở 8200C hằng số cân bằng Kp của các phản ứng như sau:

CaCO3 (r)  CaO (r) + CO2 (k) K1 = 0,2

C(r) + CO2 (k)  2CO (k)
 K2 = 2
Cho 1 mol CaCO3 và 1 mol C vào bình chân không dung tích 22,4 lít duy trì ở 8200C.
a) Tính số mol các chất khi cân bằng.
b) Ở thể tích nào của bình thì sự phân hủy CaCO3 là hoàn toàn?
Câu Nội dung Điểm
3.2 a) CaCO3 (r)  
 CaO (r) + CO2 (k) K1 = 0,2
Ta có K1 = PCO2 = 0,2 atm
0,125
P2
K2 = P CO = 2  PCO  0,632 atm
CO2
Số mol CaCO3 phản ứng = x; số mol CO2 phản ứng = y
 Số mol các chất ở trạng thái cân bằng:
CaCO3 CaO CO2 C CO
1–x x x–y 1–y 2y
PCO2 .V
Số mol CO2 = x – y =  0,05 mol
RT 0,125
P .V
Số mol CO = 2y = CO  0,158 mol
RT
nCaO = 0,129 mol; nCaCO3 = 0,871 mol; nC = 0,921 mol 0,25
Khi sự phân hủy hoàn toàn thì x = 1  nCO2 = 1 – y; nCO = 2y
Vì áp suất CO2 và CO không đổi nên
0,632.V = 2yRT (1) 0,125
0,2.V = (1-y)RT (2) 0,125
Giải hệ (1) (2)  V = 173,69 lít 0,25
(HS giải theo cách khác nếu đúng vẫn đạt điểm tối đa)

Câu 4 (2,5 điểm): Động hóa học (không có cơ chế).


4.1. Phản ứng chuyển hóa của một loại kháng sinh trong cơ thể người ở nhiệt độ 37oC có hằng số tốc độ bằng
4,2.10-5 (s-1). Việc điều trị bằng loại kháng sinh trên chỉ có kết quả nếu hàm lượng kháng sinh luôn luôn lớn
hơn 2,00mg trên 1,00 kg trọng lượng cơ thể. Một bệnh nhân nặng 58 kg uống mỗi lần một viên thuốc chứa
300 mg kháng sinh đó.
a) Hỏi bậc của phản ứng chuyển hóa?
b) Bệnh nhân cần uống viên thuốc thứ hai cách lần đầu bao nhiêu lâu?
c) Khi bệnh nhân sốt đến 38,5oC thì khoảng cách giữa hai lần uống thuốc trên thay đổi như thế nào? Biết
năng lượng hoạt hóa của phản ứng bằng 93,322 kJ.mol-1.
Câu Nội dung Điểm
4.1 a) Đơn vị của hằng số tốc độ là s  thứ nguyên là (thời gian)
-1 -1

1 N 0,25
 phản ứng là bậc nhất: t  ln 0
k N
b) Hàm lượng một viên thuốc là 300 mg (N0), còn hàm lượng tối thiểu để chữa bệnh là
2 x 58 = 116 mg (N). 0,25
1 300
t 5
ln  22623, 6 (s) hay 6,28 giờ. 0,25
4, 2.10 116
c) Coi Ea thay đổi không đáng kể trong khoảng nhiệt độ thay đổi, theo
kT E  1 1 
ln 1  a   
kT2 R  T2 T1 
4, 2.105 93,322.103  1 1  0,25
   k(311,5)  5.105 ( s 1 )
8,314  311,5 310 

Ta có: ln
k(311,5)
Thời gian giữa hai lần uống thuốc là:
0,25
1 300
t 5
ln  19003,85 (s) hay 5,28 giờ (  5 giờ 17 phút)
5.10 116

4.2. Cho phản ứng: 2A  sản phẩm. Đồ thị biểu diễn sự thay đổi của
1/[A] (M-1) với thời gian (phút) ở 200C và 400C như bên:
Ở 200C: 1/[A] = 10 + 0,024t
Ở 400C: 1/[A] = 10 + 0,096t
a) Viết phương trình tốc độ của phản ứng.
b) Tính hằng số tốc độ của phản ứng ở nhiệt độ 200C và 400C.
c) Tính thời gian bán phản ứng ở 200C.
d) Tính năng lượng hoạt hoá (kJ.mol-1) của phản ứng.

Câu Nội dung Điểm


4.2 a) 1/[A] phụ thuộc tuyến tính vào t, vậy phản ứng tuân theo quy luật động học phản ứng 0,125
bậc 2.
Biểu thức tốc độ: v= k[A]2 0,125
0, 024 0,25
k 20oC   4.104 M 1.s 1
60
0, 096 0,25
k 40oC   1, 6.103 M 1.s 1
60
Hoặc ở 20oC : k = 0,024 M-1.phút-1 ở 40oC : k = 0,096 M-1.phút-1
1 1 0,25
c) t1/2(20oC ) = = 4
=25000s
k 20oC [A]o 4.10 .0,10
k 40oC  1 1 
Ea 1, 6.103 E  1 1 
ln     ln 4
 a   
d) k 20oC  T40 T20 
R 4.10 8,314  313 293  0,25
 E a =52850 J/mol  52,850 kJ/mol

Câu 5 (2,5 điểm): Cân bằng acid – base và cân bằng ít tan.

Câu 5 Nội dung Điểm


5.1.a o
Đặt nồng độ của SO2 và Na2SO3 trước khi trộn là C1 = CSO ; C2 = C0 2-
2 SO 3

 sau khi trộn: CSO = 0,5C1; CSO2- = 0,5C2.


2 3
pK a1 + pK a2
Khi chuẩn độ dung dịch A đến pH = 4,4  = 4,485 = pH
2 HSO3-

 thành phần của hệ thu được là HSO3- , nghĩa là trong hỗn hợp A phải có SO2. Như 0,25
vậy sau khi trộn dung dịch SO2 với dung dịch Na2SO3, SO2 còn dư:
SO2 + H2O + SO32   2 HSO3-
0,5C1 0,5C2
0,5(C1 – C2) 0 C2
Thành phần của dung dịch A: SO2 0,5(C1 – C2) M; HSO3- C2 M, môi trường axit 
Khi chuẩn độ dung dịch A đến pH = 4,4 dung dịch sẽ chuyển từ màu đỏ sang màu vàng.
Thêm tiếp phenolphtalein vào hỗn hợp chuẩn độ và chuẩn độ đến pH = 9,0 dung dịch
sẽ chuyển từ màu vàng sang màu hồng.
Chuẩn độ đến pH = 4,4 hệ thu được gồm HSO3- và SO2 (bỏ qua SO32–) trong đó HSO3-
chiếm
[HSO3- ] K a1 101,76
=   0,998  99,8%  100%
[HSO3- ]+[SO 2 ] [H + ]+K a1 10 4,4  10 1,76
=> chuẩn hết nấc 1
SO2 + OH–  HSO3- 0,125
 20,00 . 0,5.(C1 – C2) = 12,50 . 0,20 (1)
Chuẩn độ đến pH = 9,0, hệ gồm HSO3- và SO32- (bỏ qua SO2):
[SO32- ] 10-7,21
=0,984 = 98,4%  100%
K a2
= =
[HSO3- ]+[SO32- ] [H + ]+K a2 10-9,0 +10-7,21
 thành phần của hệ gồm  100% SO32 , chuẩn độ hết nấc 2 0,125
HSO3- + OH–  SO32  + H2O
 20,00 . {0,5(C1 – C2) + C2} = 27,50 . 0,20 (2). 0,25
Từ (1) và (2)  C2 = 0,15 M; C1 = 0,40 M
5.1.b Khi chưa chuẩn độ, pHA chính là pH của hệ đệm:

 HSO3- +
SO 2 + H2O 
 H+ Ka1 = 10-1,76
0,125 – x 0,15 + x x
x.(0,15  x)
 101,76
0,125  x 0,25
 x = [H ] = 0,012  pHA = 1,92
+

5.1.c Gọi độ tan của BaSO3 trong nước là s  s = 0,016 .1000 = 7,36.10-4 (M)
217,33 100

 Ba2+ + SO32 
BaSO3 
 Ks = ? (1)
s s

 HSO3- + OH-
SO32  + H2O 
 Kb1 = 10-6,79 (2)

 SO2 + OH–
HSO3- 
 Kb2 = 10-12,24 (3)
Vì Kb2 << Kb1 nên có thể bỏ qua (3). Xét quá trình (2): 0,25
SO32  + H2O   HSO3- + OH– Kb1 = 10-6,79
C 7,36.10-4
[ ] 7,36.10-4 – x x x  x = 1,08.10-5
Vậy [OH-] = [HSO3-] >> Kb2. Bỏ qua (3) là hợp lý.
[Ba2+] = s = 7,36.10-4 (M); [ SO32  ] = 7,36.10-4 - 1,08.10-5 = 7,25.10-4 (M) 0,25
 Ks = [Ba2+].[ SO32  ] = 5,33.10-7
5.2. a) Tính hằng số cân bằng của phản ứng:
Cr2O72- + H2O 2CrO42- + 2H+
b) Trộn 10,00 mL dung dịch K2Cr2O7 0,80M với 10,00 mL dung dịch A gồm BaCl2 0,08M và SrCl2
0,08M thu được hỗn hợp B. Hãy cho biết hiện tượng xảy ra và thành phần hỗn hợp B.
Cho: CrO42- + H2O HCrO4- + OH- Kb = 10-7,5
2-
Cr2O7 + H2O 2HCrO4- K = 10-1,64
pKa(CH3COOH) = 4,76; pKa(C6H5COOH) = 4,20; pKw(H2O) = 14,0;
pKs(BaCrO4) = 9,93; pKs(SrCrO4) = 4,65.
Câu 5 Nội dung Điểm
5.2.a 2- - -1,64
Cr2O7 + H2O 2 HCrO4 K1 = 10
2 HCrO-4 + OH- CrO 2-
4 + H2O Kb-1 = 107,5
2 H2O H+ + OH- Kw = 10-14
Cr2O72- + H2O 2 CrO 2-
4 + 2H
+
K2 = 10-14,64 0,25

5.2.b Cr2O72- + H2O 2 HCrO-4 K1 = 10-1,64 (1)


Cr2O72- + H2O 2 CrO 2-
4
+ 2H K2 = 10+
(2) -14,64

Sau khi trộn: C(Cr2O7 ) = 0,4M; C(Sr ) = 0,04M; C(Ba2+) = 0,04M.


2- 2+

Trong dung dịch, crom tồn tại dưới 3 dạng là Cr2O72-, HCrO4- và CrO42-.
'
Từ (1) và (2) ta thấy: K1  K2  tính C HCrO- theo (1):
4

Cr2O72- + H2O 2 HCrO-4 K1 = 10-1,64


'
0,4 – x 2x  x = 0,045  C HCrO- = 0,09 M.
4
Thay vào cân bằng: HCrO-4 CrO 2-
4 + H
+
K a =106,5 (3)
009 – y y y
 CCrO2- = y = 1,685.10 M  1,7.10 M.
' -4 -4
4 0,25
'
 CCrO2- . CBa 2+ = 1,7.10 .0,04 = 6,8.10 > KS(BaCrO ) = 10
' -4 -6 -9,93
4 4

 có kết tủa BaCrO4 vàng theo phản ứng (K = 10 ): 5,22


0,25
Cr2O72- + 2 Ba2+ + H2O   2 BaCrO  + 2H+
4 (4)
0,4 0,04
0,38 - 0,04
"
Vì sau phản ứng (4), CCrO2-
4
< 1,7.10-4 M, do đó:
4 6
C"CrO2- .CSr
'
2+  1.7.10 .0, 04  6,8.10  K S(SrCrO )  104,65
4
4 0,25
 không có kết tủa SrCrO4 tách ra.
Vậy hỗn hợp B thu được gồm kết tủa BaCrO4; Cr2O72-dư; Sr2+; H+ tạo thành (K+; Cl-)
Câu 6 (2,5 điểm): Phản ứng oxi hóa – khử. Pin điện (không liên quan phức chất).
6.1. Cho hai pin điện hóa có sơ đồ:
Pin 1: Pt, H2 (1 atm)│HCl 10-3 M│Hg2Cl2, Hg
Pin 2: Pt, H2 (1 atm)│NaOH 10-3 M, NaCl 10-3 M│Hg2Cl2, Hg
Sức điện động của các pin tương ứng là E1 và E2. Biết E oHg 2Cl2 /Hg = 0,2682 V.
a) Viết phương trình hóa học của phản ứng xảy ra tại các điện cực và phản ứng tổng quát khi các pin làm
việc.
b) Tính E1 và thiết lập mối liên hệ giữa E2 và K H2O ở 25oC.
c) Nối hai điện cực calomen của hai pin với nhau để tạo thành một pin kép. Ở 25oC, sức điện động của pin
này là 0,4726 V. Xác định K H2O ở nhiệt độ này.
Câu 6 Nội dung Điểm
6.1.a Các phản ứng:
Pin 1: Tại anot (-): H2 → 2H+ + 2e
Tại catot (+): Hg2Cl2 + 2e → 2Hg + 2Cl- 0,25
Tổng quát: H2 + Hg2Cl2 → 2H+ + 2Cl- + 2Hg
Pin 2: Tại anot (-): H2 + 2OH- → 2H2O + 2e
Tại catot (+): Hg2Cl2 + 2e → 2Hg + 2Cl-
Tổng quát: Hg2Cl2 + 2OH- + H2 → 2Hg + 2Cl- + 2H2O 0,25
6.1.b Sức điện động của pin 1 ở 25 C: o

0, 0592 [H + ]2 [Cl- ]2 [H + ][Cl- ]


E1  E1o  lg  E oHg 2Cl2 /Hg  E o2H+ /H  0, 0592 lg
2 PH2 2
PH2
103.103 0,25
 E1  0, 2682  0, 0592 lg  0, 6234V
1
Sức điện động của pin 2 ở 25oC:
0, 0592 [Cl- ]2 0, 0592 [Cl- ]2
E2  E2 
o
lg  E Hg2Cl2 /Hg  E H 2O/H 2 
o o
lg
2 [OH  ]2 PH2 2 [OH  ]2 PH2 0,25
3 2
0, 0592 (10 )
 E 2  0, 2682  0, 0592 lg K H2O  lg  0, 2682  0, 0592 lg K H2O (V)
2 (103 ) 2 .1
6.1.c Khi nối hai điện cực calomen của hai pin ta sẽ thu được pin kép là pin nồng độ, khi
đó điện cực hidro của pin 1 sẽ trở thành điện cực dương, điện cực hidro của pin 2 là 0,25
điện cực âm (vì nồng độ H+ trong pin 1 lớn hơn pin 2).
Sức điện động của pin kép:
E  (E hidro )pin 1  (E hidro ) pin 2  (E Hg2Cl2 /Hg  E 2H+ /H )  (E Hg2Cl2 /Hg  E H2O/H2 )  E 2  E1
2

 E  0, 2682  0, 0592 lg K H2O  0, 6234  0,3552  0, 0592 lg K H2O


Vì E  0, 4726  0,3552  0, 0592 lg K H O  0, 47262

0,4726  0,3552 0,25


 K H2O  10 0,0592
 1, 04.1014

6.2. Thêm V (mL) dung dịch K2Cr2O7 0,02M vào 100mL dung dịch FeSO4 0,12M (tại pH = 0 và không đổi
trong suốt quá tình phản ứng), thu được dung dịch A. Tính thế khử của cặp Fe3+/Fe2+ trong dung dịch A ở mỗi
trường hợp sau đây:
a) V = 50 mL;
b) V = 100 mL;
Cho biết: E oFe3+ /Fe2+ = 0,771 V; E oCr O2 /2Cr3+ = 1,330 V.
2 7

Câu 6 Nội dung Điểm


6.2. 2-
Trong môi trường axit mạnh, Cr2 O 7 oxi hóa Fe2+ theo phản ứng sau:
6.(1,330 0,771)
6Fe2+ + Cr2 O 7 + 14H+ 
2-   6Fe3+ + 2Cr3+ + H2O K  10 0,0592  1056,66
 (1)
cb

Vì Kcb rất lớn nên coi phản ứng (1) xảy ra hoàn toàn.
a) Khi V = 50 mL
Sau khi cho hết 50 mL dung dịch K2Cr2O7, tính lại nồng độ các chất:
0,12.100
C Fe2+   0,08 (M)
100  50
0,02.50 0,02
C Cr O2   (M)
2 7
100  50 3
2-
Theo (1): 6Fe2+ + Cr2 O 7 + 14H+ → 6Fe3+ + 2Cr3+ + H2O
0,02
Trước phản ứng (M): 0,08
3
0,04
Sau phản ứng (M): 0,04 – 0,04
3 0,25
Ở thành phần giới hạn:
0,04
CFe2+ =0,04 M; CFe3+ =0,04 M; CCr3+ = M
3
Vì phản ứng (1) có Kcb rất lớn nên cân bằng ngược lại có hằng số cân bằng rất nhỏ, coi
như sự phân li ngược lại không đáng kể.
Do đó: [Fe2+ ] = CFe2+ =0,04 M; [Fe3+ ] = CFe3+ = 0,04 M
0,0592 [Fe3+ ]
→ E Fe3+ /Fe2+ = E oFe3+ /Fe2+ + lg 2+ = 0,771 (V)
1 [Fe ]
b) Khi V = 100 mL
Sau khi cho hết 100 mL dung dịch K2Cr2O7, tính lại nồng độ các chất:
0,12.100 0,25
CFe2+ = = 0,06 (M)
100+100
0,02.100
CCr O2- = = 0,01 (M)
2 7
100+100

2-
Theo (1): 6Fe2+ + Cr2 O 7 + 14H+ → 6Fe3+ + 2Cr3+ + H2O
Trước phản ứng (M): 0,06 0,01
Sau phản ứng (M): – – 0,06 0,02
Phản ứng xảy ra vừa đủ, thành phần giới hạn gồm: 0,25
CFe3+ = 0,06 M; CCr3+ = 0,02 M
Vì phản ứng (1) có Kcb rất lớn nên cân bằng ngược lại có hằng số cân bằng rất nhỏ, coi
như sự phân li ngược lại không đáng kể.
Do đó: [Fe3+ ] = CFe3+ = 0,06 M; [Cr 3+ ] = CCr3+ = 0,02 M
2+ 2- 3+ 3+
Tại điểm tương đương, ta có: [Fe ] = 6[Cr2O7 ] ; [Fe ] = 3[Cr ]
0,0592 [Fe3+ ]
Ta tính E Fe3+ /Fe2+ tại điểm tương đương: E Fe3+ /Fe2+ = E oFe3+ /Fe2+ + lg 2+
1 [Fe ]
Khi hệ đạt tới trạng thái cân bằng thì: E Fe3+ /Fe2+ = E Cr O2- /2Cr3+ = E
2 7

Trong đó:
o 0,0592 [Cr2O72- ]
E Cr O2- /2Cr3+ = E + lg
Cr2O72- /2Cr 3+
(vì pH = 0)
2 7
6 [Cr 3+ ]2
 [Fe3+ ] [Cr2 O72- ] 
 7E = E Fe3+ /Fe2+ + 6E Cr O2- /2Cr3+ + 0,0592lg  2+ .
o o
3+ 2 
2 7
 [Fe ] [Cr ] 
E oFe3+ /Fe2+ + 6E Cr
o
O 2- /2Cr 3+ 0,0592  [Fe3+ ] [Cr2O72- ] 
E= 2 7
+ lg  2+ . 3+ 2  0,25
7 7  [Fe ] [Cr ] 
E oFe3+ /Fe2+ +6E oCr O2- /2Cr3+ 0,0592 1
= 2 7
+ lg =1,262 (V)
7 7 2[Cr 3+ ]
Câu 7 (2,5 điểm): Halogen. Oxygen – Sulfur.
7.1. Bằng hiểu biết về liên kết hoá học, hãy giải thích sự biến đổi năng lượng liên kết trong dãy halogen:
Phân tử F2 Cl2 Br2 I2
Năng lượng liên kết X – X (kJ/mol) 159 242 192 150

Câu 7 Nội dung Điểm


7.1 Sự biến đổi năng lượng liên kết trong dãy halogen :
- Năng lượng liên kết X – X trong F2 nhỏ hơn trong Cl2 và Br2 do trong F2 chỉ có xen
phủ p–p còn trong Cl2 I2 ngoài sự xen phủ p –p còn có xen phủ p – d làm tăng độ bền 0,25
liên kết.
- Năng lượng liên kết giảm từ Cl2  I2 là do bán kính nguyên tử tăng dần từ F  I nên
mức độ xen phủ giữa các AO giảm dần  độ bền liên kết X – X giảm từ Cl2  I2 0,25

7.2. Có thể tổng hợp ba chất lỏng không màu H, I, J từ lưu huỳnh (S) theo sơ đồ dưới đây. E và G là những
chất khí độc. Trong điều kiện thường, khối lượng riêng của G là 4,42 gram/L. F là chất lỏng không màu, tan
vô hạn trong nước. Thuỷ phân H, I, J bởi dung dịch kiềm thu được cùng hỗn hợp của các sản phẩm tan trong
nước, phản ứng với dung dịch Ba(NO3)2 dư tạo thành kết tủa K (khối lượng m1). Sau khi gạn kết tủa K, dung
dịch còn lại có thể tạo thành kết tủa L (khối lượng m2) khi phản ứng với lượng dư dung dịch AgNO3. Cả K và
L đều có màu trắng và không tan trong acid. Trong trường hợp của H, tỉ lệ m1/m2 = 0,8118, trong khi đó I và
J có tỉ lệ m1/m2 = 1,624.

a) Xác định các chất E - L, biết rằng H, I, J có các nguyên tử lưu huỳnh (S), còn G thì không.
b) Viết các phương trình phản ứng tổng hợp và thuỷ phân H - J bởi dung dịch NaOH.
c) Xác định công thức cấu tạo của G, H, I, J.
Câu 7 Nội dung Điểm
7.2.a E : SO2; F : SO3; G : COCl2; H : SO2Cl2;
J: (công thức SClOn), n = (6 - 1)/2 = 2,5; I : SO2Cl(OH) hay HSO3Cl
K : BaSO4 L : AgCl 0,125x8
(Xác định đúng mỗi chất: 0,125)
7.2.b S+ O 2   SO 2
2SO 2 + O 2 
Pt
 2SO3
SO 2 + Cl2 
hv
 SO 2Cl 2
0,125x8
SO3 + HCl   HSO3Cl
o
t

SO3 + CCl 4   COCl 2  +S2O5Cl 2


o
t

SO2 Cl2 + 4 NaOH 


 Na2SO4 + 2 NaCl + 2 H 2O
HSO3Cl + 3NaOH 
 Na2 SO4 + NaCl + 2 H 2O
S2 O5Cl2 + 6 NaOH 
 2 Na2SO 4 + 2 NaCl + 3H 2O
(Viết đúng mỗi phương trình: 0,125)
Câu 8 (2,5 điểm): Đại cương hữu cơ (Quan hệ giữa cấu trúc và tính chất)
8.1. Xác định ketone có momen lưỡng cực lớn nhất trong số các ketone sau, giải thích:

A: B: C: D:
Câu 8 Nội dung Điểm
8.1. B và D có momen lưỡng cực lớn hơn do sự hình thành hệ thơm.
Tuy nhiên do khoảng cách điện tích trong B lớn hơn D nên theo công thức
nên momen lưỡng cực của B lớn hơn D. 0,25

0,25
B: D:

8.2. Cho dãy hợp chất sau:

a) So sánh khả năng phản ứng thế electrophin của A với benzene và cho biết vị trí phản ứng ưu tiên ở A.
Giải thích.
b) So sánh nhiệt độ nóng chảy, nhiệt độ sôi của dãy hợp chất trên. Giải thích.
Câu 8 Nội dung Điểm
8.2.a Mật độ e π ở mỗi vị trí của A (6e/5 vị trí) lớn hơn ở mỗi vị trí trong vòng benzen
(6e/6 vị trí) nên A dễ tham gia phản ứng thế electrophin hơn benzen.
- Sự tạo thành phức σ ở vị trí 2 (ở giai đoạn quyết định tốc độ phản ứng) bền hơn ở 0,25
vị trí 3 do điện tích dương

0,25

8.2.b * Nhiệt độ nóng chảy: G > E > D > A > B >C 0,25

* Nhiệt độ sôi: G > E > D > B > C > A 0,25

* Giải thích:
- G, E và D có phân tử khối lớn hơn và có nhiều nhóm phân cực hơn so với A, B, C;
- G tạo liên kết hiđro liên phân tử mạnh hơn E nên tonc và tos đều biến đổi theo 0,125
thứ tự: G > E > D > A, B, C.
0,125
- Ở trạng thái rắn, lực Van deVan (Fv~ p.p’/rn với n ≥ 4) phụ thuộc chủ yếu vào
khoảng cách giữa các phân tử (r). Vì r < r < r nên tonc theo giảm theo thứ tự
A B C
A > B > C. 0,125
- Ở trạng thái sôi, lực Van deVan phụ thuộc chủ yếu vào điện tích p và p’ của
lưỡng cực (vì khi đó khoảng cách giữa các phân tử quá lớn).
0,125
Vì µB> µC> µA nên tos giảm theo thứ tự B > C > A.

8.3. Giải thích vì sao khi khảo sát hợp chất sau thì thấy rằng với nhóm OH nằm ở vị trí liên kết biên equatorial
(e) thì sẽ thể hiện tính acid cao hơn so với lúc nhóm OH nằm ở vị trí liên kết trục axial (a)?

Câu 8 Nội dung Điểm


8.3. Có thể giải thích theo hai hướng
- Nguyên tử hydro của nhóm OH đã bị ràng buộc bởi liên kết hydro nên khả năng 0,25
phân ly giảm
- Anion sinh ra chịu đựng tương tác bất lợi với cặp e tự do trên nguyên tử O trong 0,25
vòng làm giảm tính bền dẫn đến sự giảm tính axit của chất đầu.
ĐỀ SỐ 9
Câu 1: Cấu tạo nguyên tử, phản ứng hạt nhân, định luật tuần hoàn (2,5 điểm)
1.1. Phổ phát xạ của nguyên tử hydrogen cho thấy các vạch quang phổ ở 82259, 97492, 102824,
105292, 106632, và 107440 cm-1, tương ứng với các bước chuyển về trạng thái năng lượng thấp hơn từ trạng
thái năng lượng cao hơn liền kề với n = 2, 3,… Xác định năng lượng ion hóa của nguyên tử hydrogen.
1.2. a) Xác định biểu thức tính giá trị n của một hạt khối lượng m trong hộp thế một chiều có chiều dài
L sao cho sự chênh lệch giữa các mức năng lượng gần nhau bằng với năng lượng trung bình của chuyển động
1
nhiệt (2 kT). Tính giá trị của n trong trường hợp nguyên tử helium trong 1 hộp thế dài 1 cm ở 298 K.
b) Xét 1 hạt trong 1 hộp lập phương. Xác định độ suy biến của mức có năng lượng gấp 3 lần năng
lượng của mức thấp nhất.
Câu Nội dung Điểm
1.1 Đồ thị thể hiện sự tương quan giữa số sóng và 1/n : 2
1,0

Từ phương trình hồi quy tuyến tính, ta thấy đồ thị cắt trục tung tại điểm có số sóng là
109 679 cm-1, hay I/hc = 109 679 cm-1, do đó năng lượng ion hóa là
I = hc×(109 679cm-1)
= (6.626 08×10-34 Js) ×(2.997 945×1010 cm s-1) ×(109 679cm-1)
= 2.1787×10-18 J
1.2.a Các mức năng lượng của 1 hạt trong hộp thế một chiều có chiều dài L được tính theo 1,0
biểu thức En = h2n2/8mL2. Do đó sự chênh lệch năng lượng giữa các mức gần kề nhau
(n và n+1) là:
h2 (n+1)2 h2 n2 h2
ΔE(n) = En+1 -En = − = 8mL2 ((n + 1)2 − n2 )
8mL2 8mL2
h2 h2
= 8mL2 (n2
+ 2n + 1 − n2 ) = 8mL2 (2n + 1)
Để ΔE(n) bằng nhiệt năng trung bình thì h2(2n+1)/8mL2 = kT/2, do đó (2n+1) =
4mkTL2/h2. Suy ra:
2mkTL2 1
n= −
h2 2
Với 1 nguyên tử helium, khối lượng 4.00 u trong 1 hộp dài 1 cm:
n = [2(4.00 × 1.6605 × 10-27 kg) × (1.3806 × 10-23 J K-1) × (298 K)
2
(1 × 10−2 m) 1
× (6,6261×10−34 Js)2 ] − 2 = 1.24 × 1016
1.2.b Biểu thức năng lượng của hạt trong hộp lập phương: 0,5
h2
En1 ,n2,n3 = (n2 + n22 + n23 )
8mL2 1
Với n1, n2, n3 là các số nguyên không nhỏ hơn 1. Do đó, trạng thái năng lượng thấp nhất
có n1 = n2 = n3 = 1, và năng lượng:
h2 2 2 2)
3h2
E(1,1,1) = (1 +1 +1 =
8mL2 8mL2
Suy ra năng lượng của mức có năng lượng cao gấp 3 lần mức thấp nhất là 9h2/8mL2,
hay: n12 + n22 + n23 = 9. Có 3 mức với năng lượng này, (n1, n2, n3) = (1, 2, 2), (2, 1, 2)
và (2, 2, 1) nên độ suy biến là 3.

Câu 2: Cấu tạo phân tử, tinh thể (2,5 điểm)


2.1. Xét hệ liên hợp polyene thẳng với mỗi một carbon trong số N nguyên tử carbon đóng góp một electron
trong orbital 2p, các mức năng lượng 𝐸𝑘 của các orbital phân tử π được tính theo biểu thức sau:
𝑘𝜋
𝐸𝑘 = 𝛼 + 2𝛽𝑐𝑜𝑠 𝑘 = 1,2, … , 𝑁
𝑁+1
Sử dụng biểu thức trên để ước tính giá trị thực nghiệm hợp lý của hằng số tích phân cộng hưởng 𝛽 cho dãy
đồng đẳng sau: ethene, butadiene, hexatriene và octatetraene, cho rằng sự hấp thụ tia cực tím kích thích electron
π trong phân tử chuyển dịch từ HOMO lên LUMO lần lượt ứng với các tần số sóng: 61500, 46080, 39750 và
32900 cm-1.
2.2. Tính năng lượng mạng lưới tinh thể CaO bằng các dữ kiện sau:
∆𝐻/(𝑘𝐽 𝑚𝑜𝑙 −1 )
Nhiệt thăng hoa của Ca(s) +178
Năng lượng ion hóa Ca(g) thành Ca2+(g) +1735
Năng lượng phân li liên kết của O2(g) +498
Ái lực electron của O(g) -141
-
Ái lực electron của O (g) +844
Nhiệt tạo thành của tinh thể CaO(s) -635

Hướng dẫn chấm


Câu Nội dung Điểm
2.1. Các hệ liên hợp polyene thẳng không tồn tại các mức năng lượng suy biến, nên mỗi giá 1,5
trị của k ứng với một MO bị chiếm tối đa bởi 2 electron.
Đối với ethene, có 2 electron pi, HOMO ứng với k = 1, và LUMO ứng với k = 2. Độ
chênh lệch năng lượng HOMO-LUMO là:
2𝜋 𝜋
∆𝐸 = ℎ𝜐 = (𝛼 + 2𝛽𝑐𝑜𝑠 ) − (𝛼 + 2𝛽𝑐𝑜𝑠 ) = (𝛼 − 𝛽) − ( 𝛼 + 𝛽) = −2𝛽
3 3
Với 𝜐 = 61500 cm-1, ta được 𝛽 = −3,07. 104 cm-1.
Đối với butadiene, có 4 electron pi, HOMO ứng với k = 2, và LUMO ứng với k = 3.
Độ chênh lệch năng lượng HOMO-LUMO là:
3𝜋 2𝜋
∆𝐸 = ℎ𝜐 = (𝛼 + 2𝛽𝑐𝑜𝑠 ) − (𝛼 + 2𝛽𝑐𝑜𝑠 ) = −1,23𝛽
5 5
Với 𝜐 = 46080 cm-1, ta được 𝛽 = −3,72. 104 cm-1.
Đối với hexatriene, có 6 electron pi, HOMO ứng với k = 3, và LUMO ứng với k = 4.
Độ chênh lệch năng lượng HOMO-LUMO là:
4𝜋 3𝜋
∆𝐸 = ℎ𝜐 = (𝛼 + 2𝛽𝑐𝑜𝑠 ) − (𝛼 + 2𝛽𝑐𝑜𝑠 ) = −0,89𝛽
7 7
Với 𝜐 = 39750 cm-1, ta được 𝛽 = −4,46. 104 cm-1.
Đối với octatetraene, có 8 electron pi, HOMO ứng với k = 4, và LUMO ứng với k = 5.
Độ chênh lệch năng lượng HOMO-LUMO là:
5𝜋 4𝜋
∆𝐸 = ℎ𝜐 = (𝛼 + 2𝛽𝑐𝑜𝑠 ) − (𝛼 + 2𝛽𝑐𝑜𝑠 ) = −0,695𝛽
9 9
Với 𝜐 = 32900 cm-1, ta được 𝛽 = −4,73. 104 cm-1.
Vậy giá trị trung bình của beta là: 𝛽 = −4,00. 104 cm-1 = −4,96 𝑒𝑉
2.2. Năng lượng mạng lưới tinh thể ∆𝐻𝐿 là biến thiên enthalpy mol chuẩn của quá trình 1,0
MX(s) → M+(g) + X-(g). Giá trị này được xác định gián tiếp bằng chu trình Born-Haber
như sau:
Theo chu trình ta có biểu thức sau:
−635 + ∆𝐻𝐿 = (178 + 1735 + 249 − 141 + 844) 𝑘𝐽 𝑚𝑜𝑙 −1
Vậy ∆𝐻𝐿 = 3500 𝑘𝐽 𝑚𝑜𝑙 −1

Câu 3: Nhiệt hóa học, cân bằng hoá học trong pha khí (2,5 điểm)
Ethanoic acid (acetic acid) được hóa hơi trong một bình có thể tích 21,45 cm3 ở nhiệt độ 437 K và áp
suất bên ngoài là p =101,9 kPa, sau đó bình được hàn kín lại. Khối lượng của acid có mặt trong bình là 0,0519
g. Thí nghiệm được lặp lại cũng ở bình đó nhưng tại 471K, và sau đó trong bình chứa 0,038 g acid. Tính hằng
số cân bằng cho quá trình đime hóa của acetic acid ở thể khí và ∆rHo của phản ứng đime hóa.
Cho M(CH3COOH) = 60,0516 g.mol-1, R = 8,3145 J.K-1.mol-1; áp suất tiêu chuẩn Po = 100 kPa.

Câu Nội dung Điểm


Giả sử số mol ban đầu của CH3COOH là n, độ phân ly là 𝛼 2,5
Ta có cân bằng: 2CH3COOH(g) ⇌ (CH3COOH)2 (g)
Ban đầu n 0
1
Phân ly 𝛼𝑛 𝛼𝑛
2
1
Tại cân bằng (1- 𝛼)𝑛 𝛼𝑛
2
Tổng số mol của hệ: ntotal = (1-1/2𝛼)𝑛
Ta có phương trình khí lý tưởng: pV=ntotalRT
pV=(1-1/2𝛼)𝑛
2𝑝𝑉 2𝑝𝑉
Do đó: 𝛼 = 2- 𝑛𝑅𝑇 = 2- 𝑚 (1)
( )𝑅𝑇
𝑀
Ta có áp suất riêng phần:
1
1−𝛼 𝛼
2
p(CH3COOH)= 1 𝑝; p(CH3COOH)2= 1 p
1− 𝛼 1− 𝛼
2 2
Hằng số cân bằng:
1
𝛼
𝑝(𝐶𝐻3𝐶𝑂𝑂𝐻)2 ( 2 1 𝑝).𝑃𝑜
𝛼(𝐶𝐻3𝐶𝑂𝑂𝐻)2 ( ) 𝑝(𝐶𝐻3𝐶𝑂𝑂𝐻)2.𝑃0 1− 𝛼
𝑃0 2
K= 𝛼2 (𝐶𝐻3𝐶𝑂𝑂𝐻) = 𝑝𝐶𝐻3𝐶𝑂𝑂𝐻 2
= = 1−𝛼
( ) 𝑝(𝐶𝐻3𝐶𝑂𝑂𝐻)2 1 𝑝
𝑃0 1− 𝛼
2

1 1
( 𝛼(1− 𝛼)).𝑃𝑜
2 2
= (1−𝛼)2 .𝑝
(2)
Từ các phương trình (1), (2) ta có thể tính được 𝛼 và K:
2 x (101.9 x 103 Pa) x (21.45 x 10−6 m3 )
Ở 437K: 𝛼 = 2 - 0.0519 = 0,607
( gmol−1 ) x (8.3145 Jmol−1 K−1 ) x (437K)
60.0516

1 1
x (0.607… ) x (1− x 0.607..) x (100kPa)
2 2
K= (1−0.607… )2 x (101.9kPa)
= 1,35
Tương tự: Ở 471K 𝛼 = 0,235
K= 0,175
Ta có phương trình: o
𝐾2 ∆rH 1 1
ln(𝐾1) = − 𝑅 (𝑇2 − 𝑇1)
Với K1 = 1,35; T1 = 437 K
K2 = 0,175; T2 = 471 K
Từ đó: ∆rH° = -103 kJ/mol.

Câu 4: Động hoá học, không có cơ chế (2,5 điểm)


Trong một phản ứng đồng hồ, một sự thay đổi màu rõ rệt xảy ra ở một thời điểm được xác định bởi
nồng độ và nhiệt độ. Xét phản ứng đồng hồ iodine, với phản ứng tổng quát như sau:

2I(aq) + S2 O2− 2−
8(aq) ⟶ I2(aq) + 2SO4(aq)
I2 sau khi được tạo thành sẽ phản ứng ngay với 1 lượng 𝑆2 𝑂32− :
I2(aq) + 2S2 O2− − 2−
3(aq) ⟶ 2I(aq) + S4 O6(aq)
Khi S2 O2−
3 đã hết, lượng I2 dư tạo thành sản phẩm màu xanh đen với tinh bột trong dung dịch:
I2 + tinh bột ⟶ tinh bột ⋅ I2 (xanh đen)
Tốc độ phản ứng cũng bị ảnh hưởng bởi tổng nồng độ của các ion, nên KCl và (NH4)2SO4 được thêm vào để
tổng nồng độ không đổi. Sử dụng dữ liệu sau, ở 23°C, để xác định:
a) Tốc độ trung bình của mỗi thí nghiệm.
b) Bậc riêng phần của các chất phản ứng.
c) Hằng số tốc độ.
d) Biểu thức tốc độ cho phản ứng tổng quát.
Thí nghiệm 1 Thí nghiệm 2 Thí nghiệm 3
0,200 M KI (mL) 10,0 20,0 20,0
0,100 M Na2S2O8 (mL) 20,0 20,0 10,0
0,0050 M Na2S2O3 (mL) 10,0 10,0 10,0
0,200 M KCl (mL) 10,0 0,0 0,0
0,100 M (NH4)2SO4 (mL) 0,0 0,0 10,0
Thời gian chuyển màu (s) 29,0 14,5 14,5

Câu 4 Nội dung Điểm


a) Phản ứng: 1,0

2I(aq) + S2 O2− 2−
8(aq) ⟶ I2(aq) + 2SO4(aq) (1)
2− − 2−
I2(aq) + 2S2 O3(aq) ⟶ 2I(aq) + S4 O6(aq) (2)
Tại thời điểm chuyển màu, iodine phản ứng vừa hết với S2 O2−
3 , nên từ phản ứng (2)
có thể tính được nồng độ I2 sinh ra ở (1).
ΔCI
Biểu thức tốc độ trung bình phản ứng: v = Δt2
- Thí nghiệm 1:
ΔCS2 O2−
3
= 10-3 M suy ra: ΔCI2 =1/2×𝛥CS2 O2−
3
= 5×10-4 M
vTN1 = (5×10-4)/29 = 1,72×10-5 (M/s)
- Thí nghiệm 2:
ΔCS2 O2−
3
= 10-3 M suy ra: ΔCI2 =10-3/2 = 5×10-4 M
vTN2 = (5×10-4)/14,5 = 3,45×10-5 (M/s)
- Thí nghiệm 3:
Vì biến thiên nồng độ S2 O2− 3 và thời gian chuyển màu tương tự thí nghiệm 2 nên
vTN3 = vTN2 = 3,45×10-5 (M/s)
b) Vì thời gian chuyển màu tương đối ngắn và nồng độ của KI hay Na2S2O8 đều lớn hơn 0,75
nhiều so với nồng độ của Na2S2O3 nên xem như tốc độ trung bình của phản ứng (1)
cũng là tốc độ đầu.
Từ thí nghiệm 1 sang thí nghiệm 2, nồng độ I− tăng gấp đôi, còn nồng độ S2 O8 2−
không đổi, và tốc độ phản ứng tăng gấp đôi nên bậc của I − là 1.
Từ thí nghiệm 2 sang thí nghiệm 3, nồng độ I− không đổi, còn nồng độ S2 O8 2− giảm
gấp đôi, và tốc độ phản ứng không đổi nên bậc của S2 O8 2− là 0.
c) Tính lại nồng độ ion iodide: 0,5
CI− (TN1) = 0,04 M ; CI−(TN2) = CI−(TN3) = 0,08 M
k1 = vTN1/CI−(TN1) = (1,72×10-5)/0,04 = 4,3×10-4 s-1
k2 = k3 = (3,45×10-5)/(0,08) = 4,31×10-4 s-1
4,3×10−4 +4,31×10−4 +4,31×10−4
𝑉ậ𝑦: k̅ = ≈ 4,31 × 10−4 s −1
3
d) Biểu thức tốc độ: 0,25
v = 4,31×10-4×CI−

Câu 5: Cân bằng acid-base và hợp chất ít tan (2,5 điểm)


Một bài tập hóa phân tích có đầu bài được cho như sau: “Trộn 15,00 mL dung dịch HCl nồng độ C (mol.L-1 )
với 5 mL dung dịch Na2C2O4 0,100 M, thu được dung dịch X có pH= 1,25. Tính nồng độ C ( mol.L-1)”. Để
tính nồng độ C ( mol.L-1) đó, một học sinh lập luận như sau: “Vì pH = pKa1 = 1,25, suy ra [H2C2O4] = [HC2O4-
] >> [C2O42-], nên hệ thu được là hệ đệm. Như vậy, lượng acid cho vào trung hòa hết nấc 1 và trung hòa hết
1,5.0,100.5,00
nửa nấc 2 của C2O42-, tức là trung hòa hết 1,5 nấc của C2O42-. Do đó, C = = 0,050 (mol.L-1)”
15,00
Bằng các lập luận và tính toán, chô biết học sinh đó giải đúng hay sai.
b) Tính nồng độ C (mol.L-1 ) của dung dịch HCl đã cho.
Nếu thí sinh không tính đúng được ý b, giả sử dung dịch X chỉ chứa H2C2O4 0,016M để tính tiếp.
c) Trộn 5,00 mL dung dịch X với 5,00 mL dung dịch gồm Ca2+ 0,01M và Sr2+ 0,01M. Khi hệ đạt tới trạng thái
cân bằng, cho biết có những kết tủa nào thoát ra. Giả thiết không có sự cộng kết.
Cho biết: pKa1 (H2C2O4) = 1,25; pKa2 (H2C2O4) = 4,27; pKS (CaC2O4) = 8,75; pKS (SrC2O4)=6,40; pKa1 (H2S)
= 7,02; pKa2 (H2S) = 12,90; pKa(NH4+) = 9,24; pKw (H2O) = 14.
Câu 5 Nội dung Điểm
a) Từ nhận xét: “pH= pKa1 = 1,25, suy ra [H2C2O4] = [HC2O4 ] >> [C2O4 ]” là đúng. Từ 1,0
- 2-

mối liên hệ [H2C2O4] = [HC2O4-] học sinh đó suy ra lượng acid cho vào trung hòa hết
1,5 nấc của C2O42- theo phản ứng:
2C2O42- + 3H+ → H2C2O4 + HC2O4-
Nghĩa là, theo học sinh đó, hệ thu được là hệ đệm gồm H2C2O4 (Ca mol.L-1) và
0,100.5,00
HC2O4- (Cb mol.L-1) có cùng nồng độ Ca = Cb = 2.20,00 =0,0125(M). Trong trường
hợp này, đều này là vô lý bởi vì nếu dung dịch X là dung dịch đệm thì pHX có thể
được tính theo một trong 2 cách sau: ( không cần tính theo điều kiện proton vì [H+]
>> [OH-] )
𝐶𝑎
Cách 1: Theo công thức tính pH của hệ đệm: pH = pKa + lg 𝐶𝑏
Cách 2: Tính theo cân bằng:
H2C2O4 ⇌ H+ + HC2O4- Ka1 = 10-1,25
[ ] 0,0125-h h 0,0125+h

Nhưng cả hai cách giải trên đều không hợp lý vì nếu theo cách 1 thì h = [H+] = 10-1,25
M = 0,056 > Ca = Cb, không thỏa mãn điều kiện áp dụng công thức tính pH của hệ
đệm. Còn nếu theo cách 2 thì [H2C2O4] = 0,0125 - h = 0,0125 - 0,056 < 0. Vô lý !
Vậy cách giải của học sinh đó là không đúng.
b) Tính lại nồng độ các chất sau khi trộn: 1,0
5.0,10
CC2O42- = 5+15 = 0,025 (M)
15.𝐶
CHCl = 5+15 = 0,75C (M)
Khi trộn dung dịch HCl với dung dịch C2O42-, lượng HCl có thể chưa đủ để trung hòa
hết hai nấc của C2O42-, cũng có thể trung hòa vừa đủ C2O42- hoặc trung hòa hết C2O42-
và còn dư acid.
Xét trường hợp HCl phản ứng vừa đủ với C2O42-, thu được H2C2O4 0,025M
Khi đó pH của hệ được tính theo cân bằng:
H2C2O4 ⇌ H+ + HC2O4- Ka1 = 10-1,25
[ ] 0,025 - x x x

→ [H+] = x = 0,0187 M → pH = 1,73 > 1,25. Điều này chứng tỏ lượng HCl cho vào
vẫn còn dư, sau khi trung hòa hết 2 nấc của C2O42-. Vậy thành phần giới hạn thu
được gồm:
H2C2O4 0,025 M và H+ (0,75C- 0,05) M
pH của hệ được tính theo cân bằng:
H2C2O4 ⇌ H+ + HC2O4- Ka1 = 10-1,25
[] 0,025 - x 0,75C-0,05+x x

Vì pH = pKa1 → [H ] = Ka1 nên [H2C2O4] = [HC2O4-] hay 0,025 - x = x → x= 0,0125


+

Mặt khác, [H+] = Ka1 → 0,75C - 0,05+x = 10-1,25 → C = 0,125 (M)


(Thí sinh có thể tính C theo định luât bảo toàn điện tích hoặc tính theo điều kiện
proton với mức 0 là HCl và C2O42- :
h = [H+] = [Cl-] - [HC2O4-] - 2[H2C2O4]
15,00.𝐶 0,100.5,00 Ka1.ℎ+ℎ2
→ h = [H+] = - . ℎ2 + 𝐾𝑎1.ℎ + 𝐾𝑎1.𝐾𝑎2 → C = 0,125 (M)
20,00 20,00
c) Thành phần giới hạn của dung dịch X gồm H2C2O4 0,025 M và H+ 0,04375 M 0,5
Thành phần của hệ sau khi trộn: H2C2O4 0,0125 M, H+ 0,0219 M; Ca2+ 0,005 M và
Sr2+ 0,005 M.
Vì CCa2+ = CSr2+ và KS(CaC2O4) < KS(SrC2O4) nên nếu có kết tủa thì CaC2O4 sẽ tách
ra trước. Xét CCa2+, CSr2+ trong đó:
Do môi trường acid và do Ka1>> Ka2 nên C2O42- được tính theo cân bằng:
H2C2O4 ⇌ H+ + HC2O4- Ka1 = 10-1,25
[ ] 0,0125- x 0,0219 + x x

→ [HC2O4-] = x = 8,15.10-3 (M) → [H+] = 0,03 (M)


8.15.10−3
→ CC2O42- = 10-4,27. = 1,46.10-5 (M)
0,03
Vì CCa2+ . CC2O42- = 0,005. 1,46.10-5 = 7,3.10-8 > KS(CaC2O4) = 10-8,75
→ Có kết tủa CaC2O4 tách ra.
Mặt khác, CSr2+ . CC2O42- = 0,005.1,46.10-5 = 7,3.10-8 < KS(SrC2O4) = 10-6,40
→ Không có kết tủa SrC2O4 tách ra.

Câu 6: Phản ứng oxi hoá khử, pin điện (2,5 điểm)

Một cân bằng sinh học quan trọng giữa ion cacbonat và ion hydrocacbonat trong nước tự nhiên.
a) Cho năng lượng Gibbs chuẩn của sự hình thành CO32-(aq) và HCO3-(aq) lần lượt là -527,81 kJ/mol và -
586,77 kJ/mol. Thế điện cực chuẩn của cặp HCO3-/CO32-, H2 là gì?
b) Tính thế điện cực chuẩn của một pin trong đó phản ứng của pin là:
Na2CO3(aq) + H2O(l) → NaHCO3(aq) + NaOH(aq)
c) Viết phương trình Nernst cho pin.
d) Dự đoán và tính sự thay đổi thế điện cực của pin khi pH được thay đổi đến 7.0 ở 298K.
Câu 6 Nội dung Điểm
a) Cân bằng HCO3-(aq) CO32-(aq) + H+(aq) được tách thành hai bán phản ứng : 1,0
1
Ở catot: HCO3-(aq) + e- → 2H2(g) + CO32-(aq)
1
Ở anot: 2H2(g) → H+(aq) + e-
Ta có: E0pin = E0catot – E0anot
HCO3-(aq) CO32-(aq) + H+(aq)
Ta có: 𝛥fG = 𝛥fG (CO3 , aq) - 𝛥fG (HCO3-, aq) với 𝛥fG0(H+, aq) = 0
0 0 2- 0

= -527,81 – (-586,77) = 58,96 kJ/mol


Mà 𝛥rG = -n.F.E0pin nên E0pin = -0,611 (V)
0

Vậy E0catot = E0pin - E0anot với E0anot = 0 (V)


= - 0,611 (V)
b) Na2CO3(aq) + H2O(l) → NaHCO3(aq) + NaOH(aq) 0,5
Phản ứng trên có thể được viết gọn thành
CO32-(aq) + H2O(l) → HCO3-(aq) + OH-(aq)
và được tách thành hai bán ứng:
1
Ở catot: H2O(l) + e- → 2H2(g) + OH-(aq)
1
Ở anot: 2H2(g) + CO32-(aq) → HCO3-(aq) + e-
Ta có: E0pin = E0catot – E0anot
= -0,83 – (-0,611) = 0,22 (V)
c) Phản ứng trong pin ở câu b, là 0,5
CO32-(aq) + H2O(l) → HCO3-(aq) + OH-(aq)
Vậy phương trình Nernst là:
RT [HCO3 ][OH  ]
E pin  E pin
o
 ln
F [CO32 ]
d) Hiệu điện thế chuẩn của pin tương ứng với các chất tham gia vào phản ứng trong pin, 0,5
bao gồm OH-. Điều này có nghĩa là độ pH sẽ cần xấp xĩ 14, để cho [OH-] = 1. Ở độ pH
= 7.0, nồng độ OH- sẽ thấp hơn ở độ pH = 14, điều đó có nghĩa là phản ứng trong pin
được viết ở trên sẽ có xu hướng chuyển dịch về phía trước tốt hơn. Do đó Epin sẽ được
dự đoán lớn hơn ở pH = 7.0 so với khi [OH-] = 1.
Giả sử nồng độ của các chất khác vẫn như cũ, sự thay đổi của hiệu điện thế của pin khi
chuyển từ [OH-] = 1 sang pH = 7.0 là:
 o RT [HCO3 ][OH  ]   o RT [HCO3 ].1 
E pin   E pin  ln 
  pin
E  ln 
 F [CO32 ]   F [CO32 ] 
RT RT RT
 ln[OH  ] = -2,303 .lg[OH  ] = 2,303 pOH
F F F
Mà pH = 7 nên pOH = 7, vậy ΔEpin = 0,0592.7 = 0,414 V
Vậy thế điện cực của pin tăng từ [OH-] = 1 đến pH = 7.

Câu 7: Halogen, oxygen – sulfur (2,5 điểm)


Một trong các hợp chất được sử dụng phổ biến trong quá trình quá trình khử trùng bằng chlorine của hồ bơi
là sodium hypochlorite, NaClO.
a) Xác định trạng thái oxi hóa của chlorine trong sodium hypochlorite, NaClO.
Sau khi hòa tan, một trạng thái cân bằng được thiết lập giữa ClO- và acid liên hợp của nó
b) Hãy viết phương trình cho trạng thái cân bằng này.
Trạng thái cân bằng này phụ thuộc rất nhiều vào pH và dưới điều kiện có acid thì chlorine được tạo thành.
c) Hãy viết phương trình cho sự tạo thành của khí chlorine từ hypochlorite và HCl.
Các nhà tổ chức giải bơi lội cuối cùng đã giải thích về màu xanh trong hồ là do sự phát triển của tảo sau khi
vô tính bổ sung them một lượng hydrogen peroxide, điều này triệt tiêu hypochlorie và tạo ra các ion
chloride.
d) Hãy viết phương trình cho phản ứng giữa hydrogen peroxide và hypochlorite.
Các hypochlorite cũng có xu hướng phản ứng với ammonia và hợp chất giống ammonia để tạo ra các hợp
chất chứa nitrogen và chlorine. Hợp chất như vậy là nitrogen trichloride, chất có thể gây ra kích ứng ở mắt
và mùi đặc biệt của hồ bơi.
e) (i) Hãy viết phương trình cho sự tạo thành của nitrogen trichloride.
(ii) Vẽ cấu trúc của nitrogen trichloride, cho biết hình dạng của nó, và nêu gần đúng góc liên kết
Cl-N-Cl.
Tuỳ thuộc vào tỉ lệ phản ứng, một sản phẩm khác của phản ứng giữa ammonia và hypochlorite là hydrazine,
H2N-NH2, và ion chloride
f) Hãy viết phương trình cho phản ứng này.
Đồng(II) sulfate thỉnh thoảng bổ sung vào hồ bơi và điều này được đề xuất là nguyên nhân của màu xanh lá
cây. Ion đồng(II) cũng được cho là nguyên nhân gây ra màu xanh lục cho mái tóc bị phai màu của vận động
viên bơi lội người Mỹ Ryan Lochte. Ion đồng (II) kết tủa trên tóc bởi vì pH cao của một số dầu gội.
g) Hãy đề xuất một công thức cho kết tủa xanh trên tóc của Ryan Lochte làm cho nó chuyển sang màu xanh.
Câu 7 Nội dung Điểm
a) 0,125
+1
b) H 2 O + ClO- HOCl + OH - 0,25
c) HOCl + HCl → Cl2 + H2O 0,25
hoặc
ClO- + Cl- + 2H+ → Cl2 + H2O
d) H2O2 + ClO- → Cl- + H2O + O2 0,25
hoặc
H2O2 + NaClO → NaCl + H2O + O2
e) i) NH3 + 3NaOCl → NCl3 + 3NaOH 0,5
hoặc
NH3 + 3HOCl → NCl3 + 3H2O
e) ii) Lưỡng tháp tam giác 0,25
f) 2NH3 + NaOCl → N2H4 + NaCl + H2O 0,5
hoặc
2NH3 + HOCl → N2H4 + HCl + H2O
g) Cu(OH)2 0,125

Câu 8: Đại cương hữu cơ: quan hệ giữa cấu trúc và tính chất (2,5 điểm)
8.1. Đối với decalin có hai cách sắp xếp khác nhau trong không gian là cis-decalin và trans-decalin như sau:

decalin trans-decalin cis-decalin


a) Vẽ cấu dạng ghế của cis-decalin và trans-decalin.
b) Đồng phân trans thì bền hơn, giải thích.
8.2. Giải thích các kết quả thực nghiệm sau:
a) Calicene là một hydrocarbon nhưng có moment lưỡng cực lớn tới 5,6 D.

b) Moment lưỡng cực của furan và pyrole có hướng ngược nhau.

Hướng dẫn chấm


Câu Nội dung Điểm
8.1
a) Vẽ cấu dạng 0,5

Cấu dạng trans Cấu dạng cis

b) Đồng phân trans bền hơn vì đồng phân cis có tương tác 1,3-điaxial. 0,5
8.2.
a) Sở dĩ calicene có moment lưỡng cực lớn như vậy là do phân tử này tồn tại cấu trúc cộng 0,5
hưởng bền do tạo ra hai vòng có tính thơm mang điện tích.

b) Có ba yếu tố chính tạo nên moment lưỡng cực cho 2 phân tử đã cho: 1,0
(1) các liên kết σ phân cực C-O và C-N,
(2) các electron π không định cư hướng từ O hoặc N vào vòng,
(3) cặp electron chưa tham gia liên kết trên O và liên kết N-H.
Hai yếu tố đầu tiên gây ra chiều moment lưỡng cực giống nhau cho cả hai phân tử, tuy
nhiên yếu tố thứ 2 của pyrrole mạnh hơn. Bên cạnh đó, liên kết N-H của pyrrole tạo
moment có chiều ngược lại so với cặp electron chưa tham gia liên kết của O. Những
điều này làm cho hướng của moment lưỡng cực tổng của chúng ngược nhau.

ĐỀ SỐ 10
Câu 1:
1. Các đồng vị phóng xạ có thể được dùng trong các chẩn đoán y học và chữa bệnh cũng như dùng trong phân
tích công nghiệp. Đồng vị phóng xạ 32P có một vai trò quan trọng trong các nghiên cứu sinh học với chu kỳ
bán hủy t1/2 là 14,3 ngày. Đồng vị phóng xạ 32P có thể được dùng để xác định thể tích nước trong hồ bơi hay
thể tích máu trong động vật. Lấy 2,0 ml dung dịch chuẩn chứa 32P với hoạt độ phóng xạ 1,0 Ci/ml được đưa
vào hồ bơi. Sau khi trộn đều thì hoạt độ Ax của 1,0 mL nước trong hồ bơi được xác định là 12,4 Bq (1 Ci =
3,7.1010Bq). Tính thể tích nước trong hồ bơi.
2. Trong một hồ nước, tốc độ phân hủy phóng xạ 222Rn (thời gian bán hủy, t 1/2 = 3,8 ngày) được xác định
là 4,2 nguyên tử.phút -1.(100 L)-1. Toàn bộ lượng 222Rn được sinh ra từ quá trình phóng xạ của 226Ra (t1/2 =
1600 năm) hòa tan có hoạt độ phóng xạ 6,7 nguyên tử.phút-1.(100 L)-1. Hoạt độ phóng xạ không thay đổi
theo thời gian.
Biết rằng mỗi nguyên tử 226Ra phân rã cho một nguyên tử 222Rn, thế nên sự hao hụt trong hoạt độ của
222
Rn ám chỉ đến việc 222Rn đã bị mất khỏi hồ nước do một lượng đáng kể 222Rn đã thất thoát do một quá
trình chưa rõ.
a. Tính nồng độ 222Rn trong hồ theo đơn vị nguyên tử.(100L) -1 và mol.L-1.
b. Nếu quá trình chưa rõ ấy tuân theo quy luật động học bậc nhất thì hãy tính hằng số tốc độ của quá
trình này (phút -1).
c. Hãy thử đoán xem quá trình chưa biết này là một quá trình vật lý, hóa học hay sinh học. Giải thích
lý do.
Hướng dẫn chấm
1. Do lượng 32P trước và sau khi đưa vào dung dịch đều không đổi
 Độ phóng xạ của 32P trong 2 ml bằng độ phóng xạ của 32P trong cả hồ bơi:
 Ao.Vo = Ax. Vx
12, 4
 1.2  .Vx
3, 7.1010
 Vx  5,97.109 (ml )  5,97.103 (m3 )
2.
a. Thời gian bán hủy của 222Rn là 3,8 ngày = 5472 phút.
Hằng số tốc độ phóng xạ của 222Rn: kRn = ln2/5472 = 1,266.10-4 ph-1.
Do tốc độ phân rã của 222Rn là 4,2 nguyên tử.phút -1.(100 L)-1 ta có:
kRn. CRn = 4,2 nguyên tử.phút-1.(100 L)-1
CRn = 3,3.104 nguyên tử.(100L)-1 = 5,5.10-22 mol/L
b. Do hoạt độ của 222Rn không thay đổi theo thời gian nên quá trình sinh ra Rn qua sự phân rã của 226Ra
phải cân bằng với quá trình phân rã phóng xạ của chính nó và sự mất mát do quá trình chưa biết.
Dựa trên định luật bảo toàn khối lượng ta có :
(kRn + kqt chưa biết). CRn = 6,7 nguyên tử.phút-1.(100 L)-1 → kqt chưa biết = 0,75. 10-4 ph-1
c. Do Rn là một khí trợ nên chỉ có thể chịu ảnh hưởng bởi các yếu tổ vật lý như nhiệt độ và áp suất trong
hồ. (Thật ra sự mất Rn chính do sự khuếch tán các phân tử Rn trên bề mặt thoát ra môi trường ngoài).
Câu 2:
1. Tetrasunfutetranitua có dạng tinh thể màu cam, trên 130°C sẽ bị phân hủy nổ khi va đập để tạo thành các
nguyên tố. Trong phản ứng đó có tạo thành trung gian sunfunitrua SN có đời sống ngắn, chất này có thể đóng
vai trò như một phối tử tạo phức, ví dụ [RuCl4(H2O)NS]−.
a. Vẽ giản đồ obitan phân tử của SN và tính bậc liên kết.
b. Nếu cho tetrasunfutetranitrua tiếp xúc với bột bạc ở nhiệt độ 300°C trong chân không thì sẽ tạo thành
disunfudinitrua. Biết rằng disunfudinitrua có tính thơm. Vẽ hai cấu trúc cộng hưởng của chất này.
2. Cấu trúc tinh thể hợp chất cacbua (carbide) của kim loại kiềm thổ MC2 (M
= Mg, Ca, Sr và Ba) được nghiên cứ trong hai thập kỷ gần đây. Kết quả nhiễu
xạ tia X cho thấy: ở nhiệt độ phòng, tinh thể của chúng có cấu trúc kiểu NaCl
biến dạng. Các cation M2+ tạo thành mạng lập phương tâm diện biến dạng theo
môt trục tinh thể (trục z, hình vẽ), còn anion C22 chiếm tất cả các hốc bát diện
của mạng này. Trong mạng tinh thể của MC2 (M = Ca, Sr, Ba), các anion C22
định hướng song song với phương biến dạng. Với mạng tinh thể của MgC2,
trong một lớp vuông góc với phương biến dạng, các anion C22 song song với
nhau và song song với trục x hoặc trục y (hình vẽ); các anion C22
a. Vẽ ô mạng tinh thể của MgC2 và CaC2. Chú ý rằng, trong mỗi mạng
tinh thể MC2, sự phối trí của các cation M2+ hoàn toàn tương đồng.
b. Độ dài liên kết C-C trong mạng tinh thể MgC2, CaC2, BaC2 lần lượt là 1,215 Å, 1,200 Å và 1,186 Å.
Giải thích sự khác biết giữa các giá trị độ dài liên kết C-C trong các hợp chất MC2 (M = Mg, Ca, Ba).
c. Ở nhiệt độ cao, tinh thể MC2 (M = Ca, Sr, Ba) chuyển sang trạng thái tinh thể mới với kiểu cấu trúc
NaCl chuẩn. Nhiệt độ chuyển pha của CaC2, SrC2 và BaC2 lần lượt là 490oC, 370oC và 250oC. Trong khi đó,
tinh thể MgC2 phân hủy ở 497oC và không chuyển pha. Giải thích hiện tượng này.
Hướng dẫn chấm
1. a. Giản đồ Obitan phân tử của phân từ SN:
6 1
Bậc liên kết của phân tử SN: N =  2,5
2
b. Do phân tử có tính thơm nên S2N2 có cấu trúc vòng.
Công thức của S2N2 là:

2. a. + Cấu trúc mạng tinh thể MgC2:

Hai cách biểu diễn trên là tương đương nhau. Vẽ đúng một trong hai tinh thể thì cho điểm tối đa.
+ Câu trúc mạng tinh thể của CaC2:
b. Khi bán kính cation M2+ giảm từ Ba tới Mg, mật độ điện tích dương trên M2+ tăng và M2+ sẽ làm phân cực
hóa ion C22- mạnh hơn (tính cộng hóa trị trong tương tác nhiều hơn). Điều này làm cho anion bị biến dạng
mạnh hơn và có kích thước lớn hơn. Do đó độ dài liên kết C-C trong anion C22- tăng dần khi từ mạng tinh thể
BaC2 đến MgC2.
c. - Do tương tác tĩnh điện có tính định hướng thấp, nên khi cung cấp nhiệt trong không gian tinh thể MC 2
tới một nhiệt độ nào đó, anion C 22- sẽ dao động nhiệt như nhau theo tất cả các phương trong không gian
tinh thể. Khi đó, anion sẽ có tính đối xứng cầu và hệ quả là tinh thể chuyển từ kiểu mạng NaCI biến dạng
về kiểu mạng NaCl chuấn.
- Tính cộng hóa trị trong tương tác cation-anion trong mạng MC 2 càng nhỏ, nhiệt độ chuyến pha càng
thấp. Như trong câu trên đã giải thích, tính cộng hóa trị trong MC2 giảm dần trong nhóm từ M= Mg đến
M = Ba. Do đó, nhiệt độ chuyển pha giảm. Cụ thể là CaC 2 (490°C), SrC2 (370°C) và BaC2 (250°C).
Trong trường hợp của MgC 2, sự chuyển pha tinh thể sẽ xảy ra ở nhiệt độ cao hơn nhiều so với 490°C và
do đó cao hơn nhis độ phân hủy của nó (497°C). Vì vậy, tinh thể MgC 2 phân hủy trước khi quá trình
chuyển pha xảy ra.

Câu 3:
1. Cubane (C8H8) với khối lượng riêng 1.29 g/cm3 là hydrocarbon
“đặc” nhất. Khi đốt cháy một đơn vị thể tích cubane, năng lượng
giải phóng lớn hơn gấp 1.58 lần so với việc đốt cháy cùng một đơn
vị thể tích hợp chất thơm styrene (C8H8) (0.909 g/cm3), gấp 1.80 lần
so với octane (C8H18) (0.703 g/cm3) và gấp 6.06 so với hydrogen
lỏng (0.070 g/cm3).
a. Tính năng lượng giải phóng (kJ/dm3) khi đốt cháy H2, cho
biết enthalpy tạo thành của CO2 và C8H18 lần lượt là -393.9 kJ/mol
và -250.0 kJ/mol.
b. Tính enthalpy đốt cháy và tạo thành (kJ/mol) của cubane và styrene.
c. Giải thích tại sao enthalpy tạo thành của cubane và styrene khác nhau dù hai chất này có cùng công
thức phân tử.
2. Heptane (“hep”) và iso-octane (2,2,4-trimethyl pentane, “oct”). đóng vai trò quan trọng trong các động cơ
đốt trong. Dưới đây là một số dữ liệu Hóa-lí ở nhiệt độ 298 K. Trong phản ứng đốt cháy, sẽ tạo thành H2O (k).
+ heptane, lỏng: ρ = 0.680 g.cm-3; biến thiên enthalpy tạo thành ΔHf° = -224.4 kJ/mol,
+ iso-octane, lỏng: ρ = 0.692 g.cm-3; biến thiên enthalpy đốt cháy ΔHc° = -44328 kJ/kg
ΔHf° (CO2) = -393.5 kJ mol-1; ΔHf° (H2O(k)) = -241.8 kJ mol-1.
a. Tính biến thiên enthalpy chuẩn của phản ứng cháy (ΔHc°) heptane và biến thiên enthalpy tạo thành
chuẩn (ΔHf°) của iso-octane theo kJ/mol.
Việc kiểm tra các hỗn hợp của n-heptane và iso-octane có tầm quan trọng đặc biệt. Một loại nhiên liệu có
khả năng chống kích nổ giống hỗn hợp kiểm tra oct-hep thì có giá trị RON (“chỉ số octane nghiên cứu”) bằng
phần trăm octane về thể tích trong hỗn hợp. Iso-octane tinh khiết có RON = 100, heptane có RON = 0. Xét
một hỗn hợp kiểm tra với RON = 93, nghĩa là có phần trăm thể tích octane là 93%.
b. Tính khối lượng riêng và biến thiên enthalpy đốt cháy chuẩn của hỗn hợp kiểm tra theo gam/L.
c. Tính entropy chuẩn và năng lượng tự do Gibbs chuẩn khi trộn 100 mL hỗn hợp ở 298 K.
3. Các hydrat của axit nitric rất được chú ý do nó xúc tác cho quá trình dị thể tạo thành các lỗ thủng ozone ở
Nam cực. Worsnop đã tiến hành nghiên cứu sự thăng hoa của monohydrat, dihydrat và trihydrat của axit nitric.
Kết quả được thể hiện bởi các thông số nhiệt động sau đây ở 220K
∆Go, kJ/mol ∆Ho, kJ/mol
HNO3.H2O(r) → HNO3(k) + H2O(k) 46,2 127
HNO3.2H2O(r) → HNO3(k) + 2H2O(k) 69,4 188
HNO3.3H2O(r) → HNO3(k) + 3H2O(k) 93,2 237
a. Tính ∆Go của các phản ứng này ở 190K (là nhiệt độ của vùng cực). Giả sử ∆Ho và ∆So không thay đổi
theo nhiệt độ.
b. Hydrat nào sẽ bền vững nhất ở 190K nếu áp suất của nước là 1,3.10-7 bar và áp suất HNO3 là 4,1.10-10
bar. Biết áp suất tiêu chuẩn là 1 bar.
Hướng dẫn chấm:
1. a. Gọi x (kJ/dm3) là nhiệt của quá trình đốt cháy khí H2.
Ta dễ dàng thấy được ΔH oc (H2 ) = ΔH fo (H2O)  x (kJ/dm3 ) .
x
Như vậy ΔH oc (H2 ) = ΔH fo (H 2O)   0, 0286 x(kJ/mol)
1000.0, 07
2
Do khi đốt cháy một đơn vị thể tích cubane, năng lượng giải phóng lớn hơn gấp 1,80 lần so với việc đốt cháy
cùng một đơn vị thể tích hợp chất octane (C8H18) (0.703 g/cm3) và gấp 6,06 so với hydrogen lỏng (0.070 g/cm3)
nên:
6, 06 6, 06 x
ΔH oc (C8H18 ) = . x (kJ/dm3 )  .  0,5459 x (kJ/mol)
1,8 1,8 1000.0, 703
114
Giá trị này là nhiệt của phản ứng:
C8H18 + 25/2 O2 → 8CO2 + 9H2O ΔH oc (C8H18 )
Như vậy ta có:
ΔH oc (C8H18 )  8ΔH fo (CO2 )  9 ΔH fo (H 2O)  ΔH fo (C8H18 )
 8.(393,9)  9.0, 0286 x  (250)  0,5459 x (kJ/mol)
 x  10052 (kJ/dm3 )
b. ΔH of (H2O)  0, 0286 x  287,506(kJ/mol)
6, 06 6, 06 x
ΔH oc (Cubane) = . x (kJ/dm 3 )  .  4918,39(kJ/mol)
1 1 1000.1, 29
104
6, 06 6, 06 x
ΔH oc ( Stirene ) = . x (kJ/dm 3 )  .  4411, 297(kJ/mol)
1,58 1,58 1000.0,909
104
Khi đốt cháy: C8H8 + 10 O2 → 8CO2 + 4H2O
ΔH oc (Cubane) = 617,166(kJ/mol)
ΔH oc ( Styrene ) = 110, 073(kJ/mol)
c. Hai yếu tố chính tạo nên sự khác biệt là sức căng vòng đáng kể trong cấu trúc Cubane và tính thơm của
vòng styrene.
2.
a. Biến thiên enthalpy chuẩn của phản ứng cháy heptane:
C7H16 + 11 O2 → 7CO2 + 8H2O
ΔH c (C7 H16 )  7 ΔH f (CO2 )  8ΔH f (H2O)  ΔH fo (C7 H16 )  4464,5(kJ/mol)
o o o

Biến thiên enthalpy hình thành chuẩn iso octane:


C8H18 + 25/2 O2 → 8CO2 + 9H2O
ΔH c (C8H18 )  44328(kJ/kg). 0,11426 (kg/mol)  5053,39 (kJ/mol)
o

ΔH oc (C8H18 )  8ΔH fo (CO2 )  9 ΔH fo (H 2O)  ΔH fo (C8H18 )


 ΔH of (C8H18 )  8ΔH fo (CO2 )  9 ΔH fo (H2O)  ΔH co (C8H18 )  270,81 (kJ/mol)
b. Khối lượng riêng của hỗn hợp kiểm tra là:
d = 0,07.0,68 + 0,93.0,692 = 0,691 (g/cm3) = 691 (g/L)
Tính biến thiên enthapy đốt cháy chuẩn của hỗn hợp kiểm tra.
Xét trong 1 lít hỗn hợp khí kiểm tra ta có:
mhep = 47,6 (g) miso = 643,6 (g)
→ nhep = 0,476 (mol) niso = 5,646 (mol)
→ xhep = 0,078 xiso = 0,922 (mol)
ΔHc hh  xhep .ΔH c hep  xiso .ΔH co iso
o o


 0, 0776.(4464,5)  0,9222.(5053,39)  5018 (kJ/mol)
c. Tổng số mol khí trong 100 ml hỗn hợp khí: nhh = 0,0476 + 0,5646 = 0,6122 (mol)
ΔG tron
o
 nRT ( xhep .ln xhep  xiso .ln xiso )
 0, 6122.8,314.298(0, 078.ln 0, 078  0,922.ln 0, 922)  415,378 (J/mol)
ΔS o
tron   nR ( xhep .ln xhep  xiso .ln xiso )
 0, 6122.8,314(0, 078.ln 0, 078  0,922.ln 0,922)  1,394 (J/mol)
3.
a. Ta có biểu thức ΔGo = ΔHo - TΔSo
Ở 220K: ΔGo220 = ΔHo - 220ΔSo → ΔSo = (ΔHo - ΔGo220)/220
Ở 190K: ΔGo190 = ΔHo - 190ΔSo → ΔSo = (ΔHo - ΔGo190)/190
 190  190
Từ hai phương trình trên ta thấy rằng: ΔG190 o
 ΔH o 1    ΔG 220 .
o

 220  220
 190  190
Với dạng monohidrat: ΔG190,mono
o
127 1    46, 2.  57, 2(kJ/mol)
 220  220
 190  190
Với dạng dihidrat: ΔG190,di
o
188 1    69, 4.  85, 6(kJ/mol)
 220  220
 190  190
Với dạng dihidrat: ΔG190, tri  237 1    93, 2.  112,8(kJ/mol)
o

 220  220
b. Ta có: ΔG = ΔG o + RTlnQ = ΔG o + RTln(p nH2O .p HNO3 )
Với dạng monohidrat:
ΔG mono = ΔG omono + RTlnQ = 57, 2+ 8,314.190.10 3.ln(1,3.107 .4,1.1010 )  2, 0 (kJ/mol)
Với dạng dihidrat:
ΔG di = ΔG dio + RTlnQ = 85,6+ 8,314.190.10 3.ln((1,3.107 ) 2 .4,1.1010 )  1,36 (kJ/mol)
Với dạng trihidrat:
ΔG tri = ΔG otri + RTlnQ = 112,8+ 8,314.190.10 3 .ln((1,3.107 )3 .4,1.1010 )  3,52 (kJ/mol)
Từ các giá trị về ΔG ta thấy rằng trong điều địa cực, HNO3 tồn tại ở dạng trihidrat là bền nhât.
Câu 4:
1. H2O2 phản ứng với ion I− trong môi trường axit theo phương trình sau:
H2O2 (aq) + 3I−(aq) + 2H+(aq) → I3−(aq) + H2O (l)
Động học của phản ứng này đã được nghiên cứu bởi sự giảm nồng độ H2O2 và xây dụng các đồ thị
ln[H2O2] theo thời gian. Tất cả các đồ thị đều có dạng tuyến tính và toàn bộ các dung dịch H2O2 trong thí
nghiệm đều có [H2O2]o = 8,0.10-4 M. Hệ số góc (a) của các đường thẳng này phụ thuộc vào nồng độ đầu của
I− và H+ được cho trong bảng sau:
Thí nghiệm [I−]o (M) [H+]o (M) Hệ số góc (a) (phút-1)
1 0,1000 0,0400 -0,120
2 0,3000 0,0400 -0,360
3 0,4000 0,0400 -0,480
4 0,0750 0,0200 -0,076
5 0,0750 0,0800 -0,118
6 0,0750 0,1600 -0,174
Biểu thức động học của phản ứng có dạng:
d  H 2 O2 
 
 k1  k2  H +   I   H 2O2 
m
v
n

dt
a. Xác định bậc của phản ứng theo [H2O2] và [I−].
b. Tính giá trị hằng số tốc độ k1 và k2.
2. Phẩm mẫu xanh Brilliant Blue FCF (kí hiệu là E133) được sử
dụng nhiều trong công nghiệp thực phẩm. Trong dung dịch nước,
E133 bị oxi hóa bởi nước Javel theo phản ứng:
E133+ ClO− → Sản phẩm không màu
Động học của phản ứng này được nghiên cứu bằng cách theo
dõi biển thiên nồng độ E133 theo thời gian (nhờ phương pháp phân
tích quang học). Kết quả cho thấy phản ứng có bậc động học.
Thí nghiệm 1: Trộn 25,0 mL dung dịch E133 có nồng độ C1 =
4,545.10-6 M với 1,0 mL dung dịch NaClO nồng độ C2 = 1,360.10-2M. Kết quả theo dõi nồng độ E133 theo
thời gian ở 298 K như sau:

t (phút) 2,5 5,0 7,5 10,0


CE133 (10-6M) 2,222 1,129 0,575 0,292

a. Chứng minh rằng trong điều kiện thí nghiệm, phản ứng tuân theo quy luật động học bậc 1.
b. Tính hằng số tốc độ biểu kiến của phản ứng và thời gian nửa phản ứng trong điều kiện thí nghiệm.
Thí nghiệm 2: Trộn 25,0 mL dung dịch E133 có nồng độ C3 = 5,200.10-6 M với 1,0 mL dung dịch NaClO
nồng độ C4 = 8,500.10-3 M. Kết quả theo dõi nồng độ E133 theo thời gian ở 298 K như sau:

t (phút) 4,1 8,2


-6
CE133 (10 M) 2,50 1,25
c. Chỉ ra rằng trong điều kiện thí nghiệm 2, bậc của phản ứng không thay đổi so với thí nghiệm 1 và tính
hằng số tốc độ phản ứng trong điều kiện này.
d. Từ kết quả thu được ở hai thí nghiệm trên hãy cho biết ngoài E133, tốc độ phản ứng còn phụ thuộc
vào nồng độ của chất nào khác? Xác định bậc riêng phần của chất đó. Từ đó tính hằng số tốc độ của phản
ứng nghiên cứu ở 298 K nếu các chất phản ứng được lấy theo đúng hệ số tỉ lượng của phương trình phản
ứng.
Hướng dẫn chấm
1. a. Trong tất cả các thí nghiệm, nồng độ H2O2 nhỏ hơn rất nhiều so với nồng độ của I− và H+, do đó nồng
độ của I− và H+gần như là hằng số và biểu thức có thể được viết lại thành v  kobs  H 2 O2  , trong đó
n

 
m
kobs  k1  k2  H +   I  
do tất cả các đồ thị ln[H2O2] theo thời gian đều có dạng tuyến tính nên phản ứng có bậc 1 theo H2O2 (n=1).
 
m
Hệ số góc (a) của đồ thì ln[H2O2] theo thời gian là kobs  k1  k2  H +   I   .
Để xác định bậc I−, so sánh hệ số góc ở hai thí nghiệm (1) và (2)
a2 0,360   k1  k2 .0, 040  0,300
m

   3m  3  m  1
a1 0,120   k1  k2 .0, 040  0,100 m

Vậy phản ứng là bậc 1 theo nồng độ I−.


b. Xét thí nghiệm 1, hệ số góc a1 = -0,120 (phút-1)
 a1  kobs
   k1  k2 .0, 04  .0,1  0,120
 k1  k2 .0, 04  1, 20 (1)
Xét thí nghiệm 4, hệ số góc a4 = -0,076 (phút-1)
 a4   kobs
   k1  k2 .0, 02  .0, 075  0, 076
 k1  k2 .0, 02  1, 01 (2)
 k  0, 082
Từ (1) và (2)   1
k2  9,500
Vậy k1 = 0,082 M-1phút-1, k2 = 9,5 M-1phút-1
2. Ta có: v = k[E133]x[ClO−]y.
Xét thí nghiệm 1, sau khi trộn hai dung dịch:
4,545.106.25 1,36.102.1
CE133 (1)   4,370.106 (M) CClO (1)   5, 231.104 (M)
26 26
a. Do CE133 >> CClO− nên sự thay đổi nồng độ ClO- trong phản là không đáng kể, khi đó v =k’.[E133]x với k’
= k[ClO−]y.
1 C
Giả sử phản ứng tuân theo quy luật động học bậc 1 với nồng độ của E133, ta có: k(1) '
 ln o
t C
Từ bảng số liệu ta tính được:
k1' (1) k2' (1) k3' (1) k4' (1)
0,2705 0,2707 0,2704 0,2706
Từ bảng số liệu trên ta thấy rằng k1 ≈ k 2 ≈ k 3 ≈ k 4 nên phản ứng trên phù hợp với quy luật động học bậc 1.
' ' ' '

Vậy trong thí nghiệm 1, phản ứng tuân theo quy luật động học bậc 1.
b. Hằng số tốc độ phản ứng biểu kiến ở thí nghiệm 1:
k1' (1)  k2' (1)  k3' (1)  k4' (1)
'
k(1)   0, 2706 (phút-1)
4
ln 2
Thời gian nửa phản ứng: t1/2  = 2,56 (phút)
k'
c. Xét thí nghiệm 2, sau khi trộn hai dung dịch:
5, 200.106.25 8,5.103.1
CE133 (2)   5, 0.106 (M) CClO (2)   3, 27.104 (M)
26 26
1 C
Giả sử phản ứng ở thí nghiệm 2 vẫn tuân theo quy luật động học bậc 1, ta có: k '(2)  ln o
t C
Từ bảng số liệu ta tính được: k1 (2)  0,169 (phút )
' -1
k2 (2)  0,169 (phút-1)
'

Vậy trong thí nghiệm 2, phản ứng vẫn tuân theo quy luật động học bậc 1.
Hằng số tốc độ phản ứng biểu kiến ở thí nghiệm 2: k(2) '
 0,169 (phút-1)
d. Từ kết quả thu được ở hai thí nghiệm, ngoài E133, tốc độ phản ứng còn phụ thuộc vào nồng độ của
ClO−.
Ở thí nghiệm 1, k(1) '
 k[ClO  ](1)
y

Ở thí nghiệm 2, k(2)


'
 k[ClO  ](2)
y

'
k(1) k[ClO  ](1)
0, 2706 (5, 231.104 ) y
y

  
  y 1
'
k(2) k[ClO  ](2)
y
0,169 (3, 27.104 ) y
 phản ứng là bậc 1 đối với ClO−
k'
Hằng số tốc độ của phản ứng nghiên cứu ở 298 K: k  
 516,82 M-1phút-1
[ClO ]
Câu 5:
Cho dung dịch X gồm H3PO4 C (mol/l) và HA 0,01 M.
a. Tính nồng độ của H3PO4 và hằng số cân bằng của axit HA, biết rằng độ điện ly của H3PO4 và HA trong
dung dịch X lần lượt là 0,443 và 1,95.10-4
b. Thêm dần dung dịch NH3 vào dung dịch X đến nồng độ 0,16 M (coi thể tích không đổi khi thêm NH3)
được dung dịch B. Tính pHB.
c. Trộn 5 ml dung dịch B với 5 ml dung dịch Mg(NO3)2 0,03 M. Bằng các phép tính cụ thể, hãy cho biết
có kết tủa tách ra không? Tính pH của hệ thu được.
Cho pKa(H3PO4)= 2,15; 7,21; 12,32; pKa(NH4+)= 9,24; pKs(MgNH4PO4) = 12,6; pKs(Mg(OH)2 = 10,9.
Hướng dẫn chấm
1.
H3PO4    H+ + H2PO4−
 Ka1
H2PO4−    H+ + HPO42-
 Ka2
HPO42− 
 H+ + PO43-

 Ka3
HA 
 H+ + A−

 Ka
H2O 
 H+ + OH−

 Kw
Do Ka1 >> Ka2 >> Ka3 nên xem H3PO4 chỉ phân ly nấc 1.
Do  H3PO4  HA và  HA 1 nên xem như H3PO4 quyết định pH.
[H 2 PO4 ] K a1
Ta có:  H3PO4    0, 446  h  8,90126.103
CH3PO4 h  K a1
Ka
  HA   K a  1, 7361.106 (M)
h  Ka
H3PO4  H+ + H2PO4−
 Ka1
BĐ Ca
CB Ca – h h h
 H +   H 2 PO 4  h 2
K a1 =   102,15
 H3PO4  Ca  h
 Ca  0, 02M
b.
H3PO4 + NH3  NH 4 + H 2 PO 4 K = 107,09
C0 0,02 0,16M
C - 0,14 0,02 0,02
HA + NH3  NH 4 + A  -
K = 103,48
C0 0,01 0,14M 0,02
C - 0,13 0,03 0,01
H 2 PO 4 + NH3  NH 4 + HPO 24 
 
K = 102,03
C0 0,02 0,13M 0,03
C - 0,11 0,05 0,02
HPO 4 0, 02M
2

 
 A 0, 01M
 TPGH ddB 
 NH 3 0,11M
 NH 4 0, 05M
Do CNH3 .Kb NH3  C A .Kb A  CHPO2 .Kb HPO 2  K w nên sự phân li của NH3 sẽ quyết định pH của hệ
4 4

[NH3 ] 0,11
 pH  pK a  log 
 9, 24  log  9,58
[NH 4 ] 0, 05
c. C NH  0, 025M;C NH  0, 055M;CA  0, 005M;C HPO  0, 01M;C Mg  0, 015M

3
 2 2
4 4

1  0, 02.K a1.K a 2 .K a3  5
CPO3  .  3   1,819.10 (M)
4
2  h  h .K a1  K a1.K a 2 h  K a1.K a 2 .K a3 
2

Để xuất hiện kết tủa:


KS
MgNH 4 PO4 : CMg2 (1)   5,5245.107 (M)
C NH .CPO3
4 4

K
Mg(OH)2 : CMg2 (2)  2 S  8, 614.103 (M)
COH
 Thứ tự xuất hiện kết tủa (nếu có) là MgNH4PO4 sau đó đến Mg(OH)2.
Do CMg  C1  có kết tủa MgNH4PO4.
2

Mg2+ + NH3 + HPO 24  MgNH4PO4


C0 0,015 0,055 0,01
C 0,005 0,045
 sau khi xuất hiện kết tủa MgNH4PO4 thì nồng độ NH3 giảm nên nồng độ OH- giảm và
CMg  0, 005M  CMg (2)  không có kết tủa Mg(OH)2.
2 2

0, 045
 pHhệ = 9,24 + log = 9,5.
0, 025
Câu 6:
1. Pin điện hóa dưới dây dựa trên phản ứng ở pha rắn và hoạt động thuận nghịch ở 1000K dưới dòng khí O2(k).
Các ion F ̶ khuếch tán thông qua CaF2(r) ở 1000K:
(-) MgF2(r), MgO(r) | CaF2(r) | MgF2(r) , MgAlO4(r), Al2O3(r) (+)
a. Viết các phản ứng xảy ra trên mỗi điện cực và phản ứng tổng quát xảy ra trong pin điện hóa trên.
b. Viết phương trình Nernst cho mỗi nửa phản ứng trên hai điện cực, cho phản ứng tổng quát xảy ra trong
pin và tính suất điện động của pin. Coi rằng áp suất O2(k) là như nhau ở cả hai điện cực và được duy trì bởi
dòng khuếch tán ion F ̶ thông qua CaF2(r).
c. Suất điện động chuẩn của pin trong khoảng nhiệt độ từ 900K đến 1250K là:
Eo = 0,1223 + 3,06.10-5T (V)
Giả sử ∆H và ∆S không đổi trong khoảng nhiệt độ này. Tính giá trị ∆Ho và ∆So.
o o

2. Pin galvanic đầu tiên được A.Volta chế tạo vào năm 1800, dựa vào những thí nghiệm của L.Galvani. Sau
này, các pin galvanic đã được ứng dụng rộng rãi trong khoa học, công nghệ và cuộc sống thường ngày. Nửa
bên trái của pin (cực ấm) chứa một điện cực sắt (dư) bị oxi hoá trong quá trình hoạt động và dung dịch iron(III)
nitrate có nồng độ 0.01 M. Nửa bên phải của pin (cực dương) chứa điện cực than chì và hỗn hợp iron(II) và (III)
nitrat với nồng độ lần lượt là 0.05 M và 0.30 M. Thể tích của mỗi nửa pin là 1 L.
a. Viết sơ đồ pin, phản ứng xảy ra ở mỗi điện cực và phản ứng tổng quát xảy ra trong pin.
b. Biết rằng entropy chuẩn của Fe(s), Fe2+(aq), Fe3+(aq) ở 25oC lần lượt là 27,3 J/mol, -137,7 J/mol và -
316.0 J/mol và khi tăng nhiệt độ thêm 20oC sẽ làm giảm hằng số cân bằng K đi 85 lần. Tính ΔHo, ΔSo, ΔGo ở
25oC của phản ứng tổng quát. Chấp nhận rằng ΔHo và ΔSo là các hằng số trong khoảng nhiệt độ này, R = 8.314
J/mol.K, F = 96485 C/mol (hằng số Faraday).
c. Tính suất điện động ban đầu của pin và các thế ban đầu của catot, anot ở 25oC.
Biết rằng E oFe2 / Fe  -0.447 V.
Hướng dẫn chấm
1. a. Phản ứng xảy ra ở hai điện cực:
+ điện cực âm: MgO(r) + 2F− → MgF2(r) + ½ O2(k) + 2e
+ điện cực dương MgF2(r) + Al2O3(r) + ½ O2(k) + 2e → MgAlO4(r)
Phản ứng tổng quát xảy ra trong pin:
MgO(r) + Al2O3(r) → MgAlO4(r)
b. Phương trình Nerst cho mỗi nửa điện cực:
RT pO1/22 (  )
+ điện cực âm: E  E 
o
ln
2F  F   2
  ()
RT pO1/22 (  )
+ điện cực dương: E  Eo  ln
2F  F   2
  ()
Phương trình Nerst của phản ứng xảy ra trong pin là:

RT pO1/22 (  ) RT pO1/22 (  )
E pin  E  E  E o
 ln  E 
o
ln
2F  F   2 2F  F   2
  ()   ()
 1/2 1/2 
E pin  (E  E ) 
o o RT  ln pO2 (  )  ln pO2 (  ) 
   F  2  F   
2
2F   
 () () 

 p1/2  F   2 
RT  O2 (  ).   () 
E pin  ( Eo  Eo )  ln
2 F   F  . p1/2 
  2

   (  ) O2 (  ) 
Do áp suất O2(k) là như nhau ở cả hai điện cực và được duy trì bởi dòng khuếch tán ion F ̶ thông qua CaF2(r)
nên E pin  Eo  Eo
c. Ta có: ΔGo = ΔHo - TΔSo = -nF(0,1223 + 3,06.10-5T)
= -nF.0,1223 + -nF.3,06.10-5T
ΔHo = -nF.0,1223 = -2.96500.01223 = 23604 (J/mol)
ΔSo = -nF.3,06.10-5 = -2.96500.3,06.10-5 = 5,906 (J/mol.K)
2.
a. Sơ đồ pin:
(+) Fe | Fe3+ || Fe2+ , Fe3+| C (−)
Phản ứng xảy ra trong pin:
Tại Anot (-): Fe3+ + 3e → Fe
Tại Catot (-): Fe3+ + e → Fe2+
Phản ứng tổng quát: Fe3+ + Fe → Fe2+
b. Xét phản ứng tổng quát: Fe3+ + Fe → Fe2+
ΔSo = 3.(-137,3) – 2.(-316,0) – 27,3 = 191,9 (J/K)
G o   RT ln K  H o  T S o
K 2 H o  1 1 
Do  ln    
K1 R  T2 T1 
TT K 298.318
 H o  R. 1 2 ln 2  8,314. ln 85  175011( J / mol )
T1  T2 K1 20
G o  H o  T S o  175011  298.191, 6  232107 ( J )
c.
Fe2+ + 2e → Fe E1o (1)
Tại Anot (-): Fe3+ + 3e → Fe E2o (2)
Tại Catot (+): Fe3+ + e → Fe2+ E3o (3)
RT  Fe 
3

E+= E  o
ln  E3o  0, 046 (V)
 Fe 
2
3
F

RT  Fe 
3

E−= E2 
o
ln  E2o  0, 039 (V)
F  Fe 
2

Epin = E  E  E3o  E2o  0, 085 (V)


G pu
o

Ta có: G pu
o
 nFE pin
o
 E pin
o
  E pin
o
 E3o  E2o  0,802 (V) (*)
nF
Suất điện động của pin là: Epin = 0,802 + 0,085 = 0,887 (V)
Ta có: pt(1) = pt(2) - pt(3)
 G1o  G2o  G3o
 n1 FE1o  n2 FE2o  n3 FE3o
 2 E1o  E3o  3E2o  2.(0, 447) (**)
 E  0, 756 (V)
o
 E  0, 756  0, 046  0,802(V)
Từ (*) và (**)   3
 
 E  0, 046 (V)  E  0, 046  0, 039  0, 085(V)
o
2

Câu 7:
1. Cho sơ đồ chuyển hóa sau:
Các phân tử và ion từ A đến L ở sơ
đồ trên đều bắt nguồn từ nguyên tố
Y kết hợp với nguyên tố oxi và có
thể có hydro. Hợp chất B là một
chất có tầm quan trọng trong kỹ
thuật, nó ngưng tụ để cho một chất
rắn giống nước đá ở nhiệt độ dưới
17°C. Nó là nguyên liệu cơ bản để
tổng hợp một axit mạnh H. Hợp
chất B không thể được điều chế trực
tiếp bằng cách đốt cháy trực tiếp
nguyên tố Y mà chỉ có thể bằng
cách oxy hóa có xúc tác hợp chất A
là một khí có mùi khó chịu.
Khí này khi phản ứng với nước sẽ tạo thành một axit có độ mạnh vừa phải, nó không thể được phân lập
thành dạng tự do từ dung dịch nước. Nếu khí A phản ứng với magie hydroxit thì thu được dung dịch chứa ion
D được dùng để sản xuất xenlulo bằng phương pháp sôi. Bằng cách gắn axit Lewis A vào bazơ Lewis E thì
thu được ion F. Muối kali của F được dùng để tẩy uế, khi kết hợp với nước thì sẽ tạo ra khí A.
Hợp chất A có thể phản ứng với tác nhân khử mạnh để cho axit G gồm ba nguyên tố, trong đó %mS
= 49,23%. Anion của axit G có tính khử mạnh trong dung dịch kiềm và có khả năng tạo kết tủa bạc từ dung
dịch bạc nitrat.
Trong sơ đồ trên thì nếu ta điện phân anion của hợp chất H sẽ thu được hợp chất I có công thức H2Y2O8.
Thủy phân hợp chất I thì sau khi loại bỏ axit H sẽ thu được axit caro J.
Anion K tạo được một phức phổ biến với bạc.
Hãy xác định nguyên tố Y và công thức các chất và ion từ A đến L.
2. Các hợp chất từ A1 đến A6 được tạo thành bởi hai nguyên tố M, N được chuyển hóa theo sơ đồ sau:

Hiệu hàm lượng các nguyên tố M và N, cùng một số tính chất vật lí của các hợp chất này, được cho trong bảng
sau đây:
Chất A1 A2 A3 A4 A5 A6
%mM ̶ %mN 5,2 63,2 19,3 5,2 -15,0 -22,4
khí màu vàng khí màu chất lỏng
Tính chất chất lỏng Không bền chất lỏng
da cam vàng – lục đỏ thắm
a. Xác định các chất từ A1 đến A6 và vẽ cấu trúc của chúng.
b. Xác định X, các gốc Y, Z và viết tất cả các phương trình phản ứng đã trình bày trong sơ đồ.
c. Viết phương trình phản ứng giữa Na2[Pb(OH)4] và A4.
Câu 7:
1.
- Nguyên tố Y là Lưu huỳnh (S)
- Các chất và ion trong sơ đồ trên là:
A: SO2 B: SO3 C: H2SO3 D: HSO3− E: SO32-
2-
F: S2O5 G: H2S2O4 H: H2SO4 I: H2SO8 J: H2SO5
2- 2-
K: S2O3 L: S4O6
2.
a. Các chất từ A1 đến A6 là:
Chất A1 A2 A3 A4 A5 A6
Công thức Cl2O4 Cl2O Cl2O3 ClO2 Cl2O6 Cl2O7

Cấu tạo

b. X là Cl2, Y là gốc ClO4−, Z là gốc ClO3−


Các phương trình phản ứng như sau:
(1) AgClO4 + Cl2   AgCl + Cl2O4
(2) 2Cl2 + 3HgO   Cl2O + Hg3O2Cl2
(3) AgClO3 + Cl2   AgCl + Cl2O3
(4) 3Cl2 + 6KOH   5KCl + KClO3 + H2O
o
90 C

(5) 2KClO3 + H2C2O4 + H2SO4   K2SO4 + 2ClO2 + 2CO2 + H2O


(6) 2ClO2 + 2O3   Cl2O6 + 2O2
  3KClO4 + O2
o
400
(7) 4KClO3
(8) KClO4 + H2SO4   KHSO4 + HClO4
o
t

(9) 2HClO4 + P2O5   Cl2O7 + HPO3


o
t

3. Na2[Pb(OH)4] + 2ClO2   PbO2 + NaClO2 + H2O


Câu 8:
1. Cho các chất sau từ A1 tới A6:

a. Những phân tử nào có tính thơm, phản thơm, không thơm theo quy tắc Huckel. Giải thích?
b. Chất A5 lại không phân cực, chất A4 có momen lưỡng cực bằng 1,8D. Giải thích?
2. Cho chất hữu cơ A.
a. Chất hữu cơ A có tồn tại hai đồng phân hình học là A1 (cis) và A2 (trans). Hãy
xác định cấu trúc của A.
b. Khi hòa tan 2 đồng phân A1 và A2 vào dung dịch axit sunfuric 60% thì chỉ có đồng
phân A1 tạo dung dịch có màu vàng đậm còn đồng phân A2 cho dung dịch không màu.
Giải thích hiện tượng trên.
3. Sự deproton hóa thioacetal A sau đó cho cacbanion tạo thành bắt Deuterium từ nước
nặng thì quá trình chọn lọc sẽ xảy ra cao ở vị trí biên. Giải thích kết quả của sự chọn lọc vị trí của phản ứng
này.

Hướng dẫn chấm


1. a. Các chất A1, A3, A4, A5 đều là những hệ liên hợp kín với số eπ thỏa mãn quy tắc 4n+2 nên là những
phân tử có tính thơm.

Chất A6 là hệ liên hợp kín với số eπ thỏa mãn quy tắc 4n nên là
những phân tử có tính phản thơm.
Chất A2 là hệ có 10eπ tuy nhiên hiệu ứng không gian của hai
nguyên tử H đẩy nhau làm phân tử không phẳng, không tạo ra một
hệ liên hợp kín nên chất không có tính thơm.
2.
a. Cấu trúc hai đồng phân của A

b. Khi cho hai đồng phân này tác dụng với dung dịch axit sunfuric 60% thì hai chất A1 và A2 đều tạo
cacbocation A11 và A12.
Sự tạo thành cacbocation của đồng phân cis được ổn định bằng obitan π của liên kết đôi cộng thêm
việc có nhóm phenyl làm tăng sự xuất hiện màu dẫn đến dung dịch đồng phân cis có màu vàng. Ngược lại
đồng phân trans không có sự hỗ trợ làm bền cacbocation từ vòng nên dung dịch không có màu.
3. Phản ứng ở đây trải qua hai quá trình, quá trình đầu tiên n-BuLi sẽ deproton hóa ở một trong hai vị trí trục
và biên (chất trung gian A1 và A2).

Trong hai chất trung gian A1 và A2, chất trung gian A1 có xảy ra sự tương tác giữa cặp electron trên
nguyên tử C với obtian phản liên kết C* S trong khi chất trung gian gian A2 không có tương tác này. Điều này
làm cho sản phẩm trung gian A1 bền hơn A2.
Do BuLi là một tác nhân bazơ ưu tiên sản phẩm theo sự khống chế nhiệt động tức là tạo ra sản phẩm
trung gian bên hơn nên sẽ ưu tiên tạo ra sản phẩm trung gian A1. Điều này dẫn đến quá trình bắt Deuterium từ
nước nặng thì quá trình chọn lọc sẽ xảy ra cao ở vị trí biên.
------------HẾT------------

You might also like